abcdefgh

¡Supera tus tareas y exámenes ahora con Quizwiz!

A patient states that his ears felt stopped up all day today. He became aggravated and pinched his nose and blew out forcefully. The nurse practitioner diagnoses a ruptured left tympanic membrane. What would indicate this? A. Bright red blood in the external canal B. Pain in the affected ear C. Clear fluid in the affected external canal D. Absence of hearing in the left ear

A. Bright red blood in the external canal - This patient ruptured his tympanic membrane traumatically. It is not unusual to find bright red blood, though no active bleeding, in the external canal.

Shelby has a blister filled with clear fluid on her arm. It is the result of contact with a hot iron. How do you document this? A. Bulla B. Wheal C. Cyst D. Pustule

A. Bulla

Which of the following cancers is associated with Epstein-Barr virus? A. Burkitt's lymphoma B. Kaposi's sarcoma C. Lymphoma D. Adult T-cell leukemia

A. Burkitt's lymphoma - Kaposi's sarcoma is associated with human cytomegalovirus - Lymphoma is associated with the human herpesvirus 6 - Adult T-cell leukemia is associated with the human T-lymphotropic viruses

Which of the following underlying lung diseases is the most common cause of a secondary spontaneous pneumothorax? A. COPD B. Lung abscess C. Cystic fibrosis D. Tuberculosis

A. COPD

In older adults, the most common cause of decreased visual functioning is A. Cataract formation B. Glaucoma C. Macular degeneration D. Arcus senilis

A. Cataract formation

Harvey is taking theophylline for his COPD. Which of the following increases the clearance rate and might indicate the need for a higher dosage of theophylline to be ordered? A. Cigarette smoking B. Hepatic insufficiency C. Allopurinol (Zyloprim) D. Cimetidine (Tagamet)

A. Cigarette smoking

The nursing diagnosis of "impaired gas exchange" may be demonstrated by A. Clubbing of the fingers B. Nasal flaring C. The use of accessory muscles D. A cough

A. Clubbing of the fingers

Clients with AIDS typically experience the neurological symptomatic triad consisting of A. Cognitive, motor, and behavioral changes B. Seizures, paresthesias, and dysesthesias C. Kaposi's sarcoma, cryptococcal meningitis, and depression D. Seizures, depression, and paresthesias

A. Cognitive, motor, and behavioral changes - All are potential symptoms of AIDS, but the key word is neurological.

Susan, a new mother, states that when she pusher her index finger on one of her baby's skull bones, it presses in and then returns to normal when she removes her finger. She is concerned about this. You tell her that it is common and is called A. Craniotabes B. Molding C. Caput succedaneum D. Cephalhematoma

A. Craniotabes

Susan, a new mother, states that when she pusher her index finger on one of her baby's skull bones, it presses in and then returns to normal when she removes her finger. She is concerned about this. You tell her that it is common and is called A. Craniotabes B. Molding C. Caput succedaneum D. Cephalhematoma

A. Craniotabes - This condition corrects itself in a matter of months without treatment

Which of the following is not an effective strategy to prevent the nausea and vomiting associated with the effects of radiation and chemotherapy? A. Decreasing the amount of liquids B. Eating a soft, bland diet low in fat and sugar C. Relaxation D. Distraction

A. Decreasing the amount of liquids - It is important to maintain an adequate fluid intake to prevent dehydration, which may result in vomiting.

Which of the following is a predisposing condition for furunculosis? A. Diabetes mellitus B. Hypertension C. Peripheral vascular disease D. Chronic fatigue syndrome

A. Diabetes mellitus - Predisposing conditions for furunculosis or carbuncles include diabetes mellitus, HIV, and injection drug use

You have a new client, Robert, age 67, who present with a generalized lymphadenopathy. You know that this is indicative of A. Disseminated malignancy, particularly of the hematological system B. Cancer of the liver C. Sjorgen's syndrome D. Pancreatic cancer

A. Disseminated malignancy, particularly of the hematological system - Such as lymphoma or leukemia

You note a completely split uvula in Noi, a 42 year old Asian. What is your next course of action? A. Do nothing B. Refer Noi to a specialist C. Perform a throat culture D. Order a CBC

A. Do nothing

A child with slanted eyes with inner epicanthal folds; a short, flat nose; and a thick, protruding tongue probably has A. Down syndrome B. hypertelorism C. craniosynostosis D. Marfan's syndrome

A. Down syndrome

A client with HIV infection has a fever of unknown origin (FUO). Which of the following is a possible cause of an FUO in a client with HIV? A. Drug fever B. Upper respiratory infection C. Nothing specific; this is a systemic disease manifestation D. Urinary tract infection

A. Drug fever - A fever of unknown origin in clients with HIV is defined as temperature > 101 on multiple occasions over 4 weeks duration in an outpatient and 3 weeks duration in an inpatient, with an uncertain diagnosis after three appropriate investigations of cultures and the like.

Which bacterial infection in children appears as a firm, dry crust, surrounded by erythema that exudes purulent material? A. Ecthyma B. Impetigo C. Bullous impetigo D. Cellulitis

A. Ecthyma

Which shape of the thorax is normal in an adult? A. Elliptical B. Funnel C. Pectus carinatum D. Barrel

A. Elliptical - Th enormal adult has a thorax that has an elliptical shape with an anteroposterior:transverse diameter ratio of 1:2 or 5:7.

Which shape of the thorax is normal in an adult? A. Elliptical B. Funnel C. Pectus carinatum D. Barrel

A. Elliptical - Th enormal adult has a thorax that has an elliptical shape with an anteroposterior:transverse diameter ratio of 1:2 or 5:7.

When examining a client's chest wall, you note crepitus. What condition might you suspect? A. Emphysema B. Pleuritis C. Pulmonary edema D. Pneumonia

A. Emphysema

Hyperresonance on percussion of the chest occurs with A. Emphysema B. Pneumonia C. Pleural effusion D. Lung tumor

A. Emphysema - Found when too much air is present. A dull sound on percussion indicates an abnormal density in the lungs, such as occurs with pneumonia, pleural effusion, a lung tumor, or atelectasis

Marcia, age 4, is brought into the office by her mother. She has a sore throat, difficulty swallowing, copious oral secretions, respiratory difficulty, stridor, and a temperature of 102 F but no pharyngeal erythema or cough. What do you suspect? A. Epiglottitis B. Group A beta-hemolytic streptococcal infection pharyngitis C. Tonsillitis D. Diphtheria

A. Epiglottitis

Which disease usually starts on the cheeks and spreads to the arms and trunk? A. Erythema infectiosum (fifth disease) B. Rocky mountain spotted fever C. Rubeola D. Rubella

A. Erythema infectiosum (fifth disease)

Marjorie has a Bartholin's cyst. What is the most common offending pathogen? A. Gonococcus B. Staphylococcus aureus C. Streptococcus faecalis D. Escherichia coli

A. Gonococcus

The most common mode of transmission of the common cold in adults is A. Hand to hand transmissions B. Persons coughing into the air C. Environmental pollutants D. Unclean food utensils

A. Hand to hand transmissions

Regular ocular pressure testing is indicated for older adults taking A. High-dose inhaled glucocorticoids B. NSAIDs C. Angiotensin converting enzyme (ACE) inhibitors D. Insulin

A. High-dose inhaled glucocorticoids

Impetigo is characterized by: A. Honey-colored crusts B. Silvery scales C. Marble-like lesions D. Wheals

A. Honey-colored crusts

Which is the most common localized infection of one of the glands of the eyelids? A. Hordeolum B. Chalazion C. Bacterial conjunctivitis D. Herpes simplex

A. Hordeolum

A patient is diagnosed with tinea pedis. A microscopic examination of the sample taken from the infected area would likely demonstrate: A. Hyphae B. Yeast C. Spores D. A combination of hyphae and spores

A. Hyphae - Under microscopic exam, hyphae are long, thin and branching and indicate dermatophyte infections. Hyphae typical in tinea pedis, cruris, & corporis.

Tara was born with a cleft lip and palate. When should treatment begin for this condition? A. Immediately after birth B. At age 3 months C. At age 6 months D. When Tara is ready to drink from a cup

A. Immediately after birth

A 70 year old patient states that her father and mother both had hearing loss that began in their sixth decades. She has begun having hearing loss. Which complaint below is typical of presbycusis? A. Inability to hear consonants. B. Asymmetrical loss of hearing C. Inability to hear low pitched sounds D. Pulsatile noise in the ear

A. Inability to hear consonants - Presbycusis is influenced by genetics as well as noise exposure, medications, and infections. Loss of ability to hear speech in crowded rooms or noisy areas, inability to understand consonants, and loss of high-pitched sounds is typical. Hearing loss is symmetrical. Asymmetrical hearing loss is a red flag regardless of the age at which it occurs.

Martin, age 76, has just been given a diagnosis of pneumonia. Which of the following is an indication that he should be hospitalized? A. Inability to take oral medications and multilobar involvement on chest x-ray B. Alert and oriented status, slightly high but stable vital signs, and no one to take care of him at home C. Sputum with gram-positive organisms D. A complete blood count (CBC) showing leukocytosis

A. Inability to take oral medications and multilobar involvement on chest x-ray - Severe vital sign abnormality (pulse rate greater than 140 beats per minute, systolic blood pressure less than 90mm Hg, or a respiratory rate greater than 30 breaths per minute) is also an indication for hospitalization.

The gold standard for diagnosing pneumonia on chest x-ray is: A. Infiltrates B. Interstitial fluid C. Cavitation D. "Pooling"

A. Infiltrates - The finding of infiltrates on chest x-ray, in conjunction with clinical findings of fever, chest pain, dyspnea, and sputum production on clinical exam should direct the examiner to consider pneumonia as the diagnosis.

Sally has HIV infection and asks which method of birth control, other than abstinence, would be best for her. You suggest A. Latex condoms B. The spermicide nonoxynol-9 C. An intrauterine device (IUD) D. An oral contraceptive

A. Latex condoms

The use of which of the following medications contribute to the deveopment of acne vulgaris A. lithium B. propranolol C. tetracycline D. oral contraceptives

A. Lithium

Mark, age 72, has been living in a shelter for 4 months. Today he appears at the clinic complaining of productive cough, weight loss, weakness, anorexia, night sweats, and generalized malaise. These have been bothering him for 8 weeks. What would be one of the first tests you order? A. Mantoux test B. Chest X-ray C. Complete blood count D. Sputum culture

A. Mantoux test - All tests are indicated, by the Mantoux test should be done first. The patient is at high risk for tuberculosis due to living in a shelter and his low socioeconomic status.

Sally, age 19, presents with pain and pressure over her cheeks and discolored nasal discharge. You cannot transilluminate the sinuses. You suspect which common sinus to be affected? A. Maxillary sinus B. Ethmoid sinus C. Temporal sinus D. Frontal sinus

A. Maxillary sinus

What is a noninvasive method of treating skin cancer (other than melanomas) that uses liquid nitrogen? A. Mohs' micrographic surgery B. Curettage and electrodisection C. Radiation therapy D. Cryosurgery

A. Mohs' micrographic surgery

In which age group does respiratory retraction occur more often? A. Newborn and infant B. School-aged children C. Young adult D. Older adult

A. Newborn and infant

Marci, age 15, has been given a diagnosis of step 1 (mild intermittent) asthma. What long-term control therapy is indicated? A. None B. A single agent with anti-inflammatory activity C. An inhaled corticosteroid with the addition of long-acting bronchodilator if needed D. Multiple long-term control medications with oral corticosteroids if needed

A. None

What is the common treatment for thrush? A. Nystatin oral suspension for 2 weeks, 2-3ml in each side of the mouth, held as long as possible B. Clotrimazole oral troches (10mg) two times per day for 7 days C. Fluconazole (100mg) twice daily for 1 week D. Antiseptic mouth rinses after each meal

A. Nystatin oral suspension for 2 weeks, 2-3ml in each side of the mouth, held as long as possible

Tina, age 2, had a complete blood count (CBC) drawn at her last visit. It indicates that she has a microcytic hypochromic anemia. What should you do now at this visit? A. Obtain a lead level B. Instruct Tina's parents to increase the amount of milk in her diet. C. Start Tina on ferrous sulfate (Feosol) and check the CBC in 6 weeks. D. Recheck the CBC on this visit

A. Obtain a lead level - Provider should always check a lead level before starting iron supplementations in children because an elevated lead level will cause anemia despite a normal iron level.

What should be considered in all clients with adult onset asthma or in clients with asthma that worsens in adulthood? A. Occupational asthma B. A suppressed immune system C. Another immunologic disease D. Concurrent COPD

A. Occupational asthma

Susie asks you about the blackheads on her face. You tell her these are referred to as A. Open comedones B. Closed comedones C. Papules D. Pustules

A. Open comedone

The first-line antibiotic therapy for an adult with no known allergies and suspected group A beta-hemolytic streptococcal pharyngitis is A. Penicillin B. Erythromycin (E-Mycin) C. Azithromycin (Zithromax) D. Cephalexin (Keflex)

A. Penicillin

An eczematous skin reaction may result from A. Penicillin B. Allopurinol (Zyloprim) C. An oral contraceptive D. Phenytoin (Dilantin)

A. Penicillin - May also be caused by neomycin, phenothiazines, and local anesthetics.

After a total laryngectomy for laryngeal cancer, the client will have a A. Permanent tracheostomy B. Temporary tracheostomy C. Temporary tracheostomy until the internal surgical incision heals D. Patent normal airway

A. Permanent tracheostomy - The client will have a permanent tracheostomy because no connection exists between the trachea and the esophagus

Martin is coming in for his examination directly after a dental appointment at which he was told he had gingival overgrowth. What medication do you think he must be taking? A. Phenytoin (Dilantin) B. Methyldopa (Aldomet) C. Prednisone (Deltasone) D. Sertraline (Zoloft)

A. Phenytoin (Dilantin)

The "herald patch" is present in almost all cases of A. Pityriasis rosea B. Psoriasis C. Impetigo D. Rubella

A. Pityriasis rosea

The "herald patch" is present in almost all cases of A. Pityriasis rosea B. Psoriasis C. Impetigo D. Rubella

A. Pityriasis rosea - A solitary, oval, pink, scaly plaque, approximately 3-5cm in diameter, on the trunk or proximal extremities. Referred to as the herald patch because it has an elevated border and a central clearing

A patient presents with silvery scales on the extensor surface of the elbows, knees, and back. The plaques are erythematous. This is likely: A. Plaque psoriasis B. Guttate psoriasis C. Atopic dermatitis D. Staph cellulitis

A. Plaque psoriasis The thick, silvery scale is pathognomic & usally asymptomatic but may c/o pruritis. Pitting of fingernails found in 50% of pt.'s. Chronic skin disorder.

In which condition would the trachea be deviated toward the normal side? A. Pleural effusion and thickening B. Pneumonia C. Bronchiectasis D. Pulmonary fibrosis

A. Pleural effusion and thickening - The trachea would be deviated toward the normal side because of fluid displacing the pleural space.

In which condition would the trachea be deviated toward the normal side? A. Pleural effusion and thickening B. Pneumonia C. Bronchiectasis D. Pulmonary fibrosis

A. Pleural effusion and thickening - The trachea would be deviated toward the normal side because of fluid displacing the pleural space.

The causative agent of the community-acquired pneumonia seen most often in the client with an alcohol problem is A. Pneumococcus B. Mycoplasma C. Legionella D. Haemophilus influenzae

A. Pneumococcus - Caused by the gram-positive bacteria Pneumoccous (also called Streptococcus pneumoniae).

The causative agent of the community-acquired pneumonia seen most often in the client with an alcohol problem is A. Pneumococcus B. Mycoplasma C. Legionella D. Haemophilus influenzae

A. Pneumococcus - Caused by the gram-positive bacteria Pneumoccous (also called Streptococcus pneumoniae).

Despite successful primary prophylaxis, which infection remains a common AIDS-defining diagnosis? A. Pneumocystitis jiroveci pneumonia (PCP) B. Cryptococcosis C. Cryptosporidiosis D. Candidiasis

A. Pneumocystitis jiroveci pneumonia - Before the appearance of AIDS, Pneumocystitis jiroveci pneumonia was a rare disease that immunosuppressed persons and clients with leukemia sometimes developed.

The most common offending allergens causing allergic rhinitis are A. Pollens of grasses, trees and weeds B. Fungi C. Animal allergens D. Food sensitivity

A. Pollens of grasses, trees, and weeds

Systemic lupus erythematosus is diagnosed on the basis of A. Positive antinuclear antibody (ANA), malar rash, and photosensitivity B. Positive ANA, weight loss, and night sweats C. Negative ANA, photosensitivity, and renal disease D. Leukopenia, negative ANA, and photosensitivity

A. Positive antinuclear antibody (ANA), malar rash, and photosensitivity

Which of the following refractive errors in vision is a result of the natural loss of accomodative capacity with age? A. Presbyopia B. Hyperopia C. Myopia D. Astigmatism

A. Presbyopia

David, age 32, states that he thinks he has an ear infection because he just flew back from a business trip and feels unusual pressure in his ear. You diagnose barotrauma. What is your next action? A. Prescribe nasal steroids and oral decongestants B. Prescribe antibiotic eardrops C. Prescribe systemic eardrops D. Refer David to an ear, nose, and throat specialist

A. Prescribe nasal steroids and oral decongestants

Which group is at the highest risk for tuberculosis? A. Racial and ethnic minorities B. Foreign born individuals C. Substance abusers D. Nursing home residents

A. Racial and ethnic minorities - 70% of all cases in the U.S. are in this group - Foreign born individuals make up 24% of cases

Which group is at the highest risk for tuberculosis? A. Racial and ethnic minorities B. Foreign born individuals C. Substance abusers D. Nursing home residents

A. Racial and ethnic minorities - 70% of all cases in the U.S. are in this group - Foreign born individuals make up 24% of cases

A client come in complaining of 1 week of pain in the posterior neck with difficulty turning the head to the right. What additional history is needed? A. Recent trauma B. Difficulty swallowing C. Stiffness in the right shoulder D. Changes in sleeping habits

A. Recent trauma

Mattie, age 64, presents with blurred vision in one eye and states that it felt like "a curtain came down over my eye." She doesn't have any pain or redness. What do you suspect? A. Retinal detachment B. Acute angle-closure glaucoma C. Open-anlge glaucoma D. Cataract

A. Retinal detachment

Clindamycin is most effective against which of the following organisms? A. S. pneumoniae B. H. influenzae C. M. catarrhalis D. Adenovirus

A. S. pneumoniae

The most common cause of pneumonia in people of all ages is: A. S. pneumoniae B. Group A strep C. S. aureus D. Mycoplasma sp.

A. S. pneumoniae - Streptococcus pneumoniae is the most common pathogen of community acquired pneumonia world-wide.

Persons with which skin phototype (SPT) sunburn easily after 30 minutes in the sun but never tan? A. SPT I B. SPT II C. SPT III D. SPT IV

A. SPT I - Skin phototyping is a risk classification system designed to estimate one's risk for sun damage

Which chronic skin disorder primarily affects hairy areas of the body? A. Seborrheic dermatitis B. Atopic dermatitis C. Contact dermatitis D. Hydradenitis suppurativa

A. Seborrheic dermatitis - Causes flaking of the skin, usually the scalp. In adolescents and adults, when it affects the scalp, it it termed dandruff. When this occurs in young children or infants, it is termed cradle cap.

A 22 year old client who plays in a rock band complains that he finds it difficult to understand his fellow musicians at the end of a night of performing, a problem that is compounded by the noisy environment of the "club." These symptoms are most characteristic of which of the following? A. Sensorineural loss B. Conductive loss C. Tinnitus D. Vertigo

A. Sensorineural loss

John, age 58, is a farmer. He presents with a painful finger ulcer and a palpable olecranal lymph node. Suspecting an orf skin ulcer, you ask him if he works with A. Sheep and goats B. Horses C. Metals D. Tiles

A. Sheep and goats

John, age 58, is a farmer. He presents with a painful finger ulcer and a palpable olecranal lymph node. Suspecting an orf skin ulcer, you ask him if he works with A. Sheep and goats B. Horses C. Metals D. Tiles

A. Sheep and goats - Orf skin ulcers result from a parapoxvirus infection, a common skin disease of sheep and goats

which of the folllowing represents the most effective metthod of cancer sceening? A. skin examination B.. stool for occult blood C.. pelvic examination D. chest radiography

A. Skin examination

Sue has sickle cell anemia. In regulating her and monitoring her hemoglobin and hematocrit levels, you want to maintain them at A. Slightly below normal B. Strictly at normal C. Slightly above normal D. Around normal with only minor fluctuations

A. Slightly below normal - Clients with sickle cell anemia should have their hemoglobin and hematocrit levels maintained at a level slightly below normal because this protects from some of the vaso-occlusive infarctive complications related to the viscosity characteristics of sickle cell anemia.

Caroline, an older adult, is homeless and has iron-deficiency anemia. She smokes and drinks when she can and has an ulcer. Which of the following is not one of the risk factors of iron-deficiency anemia? A. Smoking B. Poverty C. Ulcer disease D. Age older than 60

A. Smoking

The two most predominant organisms constituting the normal flora of the oropharynx are A. Streptococci and Staphylococci B. Streptococci and Moraxella catarrhalis C. Staphylococci and Candida albicans D. Various protozoa and Staphylococci

A. Streptococci and Staphylococci

Community-acquired bacterial pneumonia is most commonly caused by A. Streptococcus pneumoniae B. Mycoplasma pneumoniae C. Haemophilus influenzae D. Staphylococcus aureus

A. Streptococcus pneumonia - Causes 30%-75% of all community acquired bacterial pneumonia

Community-acquired bacterial pneumonia is most commonly caused by A. Streptococcus pneumoniae B. Mycoplasma pneumoniae C. Haemophilus influenzae D. Staphylococcus aureus

A. Streptococcus pneumonia - Causes 30%-75% of all community acquired bacterial pneumonia

Which bacterial agent is the most common pathogen in an acute exacerbation of chronic bronchitis? A. Streptococcus pneumonia B. Haemophilus influenzae C. Branhamella catarrhalis D. Moraxella catarrhalis

A. Streptococcus pneumoniae - While viral bronchitis requiring only supportive care is the most common etiology of acute exacerbations of chronic bronchitis, bacterial involvement must be considered when there is increased sputum production lasting over a week or new chest X-ray findings.

An 18-year-old woman has a chief complaint of a "sore throat and swollen glands" for the past 3 days. Her physical examination includes a temperature of 101F (38.3C), exudative pharyngitis, and tender anterior cervical lymphadenopathy. Right and left upper quadrant abdominal tenderness is absent. The most likely diagnosis is: A. Streptococcus pyogenes pharyngitis B. infectious mononucleosis C. viral pharyngitis D. Vincent angina

A. Streptococcus pyogenes pharyngitis

The majority of malignant melanomas are A. Superficially spreading B. Lentigo maligna C. Acral-lentiginous D. Nodular

A. Superficially spreading

A darkfield examination is used to cutaneously diagnose which disease? A. Syphilis B. Viral blisters C. Scabies D. Candidiasis

A. Syphilis

Which statement about chronic obstructive pulmonary disease (COPD) is true? A. The prevalence of COPD is directly related to increasing age. B. The incidence of COPD is about equal in men and in women C. Cigar or pipe smoking does not increase the risk of developing COPD. D. Environmental factors such as smoke do not affect the potential for COPD.

A. The prevalence of COPD is directly related to increasing age. - Men are affected much more often than women because the percentage of men who smoke is greater than that of women.

A definitive test for cystic fibrosis is A. The sweat test B. A sputum culture C. A fecal fat test D. A chymex test for pancreatic insufficiency

A. The sweat test

What significant finding(s) in a child with otitis media with effusion would prompt more aggressive treatment? A. There is a change in the child's hearing threshold to less than or equal to 20 decibels (dB) B. The child becomes a fussy eater C. The child's speech and language skills seem slightly delayed D. Persistent rhinitis is present

A. There is a change in the child's hearing threshold to less than or equal to 20 decibels (dB)

Your 18 year old client, Mandy, has infectious mononucleosis. What might you expect her blood work to reflect? A. Thrombocytopenia and elevated transaminase B. Elevated white blood cells (WBCs) C. Decreased WBCs D. Decreased serum globulins

A. Thrombocytopenia and elevated transaminase - Infectious mononucleosis is a lymphocytic leukocytosis that may be confused with leukmia and other disorders. The presence of heterophile antibodies (monospot test) in the context of clinical and hematological findings is diagnostic.

Which of the following medications should be avoided in a patient who has asthma? A. Timolol ophthalmic drops B. Naproxen C. Topical hydrocortisone D. Amlodipine

A. Timolol ophthalmic drops - Timolol is a beta blocker. This class of medications can precipitate an asthma exacerbation in patients.

A Wood's light is especially useful in diagnosing which of the following? A. Tinea versicolor B. Herpes zoster C. A decubitus ulcer D. A melanoma

A. Tinea versicolor

A Wood's light is especially useful in diagnosing which of the following? A. Tinea versicolor B. Herpes zoster C. A decubitus ulcer D. A melanoma

A. Tinea versicolor - Useful in tinea versicolor and other fungal infections. It accentuates minor losses of melanin

All of the following are treatments for psoriasis except A. Topical antifungals B. Systemic medications C. Phototherapy D. Topical corticosteroids

A. Topical antifungals

All of the following are treatments for psoriasis except A. Topical antifungals B. Systemic medications C. Phototherapy D. Topical corticosteroids

A. Topical antifungals - Most common form of treatment is topical corticosteroids

Which fo the following conditions produces sharp, piercing facial pain that lasts for seconds to minutes? A. Trigeminal neuralgia B. TMJ C. Goiter D. Preauricular adenitis

A. Trigeminal neuralgia

A test used to diagnose shingles if the clinical presentation is questionable is a: A. Tzanck preparation B. Viral load C. Rapid herpetic test D. CBC

A. Tzanck preparation - A rapid test used to diagnose infections due to herpes viruses.

Johnnie, age 52, presents with pruritus with no rash present. He has hypertension, diabetes, and ESRD. One of the differential diagnoses would be A. Uremia from chronic renal disease B. Contact dermatitis C. Lichen planus D. Psoriasis

A. Uremia from chronic renal disease - All of the conditions listed result in pruritus. Only uremia from chronic renal disease, however, results in pruritus with no rash present

The aging process results in a variety of physiological changes. One change is A. a decreased absorption of fat-soluble vitamins B. an increase in pupil size C. an increase of enzymatic activity D. an increase in liver size

A. a decreased absorption of fat-soluble vitamins

Skin lesions associated with actinic keratosis can be described as: A. a slightly rough, pink or flesh-colored lesion in a sun-exposed area B. a well-defined, slightly raised, red, scaly, plaque in a skinfold C. a blistering lesion along a dermatome D. a crusting lesion along flexor aspects of the fingers

A. a slightly rough, pink or flesh-colored lesion in a sun-exposed area

When prescribing pulse dosing with itraconazole for the treatment of fingernail fungus, the clnician realizes: A. a transient increase in hepatic enzymes is commonly seen with its use B. drug-induced leukopenia is a common problem C. the paient needs to be warned about excessive bleeding because of the drug's antiplatelet effect D. its use is contraindicated n the presence of iron-deficiency anemia

A. a transient increase in hepatic enzymes is commonly seen with its use

Ms. Clancy, age 28, is complaining of a "cold that won't go away." She "thinks" that she has had a fever (she is afebrile in your office) and reports a nasal discharge that has become greenish in color. She also reports a cough. You decide that she has an acute, uncomplicated sinusitis and order A. amoxicillin (Amoxil) 500mg PO tid for 14-21 days B. sulfamethoxazole and trimethoprim (Bactrim) 1 DS tab PO bid for 7-10 days C. amoxicillin (Amoxil) 500 mg PO bid for 3-4 weeks D. ciprofloxacin (Cipro) 500 mg bid for 14 days

A. amoxicillin (Amoxil) 500mg PO tid for 14-21 days

Marty, age 7, was brought to your primary care office with dog bites on his right hand and arm. You would send him home on A. amoxicillin and clavulunate (Augmentin) B. penicillin (Pentids) C. amoxicillin (Amoxil) D. clarithromycin (Biaxin)

A. amoxicillin and clavulunate (Augmentin)

Auscultation of the chest of a client with congestive heart failure typically reveals A. an S3 gallop B. a pericardial rub C. bronchial vesicular breath sounds D. S4 sounds

A. an S3 gallop

The term "pica" refers to A. an insatiable craving for such substances as laundry starch, clay, and ice and is thought to suggest iron deficiency B. poverty and violence C. mental retardation in children D. a folic acid deficiency

A. an insatiable craving for such substances as laundry starch, clay, and ice and is thought to suggest iron deficiency

Anthralin (Drithocreme) is helpful in treating psoriasis because it has what kind of activity: A. antimitotic B. exfoliative C. vasoconstrictor D. humectant

A. antimitotic

A 70 year old man presents with absent pulses and a cool, hairless foot with a 2 cm ulcer that has a "punched out" appearance on the dorsum of the second toe. The appropriate next measure is to: A. arrange for evaluation for arterial insufficiency B. apply wet to dry dressingss C. start the patient on a peripheral vasodilator D. advise the patient to elevate the feet for 15 minutes three times a day

A. arrange for evaluation for arterial insufficiency

The most common ocular manifestation with rosacea is A. blepharitis B. conjunctivitis C. hordeolum D. tearing

A. blepharitis

A reasonable treatment option for AOM that is not improved after 3-day therapy with an appropriate dosage of amoxicillin therapy is: A. cefuroxime B. erythromycin C. cephalexin D. sulfamethoxazole

A. cefuroxime

Likely causative organisms in AOM include: A. certain gram-positive and gram-negative bacteria B. gram-negative bacteria and pathogenic viruses C. rhinovirus and S. aureus D. predominantly beta-lactamase-producing organisms

A. certain gram-positive and gram-negative bacteria

Which of the following medications is likely to cause the most sedation? A. chlorpheniramine B. cetirizine C. fexofenadine D. loratadine

A. chlorpheniramine

you exammne the left lower extremity of a patient and find an ulcerated lesion with irregular borders and edema with brown discoloration of the surrounding tiissue. The best treatment option at this time is: A.. compression therapy B. referral for surgicall debridement C. daily use of a topical antibiotic crream D. application of a dry, sterile dressing

A. compression therapy

Clients with refractive asthma should be evaluated along a number of parameters. These include A. continued exposure to known causes, evaluation of compliance with treatment, examination of behavioral aspects, and evaluation of possible other causes of airway obstruction B. underlying, unresolved infectious processes; behavioral aspects of the disease; and compliance with treatment C. environmental aspects, family support systems, and socioeconomic status D. compliance with treatment, an evaluation of underlying nutritional status, family structure, and behavioral aspects of the disease

A. continued exposure to known causes, evaluation of compliance with treatment, examination of behavioral aspects, and evaluation of possible other causes of airway obstruction

The major risk of a red blood cell transfusion is A. cytomegalovirus infection B. hepatitis B infection C. HIV infection D. a hemolytic transfusion reaction

A. cytomegalovirus infection

A 24-year-old woman presents with hive-form linear lesions that form over areas where she has scratched. These resolve within a few minutes. This most likely represent A. dermographism B. contact dermatitis C. angioedema D. allergic reaction

A. dermographism

Mrs. Graves, age 38, has been on birth control pills for approximately 15 years. She is a smoker, has a blood pressure of 110/70 mm Hg, and has lipid levels within normal limits. You advise that she should A. discontinue the birth control pills because of her smoking B. remain on the pill because her blood pressure and lipids are within normal limits C. remain on the pill until her follicle-stimulating hormone level is greater than 30 D. discontinue the pill because of her age

A. discontinue the birth control pills because of her smoking

Allergy immunotherapy is most successful in controlling allergies caused by: A. dust mites B. molds C. animal dander D. air pollution

A. dust mites

Which of the following do you expect to find in the assessment of a person with urticaria? A. eosinophilia B. low ESR C. elevated TSH D. leukopenia

A. eosinophilia

Which of the following is most likely to be found on the fundoscopic examination in a patient with untreated POAG? A. excessive cupping of the optic disk B. arteriovenous nicking C. papilledema D. flame-shaped hemorrhages

A. excessive cupping of the optic disk

A 78 year old resident complaints of generalized itchiness at night that disturbs her sleep. Her examination is consistent with scabies. Which of the following do you expect to find on examination: A. excoriated papules on the interdigital area B annular lesions over the butocks C vesicular lesions in a linear pattern D. honey-colored crusted lesions that began as vesicles

A. excoriated papules on the interdigital area

A common trigger agent for contact dermatitis is: A. exposure to nickle B. use of fabric softener C. washing with strong soap D. eating spicy food

A. exposure to nickel

An 88-year-old, community-dwelling man who lives alone has limited mobility because of osteoarthritis. Since his last office visit 2 months ago, he has lost 5% of his body weight and has developed angular chelitis. You expect to find the following on examination: A. fissuring and cracking at the corners of the mouth B. marked erythema of the hard and soft palates C. white plaques on the lateral borders of the buccal mucosa D. raised, painless lesions on the gingiva

A. fissuring and cracking at the corners of the mouth

The most common risk factors for thromboembolism include A. fractures and prolonged immobilization B. myocardial infarction and atrial fibrillation C. cardiomyopathy and pregnancy D. abnormal fibrinolysis and congestive heart failure

A. fractures and prolonged immobilization

List ointments from least to most potency A. gel, lotion, cream, ointment B. ointment, cream, lotion, gel C. lotion, gel, ointment, cream D. cream, ointment, gel, lotion

A. gel, lotion, cream, ointment

A 74 year-old woman with well-controlled hypertension who is taking hydrochlorothiazide presents with a 3-day history of unilateral throbbing headache with difficulty chewing because of the pain. On physical examination, you find a tender, noncompressible temporal artery. Blood pressure (BP) is 160/88 mm Hg, apical pulse is 98 bpm, and respiratory rate is 22/min; the patient is visibly uncomfortable. The most likely diagnosis is: A. giant cell arteritis B. impending transient ischemic attack C. complicated migraine D. temporal mandibular joint dysfunction

A. giant cell arteritis

A 38 year old woman has a chief complaint of painful, itchy rash over her trunk. Examination reeals linear vesicular lesions that do not cross the midline and are distributed over the posterior thorax. This is most consistent with: A. herpes zoster B. dermaitis herpetiformis C. molluscum contagiosum D. impetigo

A. herpes zoster

A Tzanck smear that is positive for giant multinucleated cells was taken from a lesion caused by: A. herpesvirus B. S. aureus C. streptococci D allergic reaction

A. herpesvirus

Characteristics of M. catarrhalis include: A. high rate of beta-lactamase production B. antimicrobial resistance resulting from altered protein-binding sites C. often being found in middle ear exudate in recurrent otitis media D. gram-positive organisms

A. high rate of beta-lactamase production

The mechanism of action of pimecrolimus (Elidel) in the treatment of atopic dermatitis is as a/an: A. immunomodulator B. antimitotic C. mast cell activator D. exfoliant

A. immunomodulator

You see a 36 year-old man with no chronic health problems who presents with two furuncles, each around 4 cm in diameter, on the right anterior thigh. These lesions have been present for 3 days, slightly increasing in size during this time. He has no fever or other systemic symptoms. You advise the following: A. incision and drainage of the lesion B. a systemic antibiotic empirically C. a topical antibiotic D. aspiration of the lesion contents and prescription of a systemic antibiotic based on culture results

A. incision and drainage of the lesion

When performing a respiratory assessment on a client with pneumococcal pneumonia, you would expect to find A. increased vocal fremitus B. fine crackles C. hyperresonance D. asymmetric chest expansion

A. increased vocal fremitus

All of the following are common causes of penicillin treatment failure in streptococcal pharyngitis except: A. infection with a beta-lactamase producing Streptococcus strain B. failure to initiate or complete the antimicrobial course C. concomitant infection or carriage with a beta-lactamase-producing organism D. inadequate penicillin dosage

A. infection with a beta-lactamase producing Streptococcus strain

Characteristics of onychomycosis include all of the following except: A. it is readily diagnosed by clinical examination B. nail hypertrophy C. brittle nails D. fingernails respond more readily to therapy than toenails

A. it is readily diagnosed by cliical examination

Which of the following is not a potential adverse effect with long term high-potency topical corticosteroid use: A. lichenification B. telangiectasia C. skin atrophy D. adrenal suppression

A. lichenification

Which of the following is indicated for the treatment of acne rosacea? A. metronidazole gel (Metrogel) B, benzyol peroxide C. erythromycine 2% solution (EryDerm) D. azelaic acid 20% ( Azele cream)

A. metronidazole gel (MetroGel)

Which of the following is inconsistent with the clinical presentation of ABRS? A. nasal congestion responsive to decongestant use B. maxillary toothache C. colored nasal discharge D. antecedent event such as acute upper respiratory tract infection or allergic rhinitis

A. nasal congestion responsive to decongestant use

A "liver flap", associated with hepatic encephalopathy, uremia, and respiratory acidosis, refers to A. nonrhythmic flapping of the wrists and hands B. a palpable liver that "flaps" against the palpating hand C. a jaundiced color of the skin D. urine that is the color of liver

A. nonrhythmic flapping of the wrists and hands

Which of the following is a common vision problem in the person with untreated primary open-angle glaucoma (POAG)? A. peripheral vision loss B. blurring of near vision C. difficulty with distant vision D. need for increased illumination

A. peripheral vision loss

Concomitant disease seen with giant cell arteritis includes: A. polymyalgia rheumatica B. acute pancreatitis C. psoriatic arthritis D. Reiter syndrome

A. polymyalgia rheumatica

In caring for the patient in Question 14 (angle-closure glaucoma), the most appropriate next action is: A. prompt referral to an ophthalmologist B. to provide analgesia and repeat the evaluation when the patient is more comfortable C. to instill a corticosteroid ophthalmic solution D. to patch the eye and arrange for follow-up in 24 hours

A. prompt referral to an ophthalmologist

The system that is affected in about 75% of all clients with systemic lupus erythematosus and has one of the most serious systemic sequelae, leading to significant morbidity and mortality, is the A. renal system B. cardiovascular system C. neuromuscular system D. integumentary system

A. renal system

Select a statement that is true about the erythrocyte sedimentation rate (ESR). A. It is a very specific indicator of inflammation B. A rise in the ESR is a normal part of aging C. It is useful in detecting pancreatic cancer D. It is diagnostic for rheumatoid arthritis

B. A rise in the ESR is a normal part of aging - The ESR is a very nonspecific indicator of inflammation and is often elevated in inflammatory musculoskeletal conditions; it is not, however, diagnostic for rheumatoid arthritis.

Matthew, age 52, has allergic rhinitis and would like some medicine to relieve his symptoms. He is taking cimetidine (Tagamet) for gastroesophageal reflux disease. Which medication would you not order? A. A first-generation antihistamine B. A second-generation antihistamine C. A decongestant D. A topical nasal corticosteroid

B. A second-generation antihistamine

What is the connection between the surface of the skin and an underlying structure called? A. An ulcer B. A sinus C. An erosion D. An abscess

B. A sinus - An ulcer is a depressed lesion in which the epidermis and part of the dermis have been lost - An erosion is a moist, red shiny, circumscribed lesion that lacks the upper layer of the skin - An abscess is a circumscribed collection of pus that involves the deeper layers of the skin

To ease their breathing, clients with COPD often position themselves in: A. An erect sitting position B. A tripod position C. A supine position D. A prone position

B. A tripod position - This provides clients with leverage so that their rectus abdominal, intercostal and accessory neck muscles can all assist with expiration

Which form of acne is more common in the middle-aged to older adult and causes changes in skin color, enlarged pores, and thickening of the soft tissues of the nose? A. Acne vulgaris B. Acne rosacea C. Acne conglobata D. Nodulocystic acne

B. Acne rosacea

Which form of acne is more common in the middle-aged to older adult and causes changes in skin color, enlarged pores, and thickening of the soft tissues of the nose? A. Acne vulgaris B. Acne rosacea C. Acne conglobata D. Nodulocystic acne

B. Acne rosacea - A chronic type of facial acne that occurs in middle-aged to older adults.

In which condition would you assess vesicular breath sounds, moderate vocal resonance, and localized crackles with sibilant wheezes? A. Bronchiectasis B. Acute bronchitis C. Emphysema D. Asthma

B. Acute bronchitis

A medication which should be avoided in patients with mononucleosis is: A. Azithromycin B. Ampicillin C. Acetaminophen D. Topical lidocaine

B. Ampicillin - A generalized rash may be seen in patients with mono who are given amoxicillin or ampicillin at the time of the acute phase of the illness. The rash does not represent an allergic reaction, but, instead probably respresents a reaction between the Epstein Barr virus and the penicillin molecule.

Sam is being worked up for pancreatic cancer. He states that the doctor wants to put a "scope" in and inject dye into his ducts. He wants to know more about this. What procedure is he referring to? A. Percutaneous transhepatic cholangiography B. An endoscopic retrograde cholangiopancreatography C. An angiography D. An upper gastrointestinal (GI) series

B. An endoscopic retrograde cholangiopancreatography - An endoscope is inserted via the mouth and passed by the stomach and into the small intestine, where the dye is injected into the pancreatic ducts and x-rays are taken to determine if any obstruction is apparent.

Some pharmacological adjuncts to analgesics in clients with uncontrolled cancer pain include A. Anticonvulsants and tricyclic antidepressants B. Anticonvulsants, tricyclic antidepressants, and corticosteroids C. Selective serotonin receptor inhibitors D. Benzodiazepines

B. Anticonvulsants, tricyclic antidepressants, and corticosteroids - Pain that is poorly controlled with opioids and NSAIDs is often neuropathic in nature, meaning that it is a result of direct nerve injury, such as nerve compression

Sickle cell anemia is an autosomal recessive disorder caused by the hemoglobin S gene. An abnormal hemoglobin leads to chronic hemolytic anemia with numerous clinical manifestations and becomes a chronic multisystem disease, with death from organ failure, usually between ages 40 and 50. The hemoglobin S gene is carried by A. Approximately 4% of the U.S. population B. Approximately 8% of American blacks C. Approximately 4% of Latinos D. Approximately 12% of Native Americans

B. Approximately 8% of American blacks - The hemoglobin S gene is carried by approximately 8% of American blacks and 1 birth in 400 in American blacks will produce a child with sickle cell.

Patients with atopic dermatitis are also likely to exhibit: A. Allergic rhinitis and anaphylaxis B. Asthma and allergic rhinitis C. Nasal polyps and asthma D. Allergic conjunctivitis and wheezing

B. Asthma and allergic rhinitis The atopic triad commonly seen in patients includes atopic dermatitis, asthma, and allergic rhinitis.

Most cases of epistaxis occur: A. In women B. At Kiesselbach's plexus C. In the posterior septum D. In patients on anticoagulants

B. At Kiesselbach's plexus - Most nosebleeds occur in men and 80% are anterior. Kiesselbach's plexus is the most common site for any epistaxis to occur because this site represents the anastamosis of branches of three primary vessels, the ethmoidal artery, the sphenopalatine artery, and the facial artery.

The most common complication of influenza is: A. Cough B. Bacterial pneumonia C. Viral pneumonia D. Bronchitis

B. Bacterial pneumonia - Pneumonia is the most common complication of influenza; bacterial pneumonia is the most common form. Streptococcus pneumoniae is the most common bacterial pathogen.

The most common form of skin cancer is: A. Squamous cell carcinoma B. Basal cell carcinoma C. Malignant melanoma D. Cutaneous carcinoma

B. Basal cell carcinoma - Skin cancers are divided into two major groups: non-melanoma and melanoma skin cancer. Basal cell carcinoma is considered non-melanoma skin cancer and is the most common form of skin cancer in the US.

Mary just came from visiting her husband, Sam, age 82, who recently had an ileostomy resulting in a stoma. She did not think that Sam's stoma looked "right." You tell her that the color fo the stoma should be A. Pale pink B. Beefy red C. Dark red or purple D. Flesh colored

B. Beefy red

Mary just came from visiting her husband, Sam, age 82, who recently had an ileostomy resulting in a stoma. She did not think that Sam's stoma looked "right." You tell her that the color fo the stoma should be A. Pale pink B. Beefy red C. Dark red or purple D. Flesh colored

B. Beefy red - A pale pink color may indicate a low hemoglobin level - A dark red or purple stoma may indicate early ischemia - A black stoma is the result of necrosis

Which irregular respiratory pattern has a series of three to four normal respirations followed by a period of apnea and is seen with head trauma, brain abscess, heat stroke, spinal meningitis, and encephalitis? A. Cheyne-Stokes respiration B. Biot's breathing C. Kussmaul's respiration D. Hypoventilation

B. Biot's breathing - Cheyne-Stokes respiration is similar, except that the pattern is regular

Tobacco has been linked to which of the following types of cancer? A. Colon cancer B. Bladder cancer C. Prostate cancer D. Cervical cancer

B. Bladder cancer - Tobacco use has been linked to an increased risk for bladder, pancreatic, laryngeal, esophageal, oropharyngeal, and some types of gastric cancer.

Samual, age 5, is receiving radiation therapy for his acute lymphocytic leukemia. He is at increased risk of developing which type of cancer as a secondary malignancy when he becomes an adult? A. Chronic lymphocytic leukemia B. Brain tumor C. Liver cancer D. Esophageal cancer

B. Brain tumor

The inspiratory rate equals the expiratory rate with which breath sound? A. Bronchial B. Bronchovesicular C. Vesicular D. Tracheal

B. Bronchovesicular - With bronchial or tracheal breath sounds, the inspiratory rate is shorter than the expiratory rate, and with vesicular breath sounds, the inspiratory rate is greater than the expiratory rate

Jill states that her 5 year old daughter continually grinds her teeth at night. You document this as A. Temporal mandibular joint malocclusion B. Bruxism C. A psychosis D. An oropharyngeal lesion

B. Bruxism

George has ischemic changes and gangrene of the hands and fingers. This may be a result of A. Raynaud's disease B. Buerger's disease C. Allen's disease D. Hodgkin's disease

B. Buerger's disease

A patient has burned himself with hot water. He has several very large fluid filled lesions. What are these termed? A. Vesicles B. Bullae C. Erosions D. Dermal abrasions

B. Bullae - Bullae are fluid filled lesions that are greater than 1 cm in diameter. Common in second-degree burns.

How do you test for near vision? A. By using the Snellen eye chart B. By using the Rosenbaum chart C. By asking the client to read from a magazine or newspaper D. By testing the cardinal fields

B. By using the Rosenbaum chart - Hold it about 12-14 inches from the client's eyes.

A patient is found to have spoon shaped nails. What laboratory test would be prudent to perform? A. Liver function tests B. CBC C. Hepatitis B surface antigen D. Arterial blood gases

B. CBC - Spoon shaped nails often are present in patients with iron deficiency anemia. Perform CBC to r/o anemia.

Justin, an obese 42 year old, cut his right leg 3 days ago while climbing a ladder. Today his right lower leg is warm, reddened, and painful without a sharply demarcated border. What do you suspect? A. Diabetic neuropathy B. Cellulitis C. Peripheral vascular disease D. A beginning stasis ulcer

B. Cellulitis

Justin, an obese 42 year old, cut his right leg 3 days ago while climbing a ladder. Today his right lower leg is warm, reddened, and painful without a sharply demarcated border. What do you suspect? A. Diabetic neuropathy B. Cellulitis C. Peripheral vascular disease D. A beginning stasis ulcer

B. Cellulitis - Cellulitis is a spreading infection of the epidermis and subcutaneous tissue that usually begins after a break in the skin

Which manifestation is noted with carbon monoxide poisoning? A. Circumoral pallor of the lips B. Cherry-red lips C. Cyanosis of the lips D. Pale, pink lips

B. Cherry-red lips

Which bone tumor arises from cartilage and is usually located in the pelvis, femur, proximal humerus, or ribs? A. Osteosarcoma B. Chondrosarcoma C. Ewing's sarcoma D. Fibrosarcoma

B. Chondrosarcoma - It is the second most common bone malignancy, seen most frequently in men between ages 30 and 60.

Which of the following increases the risk of pancreatic cancer? A. A high-carbohydrate diet B. Cigarette smoking, diabetes, and a high-fat diet C. Diabetes and lack of activity D. Yo-yo dieting

B. Cigarette smoking, diabetes, and a high-fat diet

Which of the following statements is true when trying to differentiate pulmonary from cardiac causes of dyspnea on exercise? A. When the cause is pulmonary, the rate of recovery to normal respiration is slow, and dyspnea abates eventually after cessation of exercise. B. Clients with dyspnea from cardiac causes remain dyspneic much longer after cessation of exercise. C. In dyspnea arising from cardiac causes, the heart rate will return to preexercise levels within a few minutes after cessation of exercising. D. Clients with pulmonary dyspnea have minimal dyspnea at rest.

B. Clients with dyspnea from cardiac causes remain dyspneic much longer after cessation of exercise

An older adult has cerumen impaction in both ears. His hearing is diminished. This type of hearing loss is: A. Sensorineural B. Conductive C. Presbycusis D. Cholesteatoma

B. Conductive - A conductive hearing loss is one in which sound cannot enter the external canal or, the middle ear.

A patient with acute bronchitis for 5 days calls to report that his cough is productive of purulent sputum. He has no other new symptoms. How should the nurse practitioner manage this? A. Consider pneumonia; prescribe a macrolide antibiotic B. Consider prescribing an anti-tussive only C. Consider bacterial, acute bronchitis; prescribe an antibiotic D. Order a chest X-Ray and treat accordingly

B. Consider prescribing an anti-tussive only - Acute bronchitis is characterized by cough lasting greater than 5 days, but usually less than 20 days. More than 50% or patients report purulent sputum. Since the overwhelming majority of cases of acute bronchitis are secondary to viral causes, antibiotics are not indicated.

Julia asks how smoking increases the risk for folic acid deficiency. You respond that smoking A. Causes small-vessel disease and constricts all vessels that transport essential nutrients. B. Decreases the vitamin C absorption C. Affects the liver's ability to store folic acid D. Causes nausea, thereby inhibiting the appetite and ingestion of foods rich in folic acid

B. Decrease the vitamin C absorption - Vitamin C is necessary for folic acid absorption

Which structure fo the skin is responsible for storing melanin? A. Epidermis B. Dermis C. Sebaceous glands D. Eccrine sweat glands

B. Dermis

Lance, age 50, is complaining of an itchy rash that occurred about a half hour after putting on his leather jacket. He recalls a slightly similar rash last year when he wore his jacket. The annular lesions are on his neck and arms. They are erythematous, sharply circumscribed, and both flat and elevated. His voice seems a little raspy, although he states that his breathing is normal. What is your first action? A. Order a short course of systemic corticosteroids B. Determine the need for 0.5ml 1:1000 epinephrine subcutaneously C. Start daily antihistamines D. Tell Lance to get rid of his leather jacket

B. Determine the need for 0.5ml 1:1000 epinephrine subcutaneously

Which of the following indicates an impending complication of influenza? A. Myalgia and headache B. Diffuse crackles in the lungs C. Sore throat and productive cough D. Fever of 100.4 F with chills

B. Diffuse crackles in the lungs

When teaching smokers about using nicotine gum to aid in smoking cessation, tell them to A. Chew the gum like regular gum B. Discard the gum after 30 minutes C. Drink a cup of coffee before chewing the gum because it assists in the nicotine absorption D. Chew 6-9 pieces daily to help prevent nicotine withdrawal

B. Discard the gum after 30 minutes

With which voice sound technique do you normally hear a muffled "eee" through the stethoscope on auscultating the chest when the client says "eee"? A. Bronchophony B. Egophony C. Whispered pectoriloquy D. Tonometry

B. Egophony - When consolidation is present, the "eee" changes to an "aaa" sound.

An allergic reaction in the kidney is often characterized by which of the following urinary sediments? A. Leukocytes B. Eosinophils C. Crystals D. Erythrocytes

B. Eosinophils

Which of the following white blood cell types is elevated in parasitic infections, hypersensitivity reactions, and autoimmune disorders? A. Neutrophils B. Eosinophils C. Basophils D. Monocytes

B. Eosinophils

Which of the following peripharyngeal upper respiratory tract infections occurs most often in children ages 2-5 years? A. Peritonsillar abscess B. Epiglottitis C. Laryngotracheobronchitis (Croup) D. Bacterial tracheitis

B. Epiglottitis - Peritonsillar abscesses occur more frequently in teenage years. Croup occurs more frequently from age 3 months to 3 years and bacterial tracheitis in children ages 3-10 years.

Which of the following peripharyngeal upper respiratory tract infections occurs most often in children ages 2-5 years? A. Peritonsillar abscess B. Epiglottitis C. Laryngotracheobronchitis (Croup) D. Bacterial tracheitis

B. Epiglottitis - Peritonsillar abscesses occur more frequently in teenage years. Croup occurs more frequently from age 3 months to 3 years and bacterial tracheitis in children ages 3-10 years.

What is the drug of choice for acute anaphylaxis? A. Diphenhydramine (Benadryl) 25-100mg PO QID for adults B. Epinephrine 1:1000 subcutaneously (0.3-0.5ml) for adults C. Prednisone (2mg/kg Q24 hours) PO in one initial daily dose, tapered off over 1-2 weeks D. Amlodipine besylate (Norvasc) 5mg QID for 4 weeks

B. Epinephrine 1:1000 subcutaneously (0.3-0.5ml) for adults

Monique brings her 4 week old infant into the office because she noticed small, yellow-white, glistening bumps on her infant's gums. She says they look like teeth, but she is worried that they may be cancer. You diagnose these bumps as A. Bednar's aphthae B. Epstein's pearls C. Buccal tumors D. Exostosis

B. Epstein's pearls

A 65 year old man presents complaining of left sided deep, throbbing headache along with mild fatigue. On examination the client has a tender, tortuous temporal artery. You suspect temporal arteritis. How do you confirm your diagnosis? A. MRI fo the head B. Erythrocyte sedimentation rate (ESR) C. EEG D. Otoscopy

B. Erythrocyte sedimentation rate (ESR)

You have made a diagnosis of acute sinusitis based on Martha's history and the fact that she complains of pain behind her eye. Which sinuses are affected? A. Maxillary B. Ethmoid C. Frontal D. Sphenoid

B. Ethmoid - Maxillary sinus pain is over the cheek and into the upper teeth - Frontal sinus pain is over the lower forehead - Sphenoid sinus pain is in the occiput, vertex, or middle of the head

Maggie, a 56 year old woman, comes to the office requesting a test for thyroid disease. She has had some weight gain since menopause and she read on the internet that all women should have a thyroid test. Based on the recommendations from the U.S. Preventive Service Task Force, which one of the following statements should be considered in this woman's care? A. All adults should be screened for thyroid disease B. Evidence is insufficient for or against routine screening for thyroid disease in asymptomatic adults C. All adults older than 50 should be screened for thyroid disease D. All perimenopausal women should be screened for thyroid disease

B. Evidence is insufficient for or against routine screening for thyroid disease in asymptomatic adults

The three most common signs and symptoms of primary HIV infection are A. Weight loss, pharyngitis, and fatigue B. Fever, fatigue, and pharyngitis C. Night sweats, rash, and headache D. Myalgias, fatigue, and fever

B. Fever, fatigue, and pharyngitis

What is the term describing an auscultation sound at the mediastinum in the presence of a mediastinal "crunch" that coincides with cardiac systole and diastole? A. Homan's sign B. Hamman's sign C. Manubrium's sign D. Louis' sign

B. Hamman's sign - It is present with spontaneous mediastinal emphysema or pneumomediastinum

What is the term describing an auscultation sound at the mediastinum in the presence of a mediastinal "crunch" that coincides with cardiac systole and diastole? A. Homan's sign B. Hamman's sign C. Manubrium's sign D. Louis' sign

B. Hamman's sign - It is present with spontaneous mediastinal emphysema or pneumomediastinum

Abe, age 57, has just been given a diagnosis of herpes zoster. He asks you about exposure to others. You tell him that A. Once he has been on the medication for a full 24 hours, he is no longer contagious B. He should stay away from children and pregnant women who have not had chickenpox. C. He should wait until the rash is completely gone before going out in crowds. D. He should be isolated from all persons except his wife.

B. He should stay away from children and pregnant women who have no had chickenpox

Abe, age 57, has just been given a diagnosis of herpes zoster. He asks you about exposure to others. You tell him that A. Once he has been on the medication for a full 24 hours, he is no longer contagious B. He should stay away from children and pregnant women who have not had chickenpox. C. He should wait until the rash is completely gone before going out in crowds. D. He should be isolated from all persons except his wife.

B. He should stay away from children and pregnant women who have not had chickenpox

Which type of hepatitis is transmitted by the fecal-oral route; is rare in the United States but endemic in Southeast Asia, India, North Africa, and Mexico; and does not progress to chronic liver disease? A. Hepatitis D (HDV) B. Hepatitis E (HEV) C. Hepatitis C (HCV) D. Hepatitis A (HAV)

B. Hepatitis E (HEV)

Elizabeth, age 83, presents with a 3 day history of pain and burning in the left forehead. This morning she noticed a rash with erythematous papules in that site. What do you suspect? A. Varicella B. Herpes zoster C. Syphilis D. Rubella

B. Herpes zoster

The presence of which type of cast in the urine is not indicative of renal disease? A. Red cell casts B. Hyaline casts C. Renal tubular cell casts D. White cell casts

B. Hyaline casts

A platelet count less than 150,000/mm3 may indicate A. Possible hemorrhage B. Hypersplenism C. Polycythemia vera D. Malignancy

B. Hypersplenism - As well as possible bone marrow failure or accelerated consumption of platelets

Jill has just been given a diagnosis of HIV infection and has a normal initial Pap test. When do the Centers for Disease Control and Prevention (CDC) guidelines state that she should have a repeat Pap test? A. In 3 months B. In 6 months C. In 1 year D. She should have a colposcopy every year rather than a Pap test

B. In 6 months - CDC recommends that if a woman infected with HIV has a normal initial Pap test, then a second evaluation should be done in 6 moths to reduce the likelihood of a false-negative initial test.

Which drug category contains the drugs that are the first line of therapy for COPD? A. Corticosteroids B. Inhaled beta-2 agonist bronchodilators C. Inhaled anticholinergic bronchodilators D. Xanthines

B. Inhaled beta-2 agonist bronchodilators - All the drugs listed may be appropriate for COPD, but inhaled short-acting beta-2 agonist bronchodilators are the first line of therapy.

Maury, age 52, has throbbing pain in the left eye, an irregular pupil shape, marked photophobia, and redness around the iris. What is your initial diagnosis? A. Conjunctivitis B. Iritis C. Subconjunctival hemorrhage D. Acute glaucoma

B. Iritis

Ipratropium is very widely used in treatment of COPD. Which of the following statements about ipratropium is correct? A. It slows the progression of COPD B. It decreases parasympathetic tone and produces bronchodilation C. It has anti-inflammatory actions and reduces bronchoconstriction D. It is less effective than a beta agonist in producing bronchodilation

B. It decreases parasympathetic tone and produces bronchodilation - It produces its helpful effects by reducing cholinergic tone in the lungs.

Sara, age 92, presents with dry eyes, redness, and a scratchy feeling. You note that this is one of the most common disorders, particularly in older women, and diagnose this as A. Viral conjunctivitis B. Keratoconjunctivitis sicca C. Allergic eye disease D. Corneal ulcer

B. Keratoconjunctivitis sicca

Michael, age 52, has a dry cough, dyspnea, chills, fever, general malaise, headache, confusion, anorexia, diarrhea, myalgias, and arthralgias. Which diagnosis do you suspect? A. Bronchopneumonia B. Legionnaire's Disease C. Primary atypical pneumonia D. Pneumocystitis jiroveci pneumonia

B. Legionnaire's Disease - Has a gradual onset

The major laboratory abnormality noted in patients with pneumonia is: A. Eosinophilia B. Leukocytosis C. Gram stain positive D. Leukopenia

B. Leukocytosis - An increased white count is typical in patients with pneumonia. This is more commonly seen in bacterial pneumonia.

The five Ps - purple, polygonal, planar, pruritic papules - are present in A. Ichthyosis B. Lichen planus C. Atopic dermatitis D. Seborrheic dermatitis

B. Lichen Planus

The five Ps - purple, polygonal, planar, pruritic papules - are present in A. Ichthyosis B. Lichen planus C. Atopic dermatitis D. Seborrheic dermatitis

B. Lichen planus - Has a primary skin lesion with the 5 Ps that looks like a shiny, violaceous, flat-topped papule that is very pruritic

Jerry, age 52, has gout. What do you suggest? A. Using salicylates for an acute attack B. Limiting consumption of purine-rich foods C. Testing his uric acid level every 6 months D. Decreasing fluid intake

B. Limiting consumption of purine-rich foods

Nevi arise from A. Plugged follicles B. Melanocytes C. Capillary occlusion D. Epithelium

B. Melanocytes

A 42 year old stockbroker comes to your office for evaluation of a pulsating headache over the left temporal region and he rates the pain as an 8 on a scale of 1-10. The pain has been constant for the past several hours and is accompanied by nausea and sensitivity to light. He has had frequent headaches for many years but not as severe and they are usually relieved by over the counter medicines. He is unclear as to a precipitating event but notes that he has had visual disturbances before each headache and has been under a lot of stress in his job. Based on this description, what is the most likely diagnosis of this type of headaache? A. Tension B. Migraine C. Cluster D. Temporal arteritis

B. Migraine

All of the following medications may cause alopecia except A. Warfarin (Coumadin) B. Minoxidil (Rogaine) C. Levonorgestrel (Norplant) D. Acetylsalicylic acid (aspirin)

B. Minoxidil

What is the earliest visual sign of oral and pharyngeal squamous cell carcinoma? A. Leukoplakia B. Mucosal erythroplasia C. Loss of sensation in the tongue D. Difficulty chewing or swallowing

B. Mucosal erythroplasia - Red inflammatory lesions. Leukoplakia (thickened white patches of the tongue or mucous membranes) is the most common premalignant lesion.

The most common cause of atypical pneumonia in adults is: A. Streptococcus pneumoniae B. Mycoplasma pneumoniae C. Chlamydophila pneumoniae D. Staphylococcus pneumonia

B. Mycoplasma pneumoniae - In patients who have atypical pneumonia, Mycoplasma is the most common pathogen.

In teaching your client about the American Cancer Society's CAUTION model, which identifies signs of many cancers, you teach her that the N stands for A. Night sweats B. Nagging cough C. Nausea and vomiting D. Noxious odor

B. Nagging cough - Or hoarseness

Marnie, who has asthma, has been told that she has nasal polyps. What do you tell her about them? A. Nasal polyps are usually precancerous B. Nasal polyps are benign growths C. The majority of nasal polyps are neoplastic D. They are probably inflamed turbinates, not polyps, because polyps are infrequent in clients with asthma

B. Nasal polyps are benign growths

Mary, age 82, presents with several eye problems. She states that her eyes are always dry and look "sunken in." What do you suspect? A. Hypothyroidism B. Normal age-related changes C. Cushing's syndrome D. A detached retina

B. Normal age-related changes

Pernicious anemia is a result of A. Not enough folic acid B. Not enough intrinsic factor C. Not enough Vitamin D D. Not enough iron

B. Not enough intrinsic factor

Pernicious anemia is a result of A. Not enough folic acid B. Not enough intrinsic factor C. Not enough Vitamin D D. Not enough iron

B. Not enough intrinsic factor. - Pernicious anemia is a result of the parietal cells of the stomach lining failing to secrete enough intrinsic factor to ensure intestinal absorption of Vitamin B12.

Which statement about otitis media with effusion (OME) is correct? A. OME usually needs treatment with antibiotics B. OME can precede or follow otitis media C. Otitis media is far more common than OME D. Otitis media and OME are usually associated with fever

B. OME can precede or follow otitis media - OME is far more common than otitis media and is not associated with systemic symptoms like fever.

Which of the following characteristics is always present in a patient with COPD? A. Productive cough B. Obstructive airways C. Shortness of breath D. Hypercapnea

B. Obstructive airways - The obstruction is NOT completely reversible. Asthma is completely reversible.

Which is the drug of choice for tinea capitis? A. A topical corticosteroid B. Oral griseofulvin (Grisactin) C. A topical antifungal D. An antibiotic

B. Oral griseofulvin (Grisactin)

Stu, age 49, has slightly reduced hemoglobin and hematocrit readings. What is your next action after you ask him about his diet? A. Repeat the laboratory tests B. Perform a fecal occult blood test C. Start him on an iron preparation D. Start him on folic acid

B. Perform a fecal occult blood test - Tests for fecal occult blood in the stools should be done on all clients suspected of having iron-deficiency anemia.

Which of the following is not an inherited condition that causes hemolytic anemia? A. Hereditary spherocytosis B. Pernicious anemia C. Glucose-6-phosphate dehydrogenase deficiency D. Sickle cell anemia

B. Pernicious anemia - Caused by an inadequate absorption of vitamin B12. Symptoms develop slowly and subtly and may not be recognized right away.

Zinc oxide, magnesium silicate, ferric chloride, and kaolin are examples of A. Chemical sunscreens B. Physical sunscreens C. Agents used in tanning booths D. Emollients

B. Physical sunscreens - Reflect and scatter ultraviolet light.

A "herald patch" is a hallmark finding in which condition? A. Erythema infectiosum B. Pityriasis rosacea C. Seborrheic keratosis D. Atopic dermatitis

B. Pityriasis rosacea - Pityriasis rosa is a self-limiting exanthematous skin disorder characterized by several unique findings. It is more common in young adults. A characteristic finding is the "herald" or "mother" patch found on trunk. This looks like a ringworm and precedes the generalized "Christmas tree" pattern rash. Lesions are salmon-colored & oval. Most resolve in 4-6 weeks.

Jamie has her asthma will controlled by using only a beta-adrenergic metered-dose inhaler. Lately, however, she has had difficulty breathing during the night and her sleep has been interrupted about three times a week. What do you do? A. Prescribe a short course of steroid therapy B. Prescribe an inhaled steroid C. Prescribe a longer-acting bronchodilator D. Prescribe oral theophylline

B. Prescribe an inhaled steroid - Moderate asthma is defined as more than two episodes per week. An inhaled steroid should be prescribed and used in conjunction with the beta-2 adrenergic metered dose inhaler.

Joy, age 36, has a sudden onset of shivering, sweating, headache, aching in the orbits, and general malaise and misery. Her temperature is 102 F. You diagnose influenza (flu). What is your next course of action? A. Order amoxicillin (Amoxil) 500mg every 12 hours for 7 days B. Prescribe rest, fluids, acetaminophen (Tylenol), and possibly a decongestant and an antitussive C. Order a complete blood count D. Consult with your collaborating physician

B. Prescribe rest, fluids, acetaminophen (Tylenol), and possibly a decongestant and an antitussive

The purpose of transparent dressing such as a Tegaderm applied over a pressure ulcer is to A. Toughen intact skin and preserve skin integrity B. Prevent skin breakdown and the entrance of moisture and bacteria but allow permeability of oxygen and moisture vapor C. Allow necrotic material to soften D. Use the proteolytic enzymes in the dressing to serve as a debriding agent

B. Prevent skin breakdown and the entrance of moisture and bacteria but allow permeability of oxygen and moisture vapor

The T in the TNM staging system refers to A. Tolerance B. Primary tumor C. Tumor marker D. Turgor

B. Primary tumor - N is for regional lymph nodes, and M is for distant metastasis

Sol, age 46, is overweight and comes in with dyspnea. He also has peripheral edema, ascites, and neck vein distention. What do you further evaluate him for? A. Anemia B. Pulmonary hypertension C. Pleural effusion D. Central nervous system lesion

B. Pulmonary hypertension

The lesions seen in a patient with folliculitis might be filled with: A. Blood B. Pus C. Fluid D. Serum

B. Pus

The most common cause of a persistent cough in children of all ages is A. An allergy B. Recurrent viral bronchitis C. Asthma D. An upper respiratory infection

B. Recurrent viral bronchitis

Ty, age 68, has a hearing problem. He tells you he is ready for a drastic solution to the problem because he likes to play bingo but cannot hear the calls. What can you do for him? A. Refer him to a hearing aid specialist B. Refer him for further testing C. Perform a gross hearing test in the office, then repeat in 6 months to determine if there is any further loss D. Nothing. Tell him that a gradual hearing loss is to be expected with aging

B. Refer him to further testing

What early acid-base disturbance occurs in a teenager admitted for aspirin overdose? A. Respiratory acidosis B. Respiratory alkalosis C. Metabolic acidosis D. Metabolic alkalosis

B. Respiratory alkalosis - It results from direct stimulation of the respiratory center in the medulla, which causes an increase in pH and a decrease in PaCO2.

The most common causative bacterial pathogen in ABRS is: A. M. pneumoniae B. S. pneumoniae C. M. catarrhalis D. H. influenzae

B. S. pneumoniae

A patient's tympanic membrane reveals a chalky white mark. The patient has no complaints of ear pain or decreased hearing. This is probably: A. A normal tympanic membrane B. Scarring of the tympanic membrane C. Chronic inflammation D. Pus in the middle ear

B. Scarring of the tympanic membrane

Judy has come to have an influenza immunization today. Which of the following would be a contraindication for her to receive the injection? A. She is pregnant B. She has a temperature of 102F (38.8C) C. She has history of neurological reaction D. She had a previous anaphylactic reaction to neomycin and streptomycin

B. She has a temperature of 102F (38.8C)

Which type of hemangioma in a newborn occurs on the nape fo the neck and is usually not noticeable when it becomes covered by hair? A. Nevus flammeus (port-wine stain) B. Stork's beak mark C. Strawberry hemangioma D. Cavernous hemangioma

B. Stork's beak mark

Which type of hemangiroma in a newborn occurs on the nape fo the neck and is usually not noticeable when it becomes covered by hair? A. Nevus flammeus (port-wine stain) B. Stork's beak mark C. Strawberry hemangioma D. Cavernous hemangioma

B. Stork's beak mark

Rose, age 66, comes in with an intractable headache accompanied by weakness, difficulty chewing, and visual changes. You note some swelling and tenderness on her left forehead. What do you suspect? A. A migraine headache B. Temporal arteritis C. A cluster headache D. A cerebral aneurysm

B. Temporal arteritis

When a practitioner places a vibrating tuning fork in the midline of a client's skull and asks if the tone sounds the same in both ears or is better in one, the examiner is performing A. The Rinne test B. The Weber test C. The caloric test D. A hearing acuity test

B. The Weber test

What is the definition of the spirometric assessment of residual volume? A. The sum of the vital capacity and the residual volume B. The amount of gas left in the lung after exhaling all that is physically possible C. The volume that can be maximally exhaled after a passive exhalation D. The measurement of the maximum flow rate achieved during the forced vital capacity maneuver

B. The amount of gas left in the lung after exhaling all that is physically possible

While doing a face, head, and neck examination, you note that the palpebral fissures are abnormally narrow. What are you examining? A. Nasolabial folds B. The openings between the margins of the upper and lower eyelids C. The thyroid gland in relation to the trachea D. The distance between the trigeminal nerve branches

B. The opening between the margins of the upper and lower eyelids

A patient has been given penicillin V three times daily for 10 days for sore throat. On day 9, the patient is feeling better but has developed a pruritic full body rash. What is a likely clinical finding? A. The rash will be fine and papular B. There will be hives C. There will be large, splotchy, non-pruritic areas D. The rash will not blanch

B. There will be hives

Stephen, age 18, present with a pruritic rash on his upper trunk and shoulders. You observe flat to slightly elevated brown papules and plaques that scale when they are rubbed. You also note areas of hypopigmentation. What is your initial diagnosis? A. Lentigo syndrome B. Tinea versicolor C. Localized brown macules D. Ochronosis

B. Tinea versicolor

Stephen, age 18, present with a pruritic rash on his upper trunk and shoulders. You observe flat to slightly elevated brown papules and plaques that scale when they are rubbed. You also note areas of hypopigmentation. What is your initial diagnosis? A. Lentigo syndrome B. Tinea versicolor C. Localized brown macules D. Ochronosis

B. Tinea versicolor

You are seeing a 25-year-old man with S. pyogenes pharyngitis. He asks if he can get a "shot of penicillin" for therapy. You consider the following when counseling about the use of intramuscular penicillin: A. There is nearly a 100% cure rate in streptococcal pharyngitis when it is used B. Treatment failure rates approach 20% C. It is the preferred agent in treating group G streptococcal infection D. Injectable penicillin has a superior spectrum of antimicrobial coverage compared with the oral version of the drug

B. Treatment failure rates approach 20%

Mackenzie, age 8, is complaining of something being stuck inside her ear. You consider a number of actions. The action you should not do is which of the following? A. Instill several drops of mineral oil B. Use a small ear syringe to suction out the item C. Inspect the inside of the ear with your otoscope even though Mackenzie is crying in pain D. Flush the ear with water

B. Use a small ear syringe to suction out the item

Nathan, age 19, is a college swimmer. He frequently gets swimmer's ear and asks if there is anything he can do to help prevent it other than wearing ear plugs, which don't really work for him. What do you suggest? A. Use a cotton-tipped applicator to dry the ears after swimming B. Use eardrops made of a solution of equal parts of alcohol and vinegar in each ear after swimming C. Use a hair dryer on the highest setting to dry the ears D. Tell Nathan he must change his sport

B. Use eardrops made of a solution of equal parts of alcohol and vinegar in each ear after swimming

Sandra, age 69, is complaining of dry skin. What do you advise her to do? A. Bathe every day B. Use tepid water and a mild cleansing cream C. Use a dehumidifier D. Decrease the oral intake of fluids

B. Use tepid water and a mild cleansing cream - Advise the patient that it is not necessary to take a bath every day because soap and hot water are drying to the skin

Beulah, age 86, has stasis dermatitis of her left lower leg. Her ankle is edematous and there is a rusty, brownish discoloration of the skin. What is your diagnosis? A. Arterial valvular insufficiency B. Venous Valvular insufficiency C. Cellulitis D. Diabetic nephropathy

B. Venous Valvular insufficiency

What is the name of the acquired disorder characterized by complete loss of pigment of the involved skin? A. Tinea versicolor B. Vitiligo C. Tuberous sclerosis D. Pityriasis alba

B. Vitiligo

What is the name of the acquired disorder characterized by complete loss of pigment of the involved skin? A. Tinea versicolor B. Vitiligo C. Tuberous sclerosis D. Pityriasis alba

B. Vitiligo - Usually appears in childhood

Which of the following symptoms is more indicative of a bacterial sinusitis than viral? A. Discolored nasal discharge B. Worsening of the symptoms after improvement C. Facial pressure D. Nasal congestion

B. Worsening of the symptoms after improvement - There are no specific signs or symptoms that can clearly differentiate viral from bacterial sinsusitis. However, rhinosinusitis symptoms lasting greater than 7 days and purulent nasal discharge, unilateral facial pain or maxillary tooth pain, and worsening of symptoms after initial improvement are suggestive of bacterial infection.

Who is the best candidate for isotretinoin (Acutane) therapy: A. a 17 year old patient with pustular lesions and poor response to benzoyl peroxide B. a 20 year old patient with cystic lesions who has tried various therapies with minimal effect C. a 14 year old patient with open and closed comedones and a family history of "ice pick" scars D. an 18 year old patient with inflammatory lesions and imporvement with tretinoin (Retin-A)

B. a 20 year old patient with cystic lesions who has tried various therapies with minimal effect

A 72 year old woman presents with a newly formed painless, pearly, ulcerated nodule with an overlying telangiectasis on the upper lip. This most likely represents: A. an actinic keratosis B. a basal cell carcinoma C. a squuuamus cell carcinoma D. mmmolluscum conttagiosuuum

B. a basal cell carcinoma

The older adult is at greater risk for developing an eye infection than a younger person. This is because of the age-related change of A. change in vitreous humor B. a decrease in tear production C. loss of subcutaneous tissue D. greater emotionality

B. a decrease in tear production

The initial diagnostic procedure for evaluation of a deep venous thrombosis is A. a contrast venography B. a duplex ultrasound C. an arteriogram D. an MRI of the affected limb

B. a duplex ultrasound

A 22-year-old woman presents with a "pimple" on her right eyelid. Examination reveals a 2-mm pustule on the lateral border of the right eyelid margin. This is most consistent with: A. a chalazion B. a hordeolum C. blepharitis D. cellulitis

B. a hordeolum

Antihistamines work primarily through: A. vasoconstriction B. action on the histamine-1 (H1) receptor sites C. inflammatory mediation D. peripheral vasodilation

B. action on the histamine-1 (H1) receptor sites

According to the Allergic Rhinitis and Its Effects on Asthma (ARIA) treatment guidelines, which of the following medications affords the best relief of acute nasal congestion? A. anticholinergic nasal spray B. decongestant nasal spray C. corticosteroid nasal spray D. oral antihistamine

B. decongestant nasal spray

Which of the following best describes seborrheic dermatitis lesions: A. flaking lesions in the antecubital and popliteal spaces B. greasy, scaling lesions in the nasolabial folds C. intensely itchy lesions in the groind folds D. silvery lesions on the elbows and knees

B. greasy, scaling lesions in the nasolabial folds

The likely causative organisms of nonbullous impetigo in a 6 year old child include: A. H. influenza and S. pneumoniae B. group A streptococcus and S. aureus C. M. catarrhalis and select viruses D. P. aeruginosa and select fungi

B. group A streptococus and S. aureus

The most appropriate pharmacologic intervention for treating ABRS in a 45-year-old person who is moderately ill with the condition is: A. erythromycin B. high-dose amoxicillin with clavulunate C. cephalexin D. ciprofloxacin

B. high-dose amoxicillin with clavulunate

Risk factors for malignant melanoma include: A.. Asian ancestry B. history of blistering sunburn C. family history of psoriasis vulgariss D presence of atopic dermatitis

B. history of blistering sunburn

Physical examination findings in otitis externa include: A. tympanic membrane immobility B. increased ear pain with tragus palpation C. tympanic membrane erythema D. tympanic membrane bullae

B. increased ear pain with tragus palpation

When counseling a person who has a 2mm verruca-form lesion on the hand you advice that: A bacteria are the most common cause of these lesions. B.lesions usually resolve without therapy in 12 to 24 months C. there is a significant risk for further dermatologic malignancy D. surgical excision is the treatment of choice

B. lesions usually resolve without therapy in 12 to 24 mmontths

A 34-year-old man with penicillin allergy presents with ABRS. Three weeks ago, he was treated with doxycycline for "bronchitis". You now prescribe: A. clarithromycin B. levofloxacin C. cephalexin D. amoxicillin

B. levofloxacin

Which of the following is a characteristic of S. pneumoniae? A. mechanism of antimicrobial resistance primarily due to the production of beta-lactamase B. mechanism of antimicrobial resistance usually via altered protein-binding sites held within the microbe's cell C. organisms most commonly isolated from mucoid middle ear effusion D. gram-negative organisms

B. mechanism of antimicrobial resistance usually via altered protein-binding sites held within the microbe's cell

A woman was treated as an inpatient for a serious soft tissue infection with parenteral linezolid and now is being seen on day 3 of her illness and is being discharged to hom. She is feeling better and appears by examination to be clinically improved. Culture results reveal MRSA, sensitive to trimethoprim-sulfamethoxazole, linezolid, daptomycin, vancomycin, and clindamycin, and resistant to cephalothin and erythromycin. Her antimicrobial therapy should be completed with: A. oral cephalexin B. oral trimethoprim-sulfamethaxozale C. parenteral linezolid D oral linezolid

B. oral trimethoprim-sulfamethaxozale

Psoriatic lesions arise from: A. decreased skin exfoliation B. rapid skin cell turnover, leading to decreased maturation and keratinization C. inflammatory changes of the dermis D. lichenification

B. rapid skin cell turnover, leading to decreased maturation and keratinization

Mr. Albert has a chronic leukemia and currently is experiencing a low platelet count. This is usually manifested as A. multiple macular lesions in sun-exposed area B. red, flat, nonblanchable petechiae C. numerous brown, nonscaly macules that become more prominent after exposure to the sun D. an irregular border and a black lesion

B. red, flat, nonblanchable petechiae

You examine a patient with psoriasis vulgaris and expect to find the following lesions: A. lichenified areas in flexor areas B. red, well demarcated plaques on the knees C. greasy lesions throughout the scalp D vesicular lesions over the upper thorax

B. red, well-demarcated plaques on the knees

Karen, age 43, was recently in a car accident but is now up and about. She comes into the office with vague complaints of being tired. You assess Cullen's sign and Grey Turner's sign. You assume therefore that she has A. anemia B. retroperitoneal bleeding C. portal hypertension D. liver dysfunction

B. retroperitoneal bleeding

The spectum of antimicrobial activity of mupirocin (bactroban) includes: A. primarily gram negative organisms B. select gram-positive organisms C. Pseudomonas species and anaerobic organisms D. only organisms that do not produce beta-lactamase

B. select gram-positive organisms

Risk factors for the development of postherpetic neuralgia include: A. age younger than 50 years at the time of the outbreak B. severe prodromal symptoms C. lumbar location o lesions D. low volume of lesions

B. severe prodromal symptoms

Classic psoriasis lesions exhibit A. diffuse macules over the torso B. sharply marginated plaques and papules with marked silvery-white scales C. ulcerations on flexor surfaces of the extremities D. transient thin-roofed vesicles with honey-colored crusts

B. sharply marginated plaques and papules with marked silvery-white scales

Which of the following best describes hearing loss associated with presbycusis? A. rapidly progressive, often asymmetric, and in all frequencies B. slowly progressive, usually symmetric, and predominantly high frequency C. variable in progression, usually unilateral, and in the midrange frequencies D. primarily conductive and bilateral with slow progress

B. slowly progressive, usually symmetric, and predominantly high frequency

The TNM staging system for cancer assists in guiding therapeutic choices. The T in TNM stands for A. the type of tumor B. the extent of the primary tumor C. the length of time the tumor has been in existence D. how solid the tumor feels to touch

B. the extent of the primary tumor

When advising a patient with scarlet fever, the NP considers that: A. there is increased risk for poststreptococcal glomerulonephritis B. the rash often peels during recovery C. an injectable cephalosporin is the preferred treatment option D. throat culture is usually negative for group A streptococci

B. the rash often peels during recovery

When counseling a patient about pimecrolimus (Elidel) use; you mention that: A. this is the preferred atopic dermatitis treatment in infants B. there is a possibility of increased cancer risk with its use C. the product is used interchaneably with topical corticosteroids D. the product is a potent antihistaine

B. there is a possibility of increased cancer risk with its use

A good technique for assessment of the supraclavicular nodes is A. to have the client sit up and perform Valsalva's maneuver B. to use the diaphragm of the stethescope C. to palpate the axilla D. to use the bell of the stethescope

B. to use the diaphragm of the stethescope

Treatment options for streptococcal pharyngitis for a patient with penicillin allergy include all of the following except: A. azithromycin B. trimethoprim-sulfamethoxazole C. clarithromycin D. erythromycin

B. trimethoprim-sulfamethoxazole

Expected findings in AOM include: A. prominent bony landmarks B. tympanic membrane immobility C. itchiness and crackling in the affected ear D. submental lymphadenopathy

B. tympanic membrane immobility

When prescribing Retin-A, the NP advises the paient to: A. use it with benzoyl peroxide to minimize irritating effects B. use a sunscreen because the drug is photosensitizing C add a sulfa-based cream to enhance antiacne effects D. expect a significant improvement in acne lesions after approximately one week of use

B. use a sunscreen because the drug is photosensitizing

Which of the following is the most frequent cause of stasis ulcer: A. arterial insufficiency B. venous insufficiency C. diabetes mellitus D. fungal dermatitis

B. venous insuficiency

A mailman reports that he becomes nasally congested when he exits buildings when delivering packages. He states that the symptoms are annoying and occur during the spring and summertime when temperatures are warm. He likely has: A. Seasonal allergic rhinitis B. Perennial allergic rhinitis C. Chronic non-allergic rhinitis D. Rhinitis medicamentosa

C. Chronic non-allergic rhinitis - This patient has symptoms of non-allergic rhinitis, often called vasomotor rhinits. There is no allergic reaction involved although sneezing, rhinorrhea, nasal congestion, and post nasal drainage are typical. Possibly due to inappropriate response of the autonomic nervous system that innervates the nose.

A risk factor for pulmonary embolism in women includes which of the following? A. Extreme thinness B. Alcohol intake C. Cigarette smoking D. Hypotension

C. Cigarette smoking - Obesity, cigarette smoking and hypertension increased the risk for pulmonary embolism.

The first choice of therapy for a client who is positive for HIV and has oral candidiasis is A. Fluconazole (Diflucan) 100mg PO QD B. Ketoconazole (Nizoral) 200mg PO QD C. Clotrimazole troches (10mg) five times daily or nystatin (Mycostatin) suspension 500,000-1,000,000 units three to five times daily D. Griseofulvin (Grisactin) 500mg BID

C. Clotrimazole troches (10mg) five times daily or nystatin (Mycostatin) suspension 500,000-1,000,000 units three to five times daily - Because of the common recurrence of oral candidiasis and increased rates of drug resistance, systemic fungicides, such as fluconazole, ketoconazole, and griseofulvin, should be reserved for severe cases, such as esophageal candidiasis and clients with dysphagia.

In inner-city children, an important cause of asthma-related illness and hospitalizations is A. Heredity and genetics B. Vitamin deficiencies C. Cockroaches D. Playing on asphalt playgrounds

C. Cockroaches

What therapy has proved beneficial for long-term symptom relief of tinnitus? A. Aspirin B. Lidocaine C. Cognitive behavioral therapy D. Corticosporin otic gtts PRN

C. Cognitive behavioral therapy

Which of the following warts (HPV) looks like a cauliflower and are usually found in the anogenital region? A. Plantar warts B. Filiform and digitate warts C. Condyloma acuminata D. Verruca plana

C. Condyloma acuminata

Which of the following warts (HPV) looks like a cauliflower and are usually found in the anogenital region? A. Plantar warts B. Filiform and digitate warts C. Condyloma acuminata D. Verruca plana

C. Condyloma acuminata - Usually sexually transmitted

Before initiating cancer therapy, the first crucial step is to A. Stage the disease B. Define the goals of therapy C. Confirm the diagnosis using tissue biopsy D. Choose a treatment plan from the many therapeutic options

C. Confirm the diagnosis using tissue biopsy

Unexplained nocturnal cough in an older adult should suggest A. Allergies B. Asthma C. Congestive heart failure D. Viral syndrome

C. Congestive heart failure

Which drug contributes to a decreased response to tuberculin skin testing (TST)? A. Antibiotics B. Inhaled allergy medications C. Corticosteroids D. Birth control pills

C. Corticosteroids

Robin has HIV infection and is having a problem with massive diarrhea. You suspect the cause is A. Cryptococcosis B. Toxoplasmosis C. Cryptosporidiosis D. Cytomegalovirus

C. Cryptosporidiosis - When patients with HIV infection have massive diarrhea, a protozoa of the cryptosporidium genus is the most likely cause.

Mrs. Jameson complains of unilateral blurry vision and partial blindness in the left eye. On physical examination, you find decreased peripheral vision on her left side. Funduscopic examination reveals cotton-wool spots. Your most likely diagnosis is A. Cryptococcosis B. Toxoplasmosis C. Cytomegalovirus infection D. Herpes simplex virus infection

C. Cytomegalovirus infection - Classic S&S include cotton-wool spots (cottage cheese and ketchup appearance), hemorrhage, and exudates on funduscopic examination

A patient with acute bronchitis was diagnosed at an urgent care center 10 days ago. He reports that he was given an anti-tussive for nighttime, a steroid injection, steroids and an antibiotic by mouth for 5 days, and a decongestant. Which of these interventions was of greatest benefit in relief of his symptoms? A. Steroid injection and oral steroids B. Antibiotic C. Decongestant and antitussive D. Antibiotic and steroids

C. Decongestant and anti-tussive - Beneficial for symptomatic relief only

What assessment finding indicates sarcoidosis? A. Use of accessory muscles B. Increased resistance to airflow into the lungs C. Decreased lung compliance D. Increased vital capacity and total lung capacity

C. Decreased lung compliance

Marvin has sudden eye redness that occurred after a strenuous coughing episode. You diagnose a subconjunctival hemorrhage. Your next step is to A. Refer him to an ophthalmologist B. Order antibiotics C. Do nothing other than provide reassurance D. Consult with you collaborating physician

C. Do nothing other than provide reassurance

What is the "gold standard" used to confirm the suspicion of a true food allergy (IgE reaction) in a young child? A. Immediate-reacting IgE skin test B. Food challenge C. Double-blind, placebo-controlled food challenge D. Diagnostic food diet diary and home challenge

C. Double-blind, placebo-controlled food challenge

A patient reports that he found a tick on himself about one month ago. He was left with a red circle and a white center. He did not seek treatment. Today he complains of numbness, peripheral paresthesias, and poor concentration. What laboratory test can be used to help diagnose Lyme disease? A. CBC B. Lyme titer C. ELISA D. Cerebrospinal fluid analysis for the spirochete

C. ELISA - Historical features coupled with physical exam support the diagnosis, and thus, screening. The most common intitial serologic test for screening is an ELISA. If it is positive, it should be confirmed with a Western Blot. Unfortunately, there are a large number of false positives and so a confirmation should be performed.

Other than smoking cessation, which of the following slows the progression of COPD in smokers? A. Making sure the environment is free of all pollutants B. Eliminating all pets from the environment C. Engaging in moderate to high levels of physical activity D. Remaining indoors with air conditioning as much as possible

C. Engaging in moderate to high levels of physical activity

Leah, 4 months old, has both eyes turning inward. What is this called? A. Pseudostrabismus B. Strabismus C. Esotropia D. Exotropia

C. Esotropia

How often should a CD4 count be evaluated in a client with AIDS if the client's previous levels have been greater than 500? A. Every month B. At every visit C. Every 6 months D. Annually

C. Every 6 months

Which of the following signs of thyroid dysfunction is a sign of hyperthyroidism? A. Slow pulse B. Decreased systolic BP C. Exophthalmos D. Dry, coarse, cool skin

C. Exopthalmus

Martin, age 24, presents with an erythematous ear canal, pain, and a recent history of swimming. What do you suspect? A. Acute otitis media B. Chronic otitis media C. External otitis D. Temporomandibular joint syndrome

C. External otitis

You write a prescription for a topical agent and anticipate the greatest rate of absorption when it is applied to the: A. Palms of the hand B. Soles of the feet C. Face D. Abdomen

C. Face

Which is the best serum test to perform to spot an iron-deficiency anemia early before it progresses to full-blown anemia? A. Hemoglobin B. Hematocrit C. Ferritin D. Reticulocytes

C. Ferritin - A serum measurement of ferritin, the body's iron-storing protein, can tell exactly how much iron is on hand in the body.

Which patient can be appropriately diagnosed with acute otitis media? A. Decreased mobility of the tympanic membrane (TM) B. Visible bubbles behind the TM C. Fluid and bulging of the TM D. Marked redness of the TM

C. Fluid and bulging of the TM - The diagnosis of acute otitis media (AOM) requires the finding of a middle ear effusion AND a sign of acute inflammation, such as distinct fullness or bulging of the TM, ear pain, or marked redness of the TM. Both must be present for the diagnosis of AOM. The other finding that constitutes a diagnosis of AOM is the finding of acute, purulent otorrhea that is not due to otitis externa. This characterizes a ruptured TM from otitis media.

Acute otitis media is diagnosed when there is A. Fluid in the middle ear without signs or symptoms of an ear infection B. A diagnosis of three or more episodes of otitis media within 1 year C. Fluid in the middle ear accompanied by otalgia and fever D. Fluid within the middle ear for at least 3 months

C. Fluid in the middle ear accompanied by otalgia and fever

Cydney, age 7, is complaining that she feels as though something is stuck in her ear. What action is contraindicated? A. Inspecting the ear canal with an otoscope B. Using a small suction device to try to remove the object C. Flushing the ear with water D. Instilling several drops of mineral oil in the ear

C. Flushing the ear with water

A 62 year old obese woman comes in today complaining of difficulty swallowing for the past 3 weeks. She states that "some foods get stuck" and she has been having "heartburn" at night when she lies down, especially if she has had a heavy meal. Occasionally she will awake at night coughing. She denies weight gain and/or weight loss, vomiting, or change in bowel movements. She does not drink or smoke. There is no pertinent family history or findings on review of systems (ROS). Physical examination is normal with no abdominal tenderness, and the stool is OB negative. What is the most likely diagnosis? A. Esophageal varices B. Esophageal cancer C. Gastroesophageal reflux disease (GERD) D. Peptic ulcer disease

C. Gastroesophageal reflux disease (GERD)

Mavis, age 76, comes to the office with a unilateral throbbing headache in the periorbital region. She states that the pain has been gradually increasing over the past several hours and when she went out into the cold weather, the pain was extremely bad. What do you suspect? A. Trigeminal neuralgia B. A migraine C. Giant-cell arteritis D. A transient ischemic attack

C. Giant-cell arteritis

A patient describes a sensation that something is in his throat. He denies throat pain. His exam is normal. What diagnosis should the nurse practitioner make? A. Factitious sore throat B. Pseudo pharyngitis C. Globus D. GERD

C. Globus - Globus is the sensation that there is a lump or foreign body in the esophagus. It is not associated with sore throat or pain. If pain is present, this is not globus. The most common cause of globus is GERD, but this diagnosis cannot be made based on the information from the question.

Shane, age 15, arrives in your office with a chief complaint of a sore throat. Upon examination you observe tonsillar exudates, a fever of 102F, an absence of a cough, and anterior cervical adenopathy. Which causative agent is a high probability? A. Rhinovirus B. Epstein-Barr virus C. Goup A beta-hemolytic streptococcus D. Haemophilus influenzae

C. Goup A beta-hemolytic streptococcus

A mother complains that her newborn infant lying on his or her side may appear red on the dependent side of her body while appearing pale on the upper side. When she picks up the baby, this coloring disappears. You explain to her about A. A temporary hemangioma B. Hyperbilirubinemia C. Harlequin sign D. Mongolian spots

C. Harlequin sign

A patient with fever and pharyngitis has a negative rapid strep test. A preliminary throat culture sent to the laboratory reveals "normal flora after 24 hours." The patient: A. Probably has Strep and should be treated B. Probably has some bacterial pathogen C. Has a pharyngitis of undetermined etiology D. Should be treated with penicillin based on symptoms

C. Has a pharyngitis of undetermined etiology

A 45 year old patient describes a spinning sensation that lasts for several hours before abating. During this time, he complains of nausea and vomiting, and headache. Which symptoms is NOT characteristic of benign positional vertigo? A. The length of duration of symptoms B. Nausea and vomiting C. Headache D. Sensation of spinning

C. Headache - A spinning sensation is a typical description of a patient with benign positional vertigo. During acute attacks of vertigo, regardless of the etiology, nausea and vomiting are typical.

In a burn trauma, which blood measurement rises as a secondary result of hemoconcentration when fluid shifts from the intravascular compartment? A. Hemoglobin B. Sodium C. Hematocrit D. Blood urea nitrogen (BUN)

C. Hematocrit

Maurice is an intravenous drug abuser with chronic hepatitis B (HBV). The development of which type of hepatitis poses the greatest risk to a client with HBV? A. Hepatitis A B. Hepatitis C C. Hepatitis D D. Hepatitis E

C. Hepatitis D

When Judy tells you that she has hemophilia, you know that A. Both of her parents also have the disease B. Her maternal grandfather probably had the disease and it skipped a generation C. Her father had the disease and her mother was a carrier D. Her mother had the disease

C. Her father had the disease and her mother was a carrier - Hemophilia is a classic example of an X-linked recessive disease and, as a rule, only males are affected. In rare instances, female carriers are clinically affected if their normal X chromosomes are disproportionately inactivated.

Which is the most notable clinical manifestation of glottis (tongue) cancer? A. Earache B. Halitosis C. Hoarseness D. Frequent swallowing

C. Hoarsenss - The most notable manifestation of glottic (tongue) cancer is hoarseness or a change in the voice because the tumor prevents complete closure of the glottis during speech

Which is the most notable clinical manifestation of glottis (tongue) cancer? A. Earache B. Halitosis C. Hoarseness D. Frequent swallowing

C. Hoarsenss - The most notable manifestation of glottic (tongue) cancer is hoarseness or a change in the voice because the tumor prevents complete closure of the glottis during speech

A patient is diagnosed with new onset asthma. Which question is most important to ask when deciding on medication management? A. Do you smoke? B. How severe are your symptoms? C. How often do your symptoms occur? D. Do you ever wheeze?

C. How often do your symptoms occur? - Diagnosis of asthma, as well as pharmacologic management of asthma, is based on frequency of occurrence of symptoms.

Prostate cancer is associated with which of the following viruses? A. Herpes simplex virus types 1 and 2 B. Human herpesvirus 6 C. Human cytomegalovirus D. Human T-lymphotropic viruses

C. Human cytomegalovirus

Sally, age 8, has frequent episodes of epistaxis. What precaution strategy do you suggest to her mother? A. Vigorously blow her nose before going to bed to ensure a patent airway B. Start low-dose daily aspirin therapy C. Humidify the bedroom D. Tilt her head back and pinch her nose

C. Humidify the bedroom

Pregnant women may be prone to thrombophilias, which may be inherited or acquired. Which of the following is an example of a factor that predisposes pregnant women to acquired thrombophilic states in pregnancy? A. Factor V Leiden B. Homocystine C. Immobilization and malignancy D. Protein S and protein C

C. Immobilization and malignancy

Which treatment would you order for anogenital pruritus? A. Suppositories for pain B. Antifungal cream for itching C. A high-fiber diet for constipation D. Zinc oxide ointment

C. A high-fiber diet for constipation - Most cases of anogenital pruritus have no obvious cause and chiefly cause nocturnal itching without pain. Although the condition is benign, it may be persistent and recurrent

When the Weber test is performed with a tuning fork to assess hearing and there is no lateralization, this indicates A. Conductive deafness B. Perceptive deafness C. A normal finding D. Nerve damage

C. A normal finding

Harry, age 69, has had Meniere's disease for several years. He has some hearing loss but now has persistent vertigo. What treatment might be instituted to relieve the vertigo? A. A labyrinthectomy B. Pharmacological therapy C. A vestibular neurectomy D. Wearing an earplug in the ear with the most hearing loss

C. A vestibular neurectomy

A smooth tongue may indicate A. A normal finding B. Alcohol abuse C. A vitamin deficiency D. Nicotine addiction

C. A vitamin deficiency

Mindy, age 6, recently was discharged from the hospital after a sickle cell crisis. You are teaching her parents to be alert to the manifestations of splenic sequestration and tell them to be alert to A. Vomiting and diarrhea B. Decreased mental acuity C. Abdominal pain, pallor, and tachycardia D. Abdominal pain and vomiting

C. Abdominal pain, pallor, and tachycardia - These are all manifestations of splenic sequestrations

"Pustular" is a morphological classification of which skin lesion? A. Herpes simplex B. A wart C. Acne rosacea D. Impetigo

C. Acne rosacea

Which skin lesions are directly related to chronic sun exposure and photodamage? A. Skin tags B. Seborrheic keratoses C. Actinic keratoses D. Angiomas

C. Actinic keratoses - Also called senile or solar keratoses

Tanisha, a 24 year old African American mother of four young children, presents in the clinic today with varicella. She states that three of her children also have it and that her eruption started less than 24 hours ago. Which action may shorten the course of the disease in Tanisha? A. Calamine lotion B. Cool baths C. Acyclovir (Zovirax) D. Corticosteroids

C. Acyclovir (Zovirax)

Tanisha, a 24 year old African American mother of four young children, presents in the clinic today with varicella. She states that three of her children also have it and that her eruption started less than 24 hours ago. Which action may shorten the course of the disease in Tanisha? A. Calamine lotion B. Cool baths C. Acyclovir (Zovirax) D. Corticosteroids

C. Acyclovir (Zovirax) - If started within the first 24-48 hours, may shorten the course of varicella

Marsha states that a relative is having a carcinoembryonic antigen (CEA) test done to detect some type of cancer. She wants to know what kind. You tell her a CEA is performed to detect A. Adenocarcinoma of the prostate B. Medullary cancer of the thyroid C. Adenocarcinomas of the colon, lung, breast, ovary, stomach and pancreas D. Multiple myeloma

C. Adenocarcinomas of the colon, lung, breast, ovary, stomach and pancreas

What is the most significant reason why alcohol use is discouraged in persons with HIV infection or AIDS? A. Alcohol interferes with the pharmacokinetics of most AIDS drugs B. Filling up on empty calories of alcohol replaces the desire for food. C. Alcohol decreases the ability of persons to adhere to a prescribed medical regimen D. If clients become addicted to alcohol, when AIDS advances, they will become addicted to pain killers

C. Alcohol decreases the ability of persons to adhere to a prescribed medical regimen.

The primary reason for newborn screening for sickle cell disease is to A. Present the parents with the option for genetic screening in the future B. Test siblings if it is proved that the newborn has sickle cell disease C. Allow for the prevention of septicemia with prophylactic medication D. Prevent a sickle cell crisis

C. Allow for the prevention of septicemia with prophylactic medication - Prophylactic medication (penicillin) and prompt clinical intervention for infection and future crises.

Matthew, age 3, is rushed into the office with an upper respiratory obstruction as a result of foreign body aspiration. A partial obstruction is present. What is your next step? A. Place him facedown over your arm and give him five measured back blows B. Use the Heimlich maneuver C. Allow him to use his own cough reflex to extrude the foreign body D. Blindly probe his airway to dislodge the foreign body

C. Allow him to use his own cough reflex to extrude the foreign body

The immediate goal of myringotomy and tube placement in a child with recurrent episodes of totis media is to A. Prevent future infections B. Have an open access to the middle ear for irrigation and instillation of antibiotics C. Allow removal of suppurative or mucoid material D. Relieve pain

C. Allow removal of suppurative or mucoid material

The definitive test for sleep apnea is A. A Holter monitor B. A trial period of a continuous positive air pressure (CPAP) appliance C. An overnight polysomnogram D. An ear, nose, and throat (ENT) specialist confirming an abnormal uvula.

C. An overnight polysomnogram

Thin, spoon-shaped nails are usually seen in A. Trauma B. A fungal infection C. Anemia D. Psoriasis

C. Anemia

During the history portion of a respiratory assessment, it's particularly important to ask if the client takes which of the following drugs? A. Beta-2 agonists B. Calcium channel blockers C. Angiotensin-converting enzyme (ACE) inhibitors D. Birth control pills

C. Angiotensin-converting enzyme (ACE) inhibitors

The morphology of which lesion begins as an inflammatory papule that develops within several days into a painless, hemorrhagic, and necrotic abscess, eventually with a dense, black, necrotic eschar forming over the initial lesion? A. Furuncle-carbuncle B. Hidradenitis suppuritiva C. Anthrax D. Cellulitis

C. Anthrax

Which of the following laboratory studies is used to determine if a client has had hepatitis? A. Serum protein B. Protein electrophoresis C. Antibody testing D. Globulin levels

C. Antibody testing - Antibody titers evaluate antibody-mediated responses.

Mavis has persistent pruritus of the external auditory canal. External otitis and dermatological conditions such as seborrheic dermatitis and psoriasis have been ruled out. What can you advise her to do? A. Use a cotton tipped applicatory daily to remove all moisture and potential bacteria B. Wash daily with soap and water C. Apply mineral oil to counteract dryness D. Avoid topical corticosteroids

C. Apply mineral oil to counteract dryness

When should well individuals be screened for tuberculosis? A. Every year B. Every other year C. At age 1 year, then again at entry to preschool or kindergarten, and then at some point during adolescence D. Every 4 years

C. At age 1 year, then again at entry to preschool or kindergarten, and then at some point during adolescence.

What is the most important thing a woman can do to have youthful, attractive skin? A. Keep well hydrated B. Use sunscreen with an SPF of at least 45 C. Avoid smoking D. Use mild defatted or glycerine soaps

C. Avoid smoking - Smokers develop more wrinkles and have elastosis, decreased tissue perfusion and oxygenation, and an adverse exposure to free radicals on elastic tissue

What is the most effective treatment for urticaria? A. An oral antihistamine B. Dietary management C. Avoidance of the offending trigger D. A glucocorticosteroid

C. Avoidance of the offending trigger

A 44 year old non-smoker is diagnosed with pneumonia. He is otherwise healthy and does not need hospitalization at this time. Which antibiotic can be used for empirical treatment according to the 2007 Infectious Diseases Society of America/American Thoracic Society? A. Erythromycin B. Levofloxacin C. Azithromycin D. Amoxicillin

C. Azithromycin - These guidelines promote macrolide use or doxycycline for intitial treatment of uncomplicated pneumonia in outpatients who are otherwise healthy and have not had recent antibiotic exposure.

Marisa, who is pregnant, has just been given a diagnosis of tuberculosis. What do you do? A. Wait until Marisa delivers and then begin therapy immediately. B. Begin therapy with isoniazid (Nydrazid), rifampin (Rimactane), and pyrazinamide now. C. Begin therapy with isoniazid, rifampin, and ethambutol (Myambutol) now. D. Begin therapy with isoniazid now, wait to see how Marisa tolerates it, and then add rifampin, pyrazinamide, or ethambutol.

C. Begin therapy with isoniazid, rifampin, and ethambutol (Myambutol) now. - Treatment for tuberculosis in pregnant women is essential and should not be delayed.

Jim, age 49, comes to the office with a rapid-onset complete paralysis of one-half of his face. He is unable to raise his eyebrow, close his eye, whistle, or show his teeth. You suspect a lower motor neuron lesion resulting in cranial nerve VII paralysis. What is your working diagnosis? A. Cerebrovascular accident B. Trigeminal neuralgia C. Bell's Palsy D. Tic douloureux

C. Bell's Palsy

Which sympathomimetic agents are the drug of choice for asthma? A. Alpha agonists B. Beta-1 agonists C. Beta-2 agonists D. Alpha antagonists

C. Beta-2 agonists - Beta-2 agonists are more specific in their action to promote bronchodilation and are less likely to be associated with side effects.

Which sympathomimetic agents are the drug of choice for asthma? A. Alpha agonists B. Beta-1 agonists C. Beta-2 agonists D. Alpha antagonists

C. Beta-2 agonists - Beta-2 agonists are more specific in their action to promote bronchodilation and are less likely to be associated with side effects.

Antihistamines exhibit therapeutic effect by A. inactiviating circulating histamine B. peventing the production of histamine C. Blocking activity at histamine receptor sites D. acting as a procholinergic agent

C. Blocking activity at histamine receptor site

The American Cancer Society uses and ABCDE pneumonic to help patients and healthcare providers identify suspicious skin lesions. What does the "B" represent? A. Bleeding B. Black C. Border D. Benign

C. Border

The placement of a high dose of radioactive material directly into a malignant tumor and giving a lower dose to the normal tissues is referred to as A. Radiotherapy B. Teletherapy C. Brachytherapy D. Ionization therapy

C. Brachytherapy - Also referred to as internal, interstitial, or intracavitary radiation.

Your client has no signs or symptoms other than a fever and coarse crackles. What must you assess for further? A. Congestive heart failure B. Pneumonia C. Bronchitis D. Pulmonary Fibrosis

C. Bronchitis

Which cranial nerve (CN) is affected in sensorineural or perceptive hearing loss? A. CN II B. CN IV C. CN VIII D. CN XI

C. CN VIII

A 60 year old patient reports chronic cough and sputum production. He has a long history of exposure to second hand cigarette smoke from his wife. What diagnosis must be considered? A. Asthma B. Chronic bronchitis C. COPD D. Allergic cough

C. COPD - COPD must be considered in anyone with a history of chronic cough and sputum production.

Fecal occult blood testing (FOBT) is most effective in identifying A. Cancers in the right colon B. Polyps C. Cancers in the sigmoid colon D. Cancers in the transverse colon

C. Cancers in the sigmoid colon

Balanitis is associated with A. Diabetes B. Macular degeneration C. Candida infection of the penis D. Measles

C. Candida infection of the penis

Sara, age 29, states that she has painless, white, slightly raised patches in her mouth. They are probably caused by A. Herpes simplex B. Aphthous ulcers C. Candidiasis D. Oral cancer

C. Candidiasis

Which of the following statements is true regarding eye disease and persons with diabetes? A. Cataracts occur at the same rate in people with diabetes and those without diabetes B. Closed-angle glaucoma is more common in people with diabetes C. Cataracts occur at a younger age and progress more rapidly in people with diabetes D. Eye surgery must be approached with caution in people with diabetes because of poor healing

C. Cataracts occur at a younger age and progress more rapidly in people with diabetes

Which type of leukemia produces symptoms with an insidious onset including weakness, fatigue, massive lymphadenopathy, pruritic vesicular skin lesions, anemia, and thrombocytopenia? A. Acute lymphocytic leukemia B. Acute myelogenous leukemia C. Chronic lymphocytic leukemia D. Chronic myelogenous leukemia

C. Chronic lymphocytic leukemia

Jimmy is a 6 month old with newly diagnosed sickle cell disease. His mother brings him to the clinic for a well-baby visit. Which of the following should you do on this visit? A. Tell the parents that Jimmy will not be immunized because of his diagnosis B. Tell the parents that Jimmy should not go to day care C. Immunize Jimmy with diptheria, tetanus, and pertussis. Haemophilus influenzae type B (HIB); hepatitis B (HBV); and poliomyelitis vaccines D. Immunize Jimmy with measles, mumps, and rubella; HIB, HBV vaccines only

C. Immunize Jimmy with diptheria, tetanus, and pertussis. Haemophilus influenzae type B (HIB); hepatitis B (HBV); and poliomyelitis vaccines - Children with sickle cell disease should receive all the standard well-baby care, but in addition to the standard immunizations, they should receive the pneumococcal vaccine at age 2 years

Your client, Jackson, has decreased lymphocytes. You suspect A. Bacterial infection B. Viral infection C. Immunodeficiency D. Parasitic infection

C. Immunodeficiency - A decrease in lymphocytes would be most consistent with immunodeficiency disorders, long-term corticosteroid therapy, or debilitating diseases such as Hodgkin's lymphoma or lupus erythematosus - Lymphocytes are increased primarily in viral infections and only occasionally in bacterial infections. - Eosinophils are elevated in parasitic infections such as malaria, trichinosis, and ascariasis

Jan is having biological therapy for her pancreatic cancer. What kind of treatment is this? A. Surgery B. Radiation therapy C. Immunotherapy D. Chemotherapy

C. Immunotherapy - Uses the body's natural ability (immune system) to fight disease or to protect the body from adverse effects of treatment.

When you are assessing the corneal light reflex, an abnormal finding indicates A. Possible use of eye medications B. A neurological problem C. Improper alignment of the eyes D. Strabismus

C. Improper alignment of the eyes - It is noted when the reflections of the light are on different sites on the eyes

A patient with asthma uses one puff twice daily of an inhaled steroid and has an albuterol inhaler for PRN use. He requests a refill on his albuterol inhaler. His last prescription was filled 5 weeks ago. What action by the NP is appropriate? A. Refill the albuterol only B. Prescribe a longer acting bronchodilator, continue the steroid C. Increase the dose of the inhaled steroid, refill the albuterol D. Prescribe a long acting bronchodilator and increase the steroid

C. Increase the dose of the inhaled steroid, refill the albuterol - The patient is using his short acting bronchdilator excessively if he needs a refill of his inhaler in only 5 weeks.

Thalassemia is caused by A. Blood loss B. Impaired production of all blood-forming elements C. Increased destruction of red blood cells D. Autoimmune antibodies

C. Increased destruction of red blood cells - Caused by decreased synthesis of hemoglobin and malformation of red blood cells that increases their hemolysis.

Lorie, age 29, appears with the following signs: pale conjunctiva and nailbeds, tachycardia, heart murmur, cheilosis, stomatitis, splenomegaly, koilonychia, and glossitis. What do you suspect? A. Vitamin B12 deficiency B. Folate deficiency C. Iron-deficiency anemia D. Chronic fatigue syndrome

C. Iron-deficiency anemia - Classic signs of iron-deficiency anemia

In burn trauma, silver sulfadiazine (Silvadene), a sulfonamide, is the most commonly used topical agent. What is its mechanism of action? A. It is a synthetic antibiotic that appears to interfere with the metabolism of bacterial cells B. it is a bacteriostatic agent that inhibits a wide variety of gram-positive and gram-negative organisms by altering the microbial cell wall and membrane C. It is a bactericidal agent that acts on the cell membrane and cell wall of susceptible bacteria and binds to cellular DNA D. It is a protective covering that prevents light, air, and invading organisms from penetrating its surface

C. It is a bactericidal agent that acts on the cell membrane and cell wall of susceptible bacteria and binds to cellular DNA

In burn trauma, silver sulfadiazine (Silvadene), a sulfonamide, is the most commonly used topical agent. What is its mechanism of action? A. It is a synthetic antibiotic that appears to interfere with the metabolism of bacterial cells B. it is a bacteriostatic agent that inhibits a wide variety of gram-positive and gram-negative organisms by altering the microbial cell wall and membrane C. It is a bactericidal agent that acts on the cell membrane and cell wall of susceptible bacteria and binds to cellular DNA D. It is a protective covering that prevents light, air, and invading organisms from penetrating its surface

C. It is a bactericidal agent that acts on the cell membrane and cell wall of susceptible bacteria and binds to cellular DNA

Which of the following is true concerning Meniere disease? A. Neuroimaging helps locate the offending cochlear lesion B. Associated high-frequency hearing loss is common C. It is largely a diagnosis of exclusion D. Tinnitus is rarely reported

C. It is largely a diagnosis of exclusion

Clonazepam (Klonopin) is occasionally ordered for temporal mandibular joint (TMJ) disease. Which of the following statements applies to this medicine? A. It is ordered for inflammatory pain B. It is ordered for neuropathic pain C. It is ordered for a short course of therapy for 1-2 weeks only D. It is ordered for muscle relaxation

C. It is ordered for a short course of therapy for 1-2 weeks only

Which statement is true regarding primary spontaneous pneumothorax? A. It usually occurs after individuals have recently started an exercise program. B. It occurs more commonly in thin elderly men. C. It usually occurs in healthy individuals without preexisting lung disease. D. It frequently occurs in Marfan's syndrome.

C. It usually occurs in healthy individuals without preexisting lung disease.

Which of the following represents the most accurate patient advice when using permethrin (Elimite) for treating scabies: A. to avoid systemic absorption, the medication should be applied over the body and rinsed off within 1 hour B. The patient notices a marked reduction in pruritus within 48 hours of using the product C. Itch often persistes for a few weeks after successful treatment D. It is a second-line product in the treatment of scabies

C. Itch often persists for a few weeks after successful treatment

Which of the following individuals most likely will have a false-negative reaction to the Mantoux test? A. Marvin, age 59 B. Jane, who is on a short course of corticosteroid therapy for an acute exacerbation of asthma C. Jerry, who has lymphoid leukemia D. Mary, who recently was exposed to someone coughing

C. Jerry, who has lymphoid leukemia - Individuals predisposed to have a false-negative reaction to the Mantoux test include newborns and those older than age 60.

The Center for Disease Control and Prevention's definition of AIDS includes the presence of which of the following disorders, with or without laboratory evidence of HIV infection? A. Pneumonia in clients younger than age 60 B. Dementia in clients younger than age 60 C. Kaposi's sarcoma in clients younger than age 60 D. Primary brain lymphoma in clients older than age 60

C. Kaposi's sarcoma in clients younger than age 60 - Kaposi's sarcoma in a client younger than age 60 is considered conclusive evidence of AIDS according to CDC.

A patient will be taking oral Lamisil for fingernail fungus. The NP knows that: A. This will cure her infection 95% of the time B. A topical antifungal will work just as well if the nail matrix is involved C. Lamisil is a potent inhibitor of the CYP 3A4 enzymes D. Toenail fungus resolves faster than fingernail fungus after treatment

C. Lamisil is a potent inhibitor of the CYP 3A4 enzymes - Most oral antifungal agents inhibit the 3A4 enzymes in the cytochrome P450 system. This is precisely the reason they must be used with extreme caution (or not used) in patients who consume medications that need 3A4 enzymes for metabolism.

The trachea deviates toward the affected side in all of the following except A. Aortic aneurysm B. Unilateral thyroid lobe enlargement C. Large atelectasis D. Pneumothorax

C. Large atelectasis

Which drug class would not be used to treat COPD? A. Long-acting bronchodilator B. Long-acting anti-cholinergic C. Leukotriene blockers D. Systemic steroids

C. Leukotriene blockers

Which of the following secondary skin lesions usually results from chronic scratching or rubbing? A. Crusts B. Scales C. Lichenification D. Atrophy

C. Lichenification

A biopsy of a small, yellow-orange papulonodule on the eyelid will probably show A. Fragmented, calcified elastic tissue B. Mature sebaceous glands C. Lipid-laden cells D. Endothelial swelling and an infiltrate rich in plasma cells

C. Lipid-laden cells

A patient has seborrheic dermatitis. Which vehicle would be most appropriate to use in the hairline area to treat this? A. Ointment B. Cream C. Lotion D. Powder

C. Lotion - In hairy areas of the body, lotions are best accepted and particularly appreciated because they provide a cooling, drying effect. Foams are used in scalp area but this wasn't an option.

Multiple myeloma is a plasma cell malignancy in which the bone marrow is replaced, and there is bone destruction and paraprotein formation. Myeloma is a disease of older adults overall (median age at presentation, 65 years). Common presenting symptoms include A. Nausea and vomiting and chronic cough B. Fatigue and splenomegaly C. Lower back pain and hypercalcemia D. Nausea and vomiting and fatigue

C. Lower back pain and hypercalcemia - Bone pain is a common presenting symptom, most frequently manifested as low back pain or pain in the rib, and may present as a pathological fracture, especially in the femoral neck. Hypercalcemia is often present related to the leakage of calcium occurring from bone destruction.

You are examining Joseph, age 9 months, and note a palpable right supraclavicular node. You know that this finding is suspicious for A. Candidiasis B. Cryptococcosis C. Lymphoma of the mediastinum D. Abdominal malignancy

C. Lymphoma of the mediastinum - Palpable supraclavicular lymph nodes are not normal in infants, children or adults. A right sided palpable node is more commonly associated with lymphoma of the mediastinum, whereas a palpable left-sided node is more commonly associated with an abdominal malignancy

Which of the following statements about sarcoidosis is true? A. It commonly occurs in persons in their 50s B. It is more common in whites than in blacks C. Many organs may be involved, but the most involved organ is the lung D. It occurs more frequently in men than in women

C. Many organs may be involved, but the most involved organ is the lung.

Which of the following is the best treatment option for cellulitis when risk of infection with MR pathogen is considered low? A. dicloxacillin B. amoxicillin C.. metronidazole D. daptomycin

C. Metronidazole (Flagyl)

Your neighbor calls you because her son, age 9, fell on the sidewalk while playing outside and a tooth fell out. She wants to know what she should put the tooth in to transport it to the dentist. You tell her that the best solution to put it in is A. Salt water B. Saliva C. Milk D. Water

C. Milk

Which valve is most affected in rheumatic heart disease? A. Aortic B. Tricuspid C. Mitral D. Pulmonary

C. Mitral

Jill, age 49, has daily symptoms of asthma. She uses her inhaled short-acting beta-2 agonist daily. Her exacerbations affect her activities, and they occur at least twice weekly and may last for days. she is affected more than once weekly during the night with an exacerbation. Which category of asthma severity is Jill in? A. Mild intermittent B. Mild persistent C. Moderate persistent D. Severe persistent

C. Moderate persistent

Psoraisis may occur after months of using A. Vitamins B. Hormone replacement therapy C. NSAIDs D. Antihistamine nasal sprays

C. NSAIDs

Psoraisis may occur after months of using A. Vitamins B. Hormone replacement therapy C. NSAIDs D. Antihistamine nasal sprays

C. NSAIDs - Also after beta blockers, lithium, gold, antimalarials, ACE inhibitors and heavy alcohol use

When a neonate is initially protected against measles, mumps, and rubella because the mother is immune, this is an example of which type of immunity? A. Natural active B. Artificial active C. Natural passive D. Artificial passive

C. Natural passive

A sexual history of oral-genital contact in a client presenting with pharyngitis is significant when which of the following organisms is suspected? A. Escherichia coli B. Haemophilus influenzae C. Neisseria gonorrhoeae D. Streptococcus pneumoniae

C. Neisseria gonorrhoeae - Neisseria gonorrhoeae pharyngitis is a common sexually transmitted disease

A patient with asthma should use his rescue inhaler "minimally", according to the National Asthma Education and Prevention Program: Expert Panel Report 3, Guidelines for the Diagnosis and Management of Asthma from 2007. How often is "minimally"? A. Once monthly B. Once weekly C. Not more than twice weekly D. Not more than once daily

C. Not more than twice weekly

Julie's brother has chronic lymphatic leukemia. She overheard that he was in stage IV and asks what this means. According to the Rai classification system, stage IV is a stage A. At which the lymphocytes are greater than 10,000mm3 B. With an absolute lymphocytosis, in which the client may live 7-10 years or more C. Of thrombocytopenia, in which the life expectancy may be only 2 years D. Of anemia

C. Of thrombocytopenia, in which the life expectancy may be only 2 years

Darren, age 26, has AIDS and presents with a painful tongue covered with what looks like creamy-white, curdlike patches overlying erythematous mucosa. You are able to scrape off these "curds" with a tongue depressor, which assists you in making which of the following diagnoses? A. Leukoplakia B. Lichen planus C. Oral candidiasis D. Oral cancer

C. Oral candidiasis

Jessica, age 9 months, is brought into the clinic by her mother. She has a low-grade fever, stridor with agitation, some retractions, and a cough, but no drooling. You diagnose viral croup. Your next step would be to: A. Hospitalize Jessica B. Start antibiotic therapy C. Order supportive treatment D. Begin treatment with inhaled corticosteroids

C. Order supportive treatment - Mist therapy, oral hydration, and minimal handling are recommended. The presence of stridor at rest requires hospitalization.

The antibiotic of choice for the treatment of Streptococcus pneumoniae infection is A. Dicloxacillin B. Erythromycin C. Penicillin D. Ampicillin clavulanate

C. Penicillin

When teaching a mother who has a child with cystic fibrosis, you emphasize that the most important therapeutic approach to promote the child's pulmonary function is to: A. Continuously administer low-flow oxygen B. Administer bronchodilators on a regular basis C. Perform chest physiotherapy with postural drainage, percussion, and vibration D. Use maintenance antibiotic prophylactic therapy

C. Perform chest physiotherapy with postural drainage, percussion, and vibration.

Gouty pain in the great toe is A. Toe gout B. Hyperuricemia of the toe C. Podagra D. Tophus

C. Podagra

Which glomerular disease occurs 10-14 days after an acute illness (commonly streptococcal in children) and is characterized by tea-colored urine, mild to severe renal insufficiency, and edema? A. Henoch-Schonlein purpura glomerulonephritis B. Glomerulonephritis of systemic lupus erythematosus C. Postinfection glomerulonephritis D. Immunoglobulin A nephropathy

C. Postinfection glomerulonephritis

An area of deep abrasion, including a "road burn" on the left forearm is best treated by doing which of the following? A. icing the wound after irrigation B. Cleansing it several times a day and leaving it open to air C. Prescribing prophylactic antibiotic therapy D. Removing ground-in dirt using forceps

C. Prescribing prophylactic antibiotic therapy

In trying to differentiate between chronic bronchitis and emphysema, you know that chronic bronchitis A. Usually occurs after age 50 and has insidious progressive dyspnea B. Usually presents with a cough that is mild and with scant, clear sputum, if any C. Presents with adventitious breath sounds, wheezing and rhonchi, and a normal percussion note D. Results in increased total lung capacity with a markedly increased residual volume

C. Presents with adventitious breath sounds, wheezing and rhonchi, and a normal percussion note - Chronic bronchitis usually occurs after age 35, with recurrent respiratory infections. There is usually a persistent, productive cough of copious mucopurulent sputum and pulmonary function studies show normal or decreased total lung capacity with a moderately increased residual volume.

Jim, age 59, presents with recurrent, sharply circumscribed red papules and plaques with a powdery white scale on the extensor aspect of his elbows and knees. What do you suspect? A. Actinic keratosis B. Eczema C. Psoriasis D. Seborrheic dermatitis

C. Psoriasis

Which lesion results in scales or shedding flakes of greasy, keratinized skin tissue? A. Eczema B. Impetigo C. Psoriasis D. Herpes

C. Psoriasis

Which lesion results in scales or shedding flakes of greasy, keratinized skin tissue? A. Eczema B. Impetigo C. Psoriasis D. Herpes

C. Psoriasis - The color may be white, gray, or silver, and the texture may vary from fine to thick

Laura, age 36, has an acute onset of dyspnea. Associated symptoms include chest pain, faintness, tachypnea, peripheral cyanosis, low blood pressure, crackles and some wheezes. Her history reveals that she is taking birth control pills and that she smokes. What do you suspect? A. Asthma B. Bronchitis C. Pulmonary emboli D. Pneumothorax

C. Pulmonary emboli

Which of the following is essential to diagnose COPD? A. Chest radiograph B. CT scan of the chest C. Pulmonary function test D. Arterial blood gas

C. Pulmonary function

Evidence-based practice has shown that clients with COPD will benefit the most from which of the following single modalities? A. Nutritional supplementation B. Routine use of inspiratory muscle training C. Pulmonary rehabilitation D. Psychosocial interventions

C. Pulmonary rehabilitation

Which of the following lesions never blanches when pressure is applied? A. Spider angioma B. Spider vein C. Purpura D. Cherry angioma

C. Purpura

Which of the following statements is true regarding the recurrence of a spontaneous pneumothorax? A. A primary spontaneous pneumothorax is more likely to recur than a secondary one B. A secondary spontaneous pneumothorax is more likely to recur than a primary one. C. Recurrence rates for both primary and secondary spontaneous pneumothorax are similar D. Spontaneous pneumothorax rarely recurs.

C. Recurrence rates for both primary and secondary spontaneous pneumothorax are similar

A patient who is otherwise healthy comes in for evaluation. He states that he woke up this morning and cannot hear out of his left ear. He feels well otherwise. His ENT and neurological evaluation are normal. How should the nurse practitioner proceed? A. Prescribe an antihistamine and nasal decongestant B. Send the patient to an audiologist for hearing evaluation C. Refer the patient to ENT for evaluation D. Initiate steroids and consider referral in a week

C. Refer the patient to ENT for evaluation - Sudden hearing loss has a number of etiologies of central origin. The differential diagnosis should include acoustic neuroma, vascular insufficiency, carotid artery occlusion, and other intracranial tumors.

A patient presents with a painful blistering thermal burn involving the first, second and third digits of his right hand. The most appropriate care is: A. apply an anesthetic cream to the area and open the blisters B. apply siler sulfadiazine cream to the area followed by a bulky dressing C. refer the patient to burn specialty care D. wrap the burn loosely with a nonadherent dressing and prescribe an analgesic agent

C. Refer the patient to burn specialty care

Which of the following indicates that Jim, a 32 year old client with AIDS, has oropharyngeal candidiasis? A. Small vesicles B. Fissured, white, thickened plaques C. Removable white plaques D. Flat-topped papules with thin, bluish-white spiderweb lines

C. Removable white plaques

Greg, age 72, is brought to the office by his son, who states that his father has been unable to see clearly since last night. Greg reports that his vision is "like looking through a veil." He also sees floaters and flashing lights but is not having eye pain. What do you suspect? A. Cataracts B. Glaucoma C. Retinal detachment D. Iritis

C. Retinal detachment

When you teach clients about using steroid inhalers for asthma or COPD, what information is essential? A. Keep the inhaler in the refrigerator B. Do not use another inhaler for 10 minutes after the steroid inhaler C. Rinse your mouth after use D. Be careful not to shake the container prior to using

C. Rinse your mouth after use

Pastia lines are present in which disease? A. Toxic shock syndrome B. Rocky Mountain spotted fever C. Scarlet fever D. Meningococcemia

C. Scarlet Fever - In scarlet fever, there is diffuse erythema with a sandpaper texture and gooseflesh appearance, with accentuation of erythema in the flexual creases referred to as Pastia lines.

Which of the following is a secondary skin lesion? A. Acne nodule B. Neoplasm C. Seborrheic dermatitis D. Herpes simplex

C. Seborrheic dermatitis - Primary lesions are original lesions arising from previously normal skin. Secondary lesions can originate from primary lesions. Seborrheic dermatitis is a scale and the only secondary lesion listed.

Which of the following is a characteristic of H. influenzae? A. Newer macrolides are ineffective against the organism B. Its antimicrobial resistance results from altered protein-binding sites within the wall of the bacteria C. Some isolates exhibit antimicrobial resistance via production of beta-lactamase D. This is a gram-positive organism

C. Some isolates exhibit antimicrobial resistance via production of beta-lactamase

What is the gold standard for the diagnosis of asthma? A. Validated quality of life questionnaire B. Client's perception of "clogged" airways C. Spirometry D. Bronchoscopy

C. Spirometry - Gold standard for diagnosis as well as for periodic monitoring of the condition.

What is the gold standard for the diagnosis of asthma? A. Validated quality of life questionnaire B. Client's perception of "clogged" airways C. Spirometry D. Bronchoscopy

C. Spirometry - Gold standard for diagnosis as well as for periodic monitoring of the condition.

A skin lesion fluoresces under a Wood's lamp. What microscopic finding is consistent with this? A. Clue cells B. Herpes simplex C. Spores D. Leukocytes

C. Spores

Sophie brings in her husband, Nathan, age 72, who is in a wheelchair. On his sacral area he has a deep crater with full-thickness skin loss involving necrosis of subcutaneous tissue that extends down to the underlying fascia. Which pressure ulcer stage is this? A. Stage I B. Stage II C. Stage III D. Stage IV

C. Stage III - Stage I is nonblanchable erythema of intact skin - Stage II is partial-thickness skin loss involving the epidermis and/or dermis - Stage IV involves full thickness skin loss with extensive destruction; tissue necrosis; or damage to muscle, bone, or supporting structures

Steve, age 29, has a carbuncle on his neck. After an incision and drainage (I&D), an antibiotic is ordered. What is the most common organism involved? A. Streptococcus B. Moraxella catarrhalis C. Staphylococcus aureus D. Klebsiella

C. Staphylococcus aureus

A client with HIV infection has a CD4 count of 305 and an HIV RNA level of 13,549. The client is asymptomatic. What is your course of action? A. Negotiate with your client a time to start therapy B. Recheck the laboratory results in 1 month. If the counts remain like this, start treatment. C. Start therapy now because the client's CD4 count is less than 500 and the HIV RNA level is greater than 10,000. D. Wait to start therapy until the client becomes asymptomatic

C. Start therapy now because the client's CD4 count is less than 500 and the HIV RNA level is greater than 10,000. - Regardless of being asymptomatic, CDC recommends starting treatment in a patient with CD4 less than 500 and HIV RNA levels greater than 10,000

Johnny, age 12, just started taking amoxicillin for otitis media. His mother said that he woke up this morning with a rash on his trunk. What is your first action? A. Prescribe systemic antibiotics B. Prescribe a short course of systemic steroids C. Stop the amoxicillin D. Continue the drug; this reaction on the first day is normal

C. Stop the amoxicillin

Clients with allergic conjunctivitis have which type of discharge? A. Purulent B. Serous or clear C. Stringy and white D. Profuse mucoid or mucopurulent

C. Stringy and white

An infant who has periodic breathing with persistent or prolonged apnea (greater than 20 seconds) may have an increased risk of A. Pneumonia B. Left-sided congestive heart failure C. Sudden infant death syndrome (SIDS) D. Anemia

C. Sudden infant death syndrome (SIDS)

How does pregnancy affect asthma? A. During pregnancy, asthma usually improves B. During pregnancy, asthma usually worsens C. Symptoms in about one-third of pregnant women with asthma improve, about one-third are unchanged, and about one-third worsen D. Symptoms in about one-half of pregnant women improve; those of the other half worsen

C. Symptoms in about one-third of pregnant women with asthma improve, about one-third are unchanged, and about one-third worsen

A darkfield microscope examination is used to diagnose A. Scabies B. Leprosy C. Syphilis D. Candida infections

C. Syphilis

Debbie, age 29, has a high fever and red, warm, sharply marginated plaques on the right side of her face that are indurated and painful. You diagnose erysipelas. What treatment do you begin? A. Systemic steroids B. Topical steroids C. Systemic antibiotics D. NSAIDs

C. Systemic antibiotics

Debbie, age 29, has a high fever and red, warm, sharply marginated plaques on the right side of her face that are indurated and painful. You diagnose erysipelas. What treatment do you begin? A. Systemic steroids B. Topical steroids C. Systemic antibiotics D. NSAIDs

C. Systemic antibiotics - Erysipelas is caused by Streptococcus hemolyticus and must be treated with appropriate antibiotics

Margaret, age 32, comes into the clinic. She has painful joint and a distinctive rash in a butterfly distribution on her face. The rash has red papules and plaques with a fine scale. What do you suspect? A. Lymphocytoma cutis B. Relapsing polychondritis C. Systemic lupus erythematosus D. None of the above

C. Systemic lupus erythematosus

Which of the following situations might precipitate a sickle cell crisis in an infant? A. Taking the infant to visit a relative B. Hepatitis B immunization C. Taking the infant to a home Miami Dolphins football game D. Having the infant sleep on its back

C. Taking the infant to a home Miami Dolphins football game - Certain precautions must be taken for infants with sickle cell disease to prevent vaso-occlusive crisis. Any activity or situation that would cause dehydration should be avoided. An example is sitting in a stroller in the heat for any length of time.

Sam, age 72, has dry eye with a triad presentation of buring, itching, and a foreign body sensation in the eye. You know this as keratoconjunctivitis sicca (KCS). This symptom is frequently associated with the diagnosis of A. Reuter's syndrome B. Sjorgens syndrome C. Temporal arteritis D. glaucoma

C. Temporal arteritis

The following figure shows a method of assessing for digital clubbing called A. Phalangeal depth ratio B. Hyponychial angle C. The Schamroth sign D. The "diamond" test

C. The Schamroth sign - The dorsal surfaces of the terminal phalanges of similar fingers are placed together. With clubbing, the normal diamond shaped window at the bases of the nailbeds disappears and a prominent distal angle forms between the end of the nails.

Which assessment test is a gross measurement of peripheral vision? A. The cover test B. The corneal light reflex test C. The confrontation test D. The Snellen eye chart test

C. The confrontation test - It compares the client's peripheral vision with the practitioner's, assuming that the practitioner has normal peripheral vision.

Which manifestation of the buccal mucosa is present in a client with mumps? A. Pink, smooth, moist appearance with some patchy hyperpigmentation B. Dappled brown patches C. The orifice of Stensen's duct appearing red D. Koplik's spots

C. The orifice of Stensen's duct appearing red

Jane is the 26 year old Asian mother of Alysia, age 2 months. She is concerned about the large blue spot covering her infant's entire right lower leg. Jane tells you that Alysia was born with the spot. You tell her that A. When the infant reaches adult height, the macule can be surgically removed B. She should take the infant immediately to a plastic surgeon because this is a rare cancerous lesion C. This is a mongolian spot. It is common in Asians and blacks and no treatment is necessary because it will fade with age D. She should always keep the spot covered because sunlight will aggravate it

C. This is a mongolian spot. It is common in Asians and blacks and no treatment is necessary because it will fade with age

A 40 year old patient in good health is found to have a 0.5cm white plaque on the oral mucosa of the inner cheek. Her tongue and teeth are within normal limits. There is no pain associated with this. How should the nurse practitioner proceed? A. This is a benign oral lesion. Monitor only B. This is probably due to mechanical trauma. Monitor C. This should be biopsied D. This should be referred to a dentist

C. This should be biopsied - A differential diagnosis must be established and part of the differential is oral leukoplakia, a premalignant lesion.

John, age 46, has AIDS. He comes to the office with white lesions in his oropharynx. What do you suspect? A. Kaposi's sarcoma B. Herpes simplex virus C. Thrush or oral hairy leukoplakia D. Gingivitis

C. Thrush or oral hairy leukoplakia

Buddy, age 12, presents with annular lesions with a scaly border and central clearing on his trunk. What do you suspect? A. Psoriasis B. Erythema multiforme C. Tinea corporis D. Syphilis

C. Tinea corporis

Buddy, age 12, presents with annular lesions with a scaly border and central clearing on his trunk. What do you suspect? A. Psoriasis B. Erythema multiforme C. Tinea corporis D. Syphilis

C. Tinea corporis - Psoriasis, erythema multiforme, tinea corporis, and syphilis all have lesions with annular configurations

Why is ultraviolet light therapy used to treat psoriasis? A. To dry the lesions B. To kill the bacteria C. To decrease the growth rate of epidermal cells D. To kill the fungi

C. To decrease the growth rate of epidermal cells

Why is ultraviolet light therapy used to treat psoriasis? A. To dry the lesions B. To kill the bacteria C. To decrease the growth rate of epidermal cells D. To kill the fungi

C. To decrease the growth rate of epidermal cells - Treatments are given daily and last for only seconds

What do you include in your teaching about Spiriva (tiotropium) when you initially prescribe it for your client with COPD? A. Use it every time you use your beta-2 agonist B. Stop taking all your other COPD medications C. Use this once per day D. Stop taking Spiriva if you develop the adverse effect of dry mouth

C. Use this once per day - Spiriva is a once-daily, long-acting anticholinergic. It results in improved lung function studies, and reduction in the use of rescue medication and in COPD exacerbations.

Mark has a diagnosis of bullous myringitis. What might you expect to see during an otoscopic examination? A. Bulging of the tympanic membrane (TM) B. Serous amber fluid and air behind the TM C. Vesicles on the TM D. Perforation of the TM

C. Vesicles on the TM

The most common agent identified in patients who have acute bronchitis is: A. H. influenza B. S. pneumoniae C. Viral agents D. Pertussis

C. Viral agents

A patient has been diagnosed with acute rhinosinusitis. What is the most likely etiology? A. Strep organisms B. Staph organisms C. Viral pathogens D. Mycoplasma

C. Viral pathogens - 2% or fewer cases of acute rhinosinusitis are due to bacteria.

Mr. Smith has smoked for 45 years. He has a history of hypertension, gout, and benign prostatic hyperplasia. Which of the following medications could be given without risk of worsening any of his disease states? A. Codeine B. Nighttime oxygen C. Vitamin B12 D. Propanolol

C. Vitamin B12 - After smoking for 45 years, Mr. Smith can be assumed to have COPD. Codeine and nighttime oxygen potentially suppress the respiratory drive. Propanolol, a beta-blocker, can block the beta receptors (theoretically) in the lungs and produce respiratory difficulty in the patient.

You are teaching Harvey about the warts on his hands. What is included in your teaching/ A. Treatment is usually effective and most warts will not recur afterward B. Because warts have roots, it is difficult to remove them surgically C. Warts are caused by the human papillomavirus D. Shaving the wart may prevent its recurrence

C. Warts are caused by the human papillomavirus

Sharon, age 29, is pregnant for the first time. She complains of nasal stuffiness and occasional epistaxis. What do you do? A. Order lab tests, such as a CBC with differential, hemoglobin and hematocrit B. Prescribe an antihistamine C. You do nothing except for client teaching D. Refer the client to an ear, nose and throat specialist

C. You do nothing except for client teaching - Can occur with pregnancy because of increased vascularization in the upper respiratory tract

Which treatment would you order for anogenital pruritus? A. Suppositories for pain B. Antifungal cream for itching C. A high-fiber diet for constipation D. Zinc oxide ointment

C. a high fiber diet for constipation

One of the mechanism of action of a topical corticosteroid preparation is as: A. an antimitotic B. an exfoliant C. a vasoconstrictor D. a humectant

C. a vasoconstrictor -potency based on vasoconstrictor ability

Which skin lesions are directly related to chronic sun exposure and photodamage? A. Skin tags B. Seborrheic keratoses C. Actinic keratoses D. Angiomas

C. actinic keratosis

A 19-year-old woman presents with a complaint of bilaterally itchy, red eyes with tearing that occurs intermittently throughout the year and is often accompanied by a ropelike eye discharge and clear nasal discharge. This is most consistent with conjunctival inflammation caused by a(n): A. bacterium B. virus C. allergen D. injury

C. allergen

In examining a child with possible rheumatic fever, you may note A. a murmur consistent with valvular insufficiency B. subcutaneous nodules on the extensor surfaces of the lower extremities C. an erythematous rash over the trunk and proximal part of the limbs D. confusion caused by low fevers

C. an erythematous rash over the trunk and proximal part of the limbs

You inspect the oral cavity of a 69-year-old man who has a 100-pack year cigarettes smoking history. You find a lesion suspicious for malignancy and describe it as: A. raised, red, and painful B. a denuded patch with a removable white coating C. an ulcerated lesion with indurated margins D. a vesicular-form lesion with macerated margins

C. an ulcerated lesion with indurated margins

Spoon-shaped, thin nails are often seen in clients with A. psoriasis B. a fungal infection C. anemia D. eczema

C. anemia

A 58-year-old woman presents with a sudden left-sided headache that is most painful in her left eye. Her vision is blurred, and the left pupil is slightly dilated and poorly reactive. The left conjunctiva is markedly injected, and the eyeball is firm. Vision screen with the Snellen chart is 20/30 OD and 20/90 OS. The most likely diagnosis is: A. unilateral herpetic conjunctivitis B. open-angle glaucoma C. angle-closure glaucoma D. anterior uveitis

C. angle-closure glaucoma

Alexandra, age 48, has a bad complexion and has been treated for acne unsuccessfully. You suspect rosacea rather than acne because rosacea A. appears as blackheads and whiteheads B. causes the skin to become oily C. appears primarily around the nose and cheeks D. appears red and waxy

C. appears primarily around the nose and cheeks

The most important aspect of skin care for individuals with atopic dermatitis is: A. frequent bathing with antibacterial soap B. consistent use of medium-potency to high-potency topical steroids C. appication of lubicrants D. treatment of dermatophytes

C. application of lubicrants

A client has a diagnosis of bronchiectasis. When he asks about what caused this, the nurse practitioner tells him that the structural changes in the bronchi are usually associated with A. chronic bronchitis B. lung tumors C. bacterial infections D. congenital defects

C. bacterial infections

Lymphedema may be differentiated from venous edema by A. being a soft pitting edema with skin of normal texture B. affecting the foot but not the toes C. being a firm edema that pits poorly, with thickened skin D. showing an increase in the superficial venous pattern

C. being a firm edema that pits poorly, with thickened skin

First line therapy for acne vulgaris with closed comedones include: A. oral antibiotics B. isotretinoin C. benzoyl peroxide D. hydrocortisone cream

C. benzoyl peroide

One of the most serious complications of giant cell arteritis is: A. hemiparesis B. arthritis C. blindness D. uveitis

C. blindness

Oral decongestant use should be discouraged in patients with: A. allergic rhinitis B. migraine headache C. cardiovascular disease D. chronic bronchitis

C. cardiovascular disease

Mrs. Garvey brings her 10-year-old son, Steve, to your primary care office. He has a 3-in. laceration on his leg that occured over 24 hours before this visit. He did not tell his mother about this injury when it happened and the bleeding has stopped. No special efforts had been taken to cleanse the wound until the evening, when his mother discovered it. The wound edges are in close approximation. You should A. cleanse, suture, and dress the wound and then have Steve return in 5 days for suture removal B. cleanse the wound and apply Steri-Strips C. cleanse and dres the wound, then have the client return in 3-5 days D. irrigate and cleanse the wound with antiseptic agents

C. cleanse and dres the wound, then have the client return in 3-5 days

Sara, age 57, comes in with a complaint of pain and swelling in her right lower leg. The pain is worse if she tries to walk or stand on the leg. She smokes a half pack of cigarettes a day and takes a hormone replacement pill every night. She states that she returned from Tokyo 3 days ago and still feels jetlagged. Her vital signs are normal. Her physical examination reveals a warm and slightly reddened right lower leg with marked edema extending from the knee to the foot; intact popliteal, dorsalis pedis, and posterior tibial pulses; and an unremarkable Homan's sign. You suspect A. edema from increased salt intake from the foods she had been eating while in Tokyo B. acute arterial occlussion C. deep venous thrombosis D. dependent edema from sitting for long periods of time during her flight home

C. deep venous thrombosis

When performing an ophthalmoscopic examination, you note retinal hemorrhages and narrowing, obliteration, dilation, and tortuousness of the retinal vessels. You diagnose this as A. macular degeneration B. hypertensive retinopathy C. diabetic retinopathy D. a cataract

C. diabetic retinopathy

The most common clinical manifestation of heart failure is A. tachycardia B. syncope C. dyspnea D. peripheral edema

C. dyspnea

First-line intervention for anterior epistaxis includes: A. nasal packing B. application of topical thrombin C. firm pressure to the area superior to the nasal alar cartilage D. chemical

C. firm pressure to the area superior to the nasal alar cartilage

Appropriate oral antimicrobial therapy for otitis externa with an accompanying facial cellulitis suitable for outpatient therapy includes a course of a: A. macrolide B. cephalosporin C. fluoroquinolone D. penicillin

C. fluoroquinolone

The most common causative organism in cellulitis are: A. Escherichia coli and H. influenza B. Bacteroids species and other anaerobes C. group A beta-hemolytic streptococci and S. aureus D. pathogenic viruses

C. group A beta-hemolytic streptococci and S. aureus

Your client Mr. Lane, who suffers from chronic obstructive pulmonary disease (COPD), calls to say that, while visiting his daughter, he was exposed to influenza. He had received a flu shot in your office less than a week ago. He is calling because he is concerned that it is not working as yet. You tell him that A. the flu shot will be sufficient to protect him B. because the flu shot will not be fully effective for several weeks, he should wash his hands frequently and get adequate rest C. he should start amantadine (symmetrel) because it has been less than 48 hours since exposure to the infected person D. he should start amantadine (symmetrel) immediately because the time since exposure has been over 72 hours

C. he should start amantadine (symmetrel) because it has been less than 48 hours since exposure to the infected person

Clinical symptoms of congestive heart failure noted on physical examination may include A. collapsed neck veins B. splenomegaly C. hepatomegaly and tenderness D. hyperactive bowel sounds and diarrhea

C. hepatomegaly and tenderness

When palpating lymph nodes, it is essential to also palpate the supraclavicular nodes located A. along the chest wall, high in the axilla B. superficially on the medial side of the elbow C. hidden behind the sternocleidomastoid muscle's clavicular head D. directly in the axillary region

C. hidden behind the sternocleidomastoid muscle's clavicular head

In the treatment of acne vulgaris, which lesions respond best to topical antibiotic therapy: A. open comedones B. cysts C. inflammatory lesions D superficial lesions

C. inflammatory lesions

Food sources rich in iron include A. potatoes; bananas; and green, leafy vegetables B. enriched grain cereals, cabbage, and sweet potatoes C. liver, red meats, prunes, apples, and raisins D. enriched grain cereals, strawberries, watermelon, and honeydew melons

C. liver, red meats, prunes, apples, and raisins

Anterior epistaxis is usually caused by: A. hypertension B. bleeding disorders C. localized nasal mucosa trauma D. a foreign body

C. localized nasal mucosa trauma

Cromolyn's mechanism of action is as a/an: A. anti-immunoglobulin E antibody B. vasoconstrictor C. mast cell stabilizer D. leukotriene modifier

C. mast cell stabilizer

In diagnosing onychomycosis, the NP considers that: A. nails often have a single midline groove B. pitting is often seen C. microscopic examination reveals hyphae D. Beau lines are present

C. microscopic examination reveals hyphae

Clinical presentation of peritonsillar abscess includes: A. occipital lymphadenopathy B. congested cough C. muffled "hot potato" voice D. abdominal pain

C. muffled "hot potato" voice

Treatment of a cat bite wound should include prescribing: A. oral erythromycin B. topical bacitracin C. oral amoxicillin clavulanate D. parenteral rifampin

C. oral amoxicillin clavulanate (Augmentin) -80% get infected

The treatment of choice for an adult with a diagnosis of group A beta-hemolytic streptococcal pharyngitis is A. ofloxacin (Floxin) 500 mg PO bid for 5 days B sulfamethoxazole-trimethoprim (Bactrim) DS 1 tab PO bid for 10 days C. penicillin V (PenVee K) 500 mg PO bid for 10 days D. cefaclor (ceclor) 500 mg PO bid for 7 days

C. penicillin V (PenVee K) 500 mg PO bid for 10 days

The most common pathogen for bronchiolitis in children younger than age 2 is A. parainfluenza B. influenza C. respiratory syncytial virus D. adenovirus

C. respiratory syncytial virus

Mary, age 69, has COPD. Her oxygen saturation is less than 85%. She is to start on oxygen therapy to relieve her symptoms. How often must she be on oxygen therapy to actually improve her oxygen saturation? A. On an as-needed basis B. Continuously C. 6-12 hours per day D. 18 hours per day

D. 18 hours per day - To decrease mortality in clients with COPD whose oxygen saturations are less than 85%, oxygen must be used at least 18 hours per day to be of more than symptomatic benefit.

The incubation period for M. pneumoniae is usually: A. less than 1 week B. 1 week C. 2 weeks D. 3 weeks

D. 3 weeks

Skip, age 4, is brought in to the office by his mother. His symptoms are pallor, fatigue, bleeding, fever, bone pain, adenopathy, arthralgias, and hepatosplenomegaly. You refer him to a specialist to perform to confirm a diagnosis? A. An enzyme-linked immunosorbent assay B. A monospot test C. A prothrombin time, partial thromboplastin time, bleeding time, complete blood count, and peripheral smear D. A bone marrow spear

D. A bone marrow smear

Marian, age 79, is at a higher risk than a middle-aged client for developing an eye infection because of which age-related change? A. Increased eyestrain B. Loss of subcutaneous tissue C. Change in pupil size D. A decrease in tear production

D. A decrease in tear production

Macrocytic normochromic anemias are caused by A. Acute blood loss B. An infection or tumor C. A nutritional deficiency of iron D. A deficiency of folic acid

D. A deficiency of folic acid - Cell size is large and irregular.

Mycoplasma pneumoniae is: A. A diagnosis of exclusion B. A typical respiratory pathogen C. Only identifiable on chest x-ray D. A disease with extrapulmonary manifestations

D. A disease with extrapulmonary manifestations - Mycoplasma is an atypical pathogen and produces atypical pneumonia. It can be difficult to diagnose because symptoms can be varied and involve multiple body systems (extrapulmonary manifestations). Infection with Mycoplasma may present with a normal white blood cell count, maculopapular rash, GI symptoms, tender joints and aches, and though rare, cardiac rhythm disturbances. Respiratory symptoms may not be pronounced.

Sue, age 53, comes to your office for a routine examination. In passing, she mentions that she has Sjorgen's syndrome. What do you think is her major complaint related to this syndrome? A. Fatigue B. Diarrhea C. Constipation D. A dry, gritty sensation in her eyes

D. A dry, gritty sensation in her eyes

A patient diagnosed with Strep throat received a prescription for amoxicillin. She has not improved in 48 hours. What course of action is acceptable? A. The patient should wait another 24 hours for improvement B. The antibiotic should be changed to a first generation cephalosporin C. A macrolide antibiotic should be prescribed D. A penicillin or cephalosporin with beta lactamase coverage should be considered

D. A penicillin or cephalosporin with beta lactamase coverage should be considered.

Clubbing is defined as A. Elongation of the toes B. Broadening of each thumb C. A birth deformity of the feet D. A thickening and broadening of the ends of the fingers

D. A thickening and broadening of the ends of the fingers

Treatment for Stage I pressure ulcer may include A. An enzymatic preparation B. Systemic antibiotics C. Surgical treatment with muscle flaps D. A transparent, semipermeable membrane dressing

D. A transparent, semipermeable membrane dressing

Which skin lesion is morphologically classified as pustular? A. A wart B. Impetigo C. Herpes simplex D. Acne rosacea

D. Acne rosacea

Which skin lesion is morphologically classified as pustular? A. A wart B. Impetigo C. Herpes simplex D. Acne rosacea

D. Acne rosacea

Under which of the following circumstances is the reticulocyte count elevated? A. Aplastic anemia B. Iron-deficiency anemias C. Poisonings D. Acute blood loss

D. Acute blood loss - The reticulocyte count indicates the percentage of newly maturing red blood cells released into the circulating blood from the bone marrow.

Stridor can be heard on auscultation when a client has A. Atelectasis B. Asthma C. Diaphragmatic hernia D. Acute epiglottitis

D. Acute epiglottitis

Which ethnic group has the highest overall cancer incidence rate? A. Native Americans B. Asian and Pacific Islanders C. Hispanics D. African Americans

D. African Americans

A Gram stain of which lesion reveals large, square ended, gram positive rods that grow easily on blood agar? A. Dermatophyte infection B. Tuberculosis (scrofuloderma) C. Sarcoidosis D. Anthrax

D. Anthrax

A cough caused by a postnasal drip related to sinusitis is more prevalent at what time of day? A. Continuously throughout the day B. In the early morning C. In the afternoon and evening D. At night

D. At night - Some conditions have characteristic timing of a cough. A cough caused by a postnasal drip related to sinusitis is more prevalent at night. A cough associated with an acute illness, such as respiratory infection, is continuous throughout the day. A cough early in the morning is usually caused by chronic bronchial inflammation from habitual smoking.

Claude, age 78, is being treated with timolol maleate (Timoptic) drops for his chronic open-angle glaucoma. While performing a new client history and physical, you note that he is taking other medications. Which medication would you be most concerned about? A. Apsirin therapy as prophylaxis for heart attack B. Ranitidine (Zantac) for gastroesophageal reflux disease C. Alprazolam (Xanax), an anxiolytic for anxiety D. Atenolol (Tenormin), a beta blocker for high blood pressure

D. Atenolol (Tenormin), a beta blocker for high blood pressure

Which type of pharyngitis do you suspect when a client has a sore throat with dysphagia and thin, white nonvesicular diffues exudative ulcers on the mucosa? A. Allergic pharyngitis B. Streptococcal pharyngitis C. Mononucleosis D. Candida infection

D. Candida infection

Susan, age 46, states that she has a slowly developing, painless, hard mass on her right eye. Physical examination with eversion of the eyelid reveals a red, elevated mass that is quiet large on the meibomian gland pressing against the eye, causing nystagmus. What is your diagnosis? A. Hordeolum B. Blepharitis C. Sebaceous cell carcinoma D. Chalazion

D. Chalazion

What is effective in the treatment of pneumonia, atelectasis, and cystic fibrosis? A. Deep breathing B. Oxygen C. Inhalers D. Chest physiotherapy (CPT)

D. Chest physiotherapy - The technique uses percussion and postural drainage along with coughing and deep breathing exercises.

Which of the following drugs causes a cough by inducing mucus production (bronchorrhea)? A. Tobacco and/or marijuana B. Beta-adrenergic blockers C. Aspirin and NSAIDs D. Cholinesterase inhibitors

D. Cholinesterase inhibitors

Which of the following is the most frequent contributor to the incidence of carcinoma of the lung? A. Chronic pneumonia B. Exposure to materials such as asbestos, uranium, and radon C. Chronic interstitial lung diseases D. Cigarette smoking

D. Cigarette smoking - 85% of all lung carcinomas are secondary to cigarette use.

A client complain of frequent bouts of severe, intense, disabling left sided facial pain accompanied by excessive left eye lacrimation (tearing) and worsening anxiety. The pain wakes him at night and he has even contemplated harming himself during these episodes due to the intensity and unrelenting nature of the pain. What kind of headache is he describing? A. Classic migraine B. Tension headache C. Sinus headache D. Cluster headache

D. Cluster headache

James, age 12, just moved here from Texas. He presents with a headache, cough, fever, rash on the legs and arms, myalgias, and dysuria. His white blood cell count is 12.9 with 8% bands and 7%-10% eosinophils. Electrolyte levels are normal. Blood cultures are negative. Sputum is not available. A Mantoux skin test so far is negative. What do you suspect? A. Pulmonary tuberculosis B. Lymphoma C. Asthma D. Coccidioidomycosis

D. Coccidioidomycosis - The leading mycotic (fungal) infection in the southwestern United States, with an annual morbidity estimated at 35,000 cases.

What is a noninvasive method of treating skin cancer (other than melanomas) that uses liquid nitrogen? A. Mohs' micrographic surgery B. Curettage and electrodisection C. Radiation therapy D. Cryosurgery

D. Cryosurgery

Which of the following differential diagnoses should be considered with a chronic cough in children ages 1-5 years after the more common causes have been ruled out? A. Allergic rhinitis B. Chronic sinusitis C. Enlarged adenoids D. Cystic fibrosis

D. Cystic fibrosis

A loss of DNA control over differentiation that occurs in response to adverse conditions is referred to as A. Hyperplasia B. Metaplasia C. Anaplasia D. Dysplasia

D. Dysplasia - Dysplastic cells show an abnormal degree of variation in size, shape and appearance and a disturbance in the usual arrangement.

A topical hydrocortisone cream would be most appropriate in a patient who has been diagnosed with: A. Psoriasis B. Impetigo C. Atopic dermatitis D. Eczema

D. Eczema - Topical steroid creams are usually not strong enough to treat psoriasis. Impetigo is a superficial skin infection and a steroid cream would be contraindicated. Atopic dermatitis refers to a chronic skin disorder.

What is the initial emergency measure to limit burn severity? A. Stabilize the client's condition B. Identify the type of burn C. Prevent heat loss D. Eliminate the heat source

D. Eliminate the heat source

A patient is diagnosed with allergic rhinitis. Which symptom below is NOT associated with allergic rhinitis? A. Paroxysmal sneezing B. Rhinorrhea C. Nasal congestion D. Facial pain

D. Facial pain

A patient has cough, pharyngitis, nasal discharge and fever. Which symptom is relatively uncommon in a patient with acute bronchitis? A. Cough B. Pharyngitis C. Nasal discharge D. Fever

D. Fever - Cough is the most common symptom associated with acute bronchitis. In the first few days of acute bronchitis, upper respiratory symptoms predominate. When fever appears along with cough, pneumonia should be strongly considered.

A patient complains of tragal pain, otic discharge, otic itching, and fever. Which symptom is inconsistent with otitis externa? A. Tragal pain B. Otic discharge C. Otic itching D. Fever

D. Fever - Fever, a typical systemic symptom, is inconsistent with otitis externa since the infection is superficial.

How do you describe the cervical lymphadenopathy associated with asymptomatic HIV infection? A. Movable, discrete, soft, and nontender lymph nodes B. Enlarged, warm, tender, firm, but freely movable lymph nodes C. Hard, unilateral, nontender, and fixed lymph nodes D. Firm but not hard, nontender, and mobile lymph nodes

D. Firm but not hard, nontender, and mobile lymph nodes

Which of the following areas of the body has the greatest percutaneous absorption? A. Sole of the foot B. Scalp C. Forehead D. Genitalia

D. Genitalia - Genitalia have the highest percutaneous absorption across the entire body. This is important because low potency creams will act with greater potency in this area.

Health maintenance in adults with sickle cell anemia includes which of the following? A. Early sterilization should be performed to prevent transmission of the disease B. Administer hepatitis A vaccine C. Avoid use of oral contraceptives because of increased risk of clotting D. Give folic acid 1mg PO daily

D. Give folic acid 1mg PO daily

When assessing Lenore, age 59, who has a sore throat, you note that she has a positive history of diabetes and rheumatic fever. These facts increase the likelihood that which of the following agents caused her sore throat? A. Neisseria gonorrhoeae B. Epstein-Barr virus C. Haemophilus influenzae D. Group A beta-hemolytic streptococcus

D. Group A beta-hemolytic streptococcus

Signs and symptoms of acute angle-closure glaucoma include A. Painless redness of the eyes B. Loss of peripheral vision C. Translucent corneas D. Halos around lights

D. Halos around lights

Which skin disorder in newborns occurs when the infant is placed on one side and the dependent half develops an erythematous flush while the upper half of the body becomes pale? A. Sebaceous gland hyperplasia B. Erythema toxicum C. Erythema toxicum neonatorum D. Harlequin color change

D. Harlequin color change

What is the name of the horizontal groove in the rib cage at the level of the diaphragm, extending from the sternum to the midaxillary line that occurs normally in some children, as well as in children with rickets? A. The sternal groove B. The rickettsial groove C. The manubrial groove D. Harrison's groove

D. Harrison's Groove - It occurs normally in some children and also occurs in children with rickets.

Marta, age 52, is scheduled for sclerotherapy. She asks you what to expect during her recovery. You tell her that she will A. be able to walk out of the office wearing shorts and looking great B. have to go home and put her feet up (either in bed or in a lounge chair) for several days C. be in the hospital for several days D. have her legs wrapped afterward with graduated compression stockings

D. Have her legs wrapped afterward with graduated compression stockings

Leonard is an 18 year old man whp has been taking isotretinoin (Accutane) for the treatment of acne for the past 2 months. Which of the following is the most important question for the clinician to ask at his follow up visit? A. Are you having problems remembering to take your medication? B. Have you noticed any dry skin around your mouth since you started using Accutane? C. Do you notice any iprovement in your skin? D. Have you noticed any recent changes in your ood?

D. Have you noticed any recent changes in your mood

One major approach to cancer prevention is A. Colonoscopy B. New drug trials C. Pap smears for women of all ages D. Host modification

D. Host modification - The three major approaches to cancer prevention are education, regulation, and host modification. Host modification refers to increasing knowledge about cancer genetics.

An increase of which immunoglobulin (Ig) signifies atopic disorders such as allergic rhinitis, allergic asthma, atopic dermatitis, and parasitic infection? A. IgG B. IgM C. IgA D. IgE

D. IgE

Large, flaccid bullae with honey-colored crusts around the mouth and nose are characteristic of A. A burn B. Rocky Mountain spotted fever C. Measles D. Impetigo

D. Impetigo

If a client presents with a deep, aching, red eye and there is no discharge, you should suspect A. Bacterial conjunctivitis B. Viral conjunctivitis C. Allergic conjunctivitis D. Iritis

D. Iritis

John, age 19, has just been given a diagnosis of mononucleosis. Which of the following statements is true? A. The offending organism is bacteria and should be treated with antibiotics B. Convalescence is usually only a few days and John should be back to normal in a week. C. Mono is rarely contagious D. John should avoid contact sports and heavy lifting.

D. John should avoid contact sports and heavy lifting

Your client, Shirley, has an elevated mean cell volume (MCV). What should you be considering in terms of diagnosis? A. Iron-deficiency anemia B. Hemolytic anemias C. Lead poisoning D. Liver disease

D. Liver disease - MCV indicates the average size of individual RBCs. MCV is elevated in megaloblastic anemias, liver disease, and some drugs

Which type of cancer has the highest mortality rate of all cancers and yet is one of the most preventable? A. Cervical cancer B. Bladder cancer C. Breast cancer D. Lung cancer

D. Lung cancer

A 45-year-old man presents with otitis externa. Likely causative pathogens include all of the following except: A. fungal agents B. P. auruginosa C. S. aureus D. M. catarrhalis

D. M. catarrhalis

Which treatment is considered the gold standard in tissue-conservative skin cancer removal? A. Cryosurgery B. Simple excision C. Photodynamic treatment D. Mohs' micrographic surgery

D. Mohs' micrographic surgery

Which treatment is considered the gold standard in tissue-conservative skin cancer removal? A. Cryosurgery B. Simple excision C. Photodynamic treatment D. Mohs' micrographic surgery

D. Mohs' micrographic surgery - Involves removal of the entire tumor with the smallest possible margin of normal skin

Which of the following statements is true regarding pulmonary tuberculosis? A. Manifestations are usually confined to the respiratory system B. Dyspnea is usually present in the early stages C. Crackles and bronchial breath sounds are usually present in all phases of the disease D. Night sweats are often noted as a manifestation of fever

D. Night sweats are often noted as a manifestation of fever - Systemic manifestations are usually present with pulmonary tuberculosis and symptoms are not confined to the respiratory system.

Which of the following statements is true regarding pulmonary tuberculosis? A. Manifestations are usually confined to the respiratory system B. Dyspnea is usually present in the early stages C. Crackles and bronchial breath sounds are usually present in all phases of the disease D. Night sweats are often noted as a manifestation of fever

D. Night sweats are often noted as a manifestation of fever - Systemic manifestations are usually present with pulmonary tuberculosis and symptoms are not confined to the respiratory system.

Bladder cancer can be detected early by A. An annual urine culture B. A bladder tumor marker blood test C. An annual cystoscopy D. None of the above, there is no early detection

D. None of the above, there is no early detection

Increased severity of underlying illness, presence of an indwelling urethral catheter, and use of broad-spectrum antibiotics are risk factors predisposing clients to the development of A. Tuberculosis B. Decreased mobility C. Pressure ulcers D. Nosocomial pneumonia

D. Nosocomial pneumonia

The best means to observe for jaundice is to: A. Observe the skin in diffuse light B. Scrape the skin to identify KOH positive lesions C. Observe the skin in direct sunlight D. Note the patient's sclera

D. Note the patient's sclera

Which of the following is NOT part of the differential for a patient who complains of cough? A. CHF B. GERD C. Asthma D. Obesity

D. Obesity

You prescribe a topical medication and want it to have maximum absorption, so you choose the following vehicle; A. Gel. B. Lotion C. Cream D. Ointment

D. Ointment

Tinea unguium is also known as A. Tinea capitis B. Pityriasis versicolor C. Tinea manuum D. Onychomycosis

D. Onychomycosis - Tinea unguium is tinea of the nails, also known as onychomycosis.

You've been counseling your client about her asthma. You realize she doesn't understand your suggestions when she tells you that she'll do which of the following: A. Cover the mattress and pillows in airtight, dust-proof covers. B. Wash the bedding weekly and dry it on a hot setting for 20 minutes. C. Avoid sleeping on natural fibers such as wool or down. D. Open the windows and air out the room daily.

D. Open windows and air out the room daily. - Opening the windows daily would allow allergens to enter.

An 80 year old woman comes in today with complaints of a rash on the left side of her face that is blistered and painful and accompanied by left-sided eye pain. The rash broke out 2 days ago, and she remembers being very tired and feeling feverish for a week before the rash appeared. On examination the rash follows the trigeminal nerve on the left and she has some scleral injection and tearing. You suspect herpes zoster ophthalmicus. Based on what you know to be complications of this disease, you explain to her that she needs A. Antibiotics B. A biopsy of the rash C. Immediate hospitalization D. Ophthalmological consultation

D. Ophthalmological consultation

Sherri's blood work return with a decreased mean cell volume (MCV) and a decreased mean cellular hemoglobin concentration (MCHC). What should you do next? A. Order a serum iron and total iron binding capacity (TIBC) B. Order a serum ferritin C. Order a serum folate level D. Order a serum iron, TIBC, and serum ferritin level

D. Order a serum iron, TIBC, and serum ferritin level - A decreased MCV and MCHC is indicative of a microcytic, hypochromic anemia.

Which of the following statements regarding the respiratory status of the pregnant woman is false? A. The thoracic cage may appear wider B. The costal angle may feel wider C. Respirations may be deeper D. Oxygenation is decreased

D. Oxygenation is decreased

What is a common inhaled allergen in allergic asthma (extrinsic asthma)? A. Smoke B. Cold air C. Strong smells D. Pet dander

D. Pet dander - The symptoms of allergic and nonallergic asthma are the same, but the triggers are not. Allergic asthma is triggered by inhaled exposure to allergens.

In a 22 year old woman using isotretinoin (Accutane) therapy, the NP ensures follow up to monitor for all of the following tests except: A. hepatic enzymes B. triglyceride measurements C. pregnancy test D. platelet count

D. Platelet count

Common causative organisms of acute suppurative conjunctivitis include all of the following except: A. Staphylococcus aureus B. Haemophilus influenzae C. Streptococcus pneumoniae D. Pseudomonas aeruginosa

D. Pseudomonas aeruginosa

Which drug may be associated with cough? A. Dextromethorphan B. Guaifenesin C. Albuterol D. Ramipril

D. Ramipril - Ramipril is an ACE inhibitor. Though estimates vary, as many as 20% of patients who receive ACE inhibitors develop a cough

A client with a platelet abnormality may present with A. Red to blue macular plaques B. Multiple freckle-like macular lesions in sun-exposed areas C. Numerous, small, brown, nonscaly macules that become more prominent with sun exposure D. Red,flat, nonblanchable petechiae

D. Red, flat, nonblanchable petechiae

A client with a platelet abnormality may present with A. Red to blue macular plaques B. Multiple freckle-like macular lesions in sun-exposed areas C. Numerous, small, brown, nonscaly macules that become more prominent with sun exposure D. Red,flat, nonblanchable petechiae

D. Red, flat, nonblanchable petechiae

Which of the following statements is true regarding weight and smoking cessation? A. Smokers weigh 10-20 lbs less than nonsmokers B. When smokers quit, 90% of them gain weight C. Men gain more weight than women when they quit D. Smokers gain weight after smoking cessation because they replace cigarettes with food

D. Smokers gain weight after smoking cessation because they replace cigarettes with food. - Smokers weigh 5-10lbs less than nonsmokers of comparable age and height.

Which of the following statements is true regarding weight and smoking cessation? A. Smokers weigh 10-20 lbs less than nonsmokers B. When smokers quit, 90% of them gain weight C. Men gain more weight than women when they quit D. Smokers gain weight after smoking cessation because they replace cigarettes with food

D. Smokers gain weight after smoking cessation because they replace cigarettes with food. - Smokers weigh 5-10lbs less than nonsmokers of comparable age and height.

Risk factors for oral cancers include A. A family history, poor dental habits, and use of alcohol B. Obesity, sedentary lifestyle, and chewing tobacco C. A history of diabetes, smoking, and a high fat intake D. Smokine, use of alcohol, and chewing tobacco

D. Smoking, use of alcohol, and chewing tobacco

Your client, Ms. Jones, has an elevated platelet count. You suspect A. Systemic lupus erythematosus B. Infectious mononucleosis C. Disseminated intravascular coagulation (DIC) D. Splenectomy

D. Splenectomy - Increased platelet count is seen in myeloproliferative leukemias, polycythemia vera, and status post splenectomy

A patient with pneumonia reports that he has rust colored sputum. What pathogen should the nurse practitioner expect? A. Mycoplasm pneumoiae B. Chlamydophila pneumoniae C. Staphylococcus aureus D. Streptococcus pneumoniae

D. Streptococcus pneumoniae - Clinical descriptions of mucus do not really help in clinical decision making regarding pneumonia, but certain clinical characteristics are associated with specific types of pneumonia.

You diagnose acute epiglottitis in Sally, age 5, and immediately send her to the local emergeny room. Which of the following symptoms would indicate that an airway obstruction is imminent? A. Reddened face B. Screaming C. Grabbing her throat D. Stridor

D. Stridor

What is the most common cause of sudden and unexpected death in infants, accounting for 80% of post-neonatal infant mortality with a peak incidence at 6 months? A. Choking B. Shaken baby syndrome C. Infantile pneumonia D. Sudden infant death syndrome (SIDS)

D. Sudden infant death syndrome (SIDS)

What is the most common rosacea trigger? A. Alcohol B. Cold weather C. Skin care products D. Sun exposure

D. Sun exposure

Which of the following therapeutic modalities is not useful for th management of acute atopic dermatitis? A. Emollients B. Compresses C. Ultraviolet light D. Tars

D. Tars - Tars are useful for chronic, dry, lichenified lesions

Which cancer can be cured with chemotherapy alone? A. Breast cancer B. Malignant melanoma C. Bladder cancer D. Testicular cancer

D. Testicular cancer

The diagnosis of tuberculosis does not need to be reported when A. The client's Mantoux test shows an induration of 15mm B. A case of tuberculosis is only suspected C. An asymptomatic client has a positive chest x-ray for pulmonary tuberculosis D. The Mantoux test shows a raised injected or red area without induration

D. The Mantoux test shows a raised injected or red area without induration

Which of the following statements about malignant melanomas is true? A. They usually occur in older adult males B. The client has no family history of melanomas C. They are common in blacks D. The prognosis is directly related to the thickness of the lesions

D. The prognosis is directly related to the thickness of the lesions

Which is the accepted mass screening test for lung cancer? A. An annual physical examination B. A chest X-Ray C. Sputum cytology D. There is no accepted mass screening test for lung cancer

D. There is no accepted mass screening test for lung cancer - Because of the cost, mass screening for lung cancer in healthy individuals with no risk factors is not recommended.

What is the mechanism of action of steroid hormones in cancer chemotherapy? A. They interfere with DNA or RNA synthesis B. They interfere with DNA replication by attacking DNA synthesis throughout the cell cycle C. They inhibit protein synthesis D. They alter the host environment for cell growth

D. They alter the host environment for cell growth

Which of the following is considered a therapeutic indication for a bronchoscopy? A. To evaluate indeterminate lung lesions B. To stage cancer preoperatively C. To determine the extent of injury secondary to burns, inhalation, or other trauma D. To remove a foreign body lodged in the trachea

D. To remove a foreign body lodged in the trachea.

M. pneumoniae and C. pneumoniae are respiratory pathogens commonly found in closed communities because they are: A. Spread via direct contact B. Associated with immunosuppression C. Spread via oral secretions D. Transmitted via cough

D. Transmitted via cough - These two organisms are common causes of pneumonia. Specifically, they are called atypical pathogens because they produce atypical pneumonia.

Prophylaxis for the first episode of Pneumocystitis jiroveci pneumonia in an adult or adolescent client infected with HIV is A. Isoniazid (Nydrazid) 300mg PO and pyridoxine (Vitamin B6 [Beesix] 50mg PO QD for 12 days B. Clarithromycin (Biaxin) 500mg PO BID for 2 weeks C. Rifampin (Rimactane) 600mg PO QD for 12 months D. Trimethoprim-sulfamethoxazole (TMP-SMZ) (Bactrim) DS 1 tablet PO QD for 10 days

D. Trimethoprim-sulfamethoxazole (TMP-SMZ) (Bactrim) DS 1 tablet PO QD for 10 days

You suspect that your new client Doug has hepatitis C, although he is asymptomatic at this point in time. Your suspicion is based on his medical history, which includes which of the following factors that has been identified as a red flag for this disease? A. Lactose intolerance B. Frequent sore throats and upper respiratory infections C. A history of mononucleosis at age 17 D. Unsafe sexual behaviors

D. Unsafe sexual behaviors

A patient with a viral upper respiratory infection cannot spread this to others via: A. Hand contact B. Droplet transmission C. Fomites D. Urine or stool

D. Urine or stool

A 49 year old man presents with a skin lesion suspicious for malignant melanoma. You describe the lesion as having: A. a deep black-brown coloring throughout B. sharp borders C. a diamer of 3mm or less D. variable pigmentation

D. Variable pigmentation

Jessica, an 8 year old third grader is broght to the office by her grandmother, who is the child's babysitter. She has complained of fever and sore throat for the past 2 days. Five other children in her class have been sick with sore throats. She denies difficulty swallowing and has been drinking fluids but has no appetite. ROS reveals that she has clear nasal drainage, hoarseness, and nonproductive cough. She denies vomiting but has had mild diarrhea. On examination she has a temperature of 101.5 F, 3+ erythematous tonsils, and palpable, tender cervical lymph nodes. Based on these findings, what is the most likely diagnosis? A. Mono B. Sinusitis C. Strep pharyingitis D. Viral pharyngitis

D. Viral pharyngitis

A metastatic tumor from below the diaphragm is suspected when you palpate which of the following nodes in the left supraclavicular space? A. Wringer's node B. Sims' node C. Wiskott-Aldrich node D. Virchow's node

D. Virchow's node - Virchow's node is an enlarged left supraclavicular node usually infiltrated with a metastatic tumor from below the diaphragm, especially of gastrointestinal origin

When should a rescue course of prednisolone be initiated for an attack of asthma? A. When the client is in step 1 (intermittent stage) B. When the client is in step 2 (mild persistent stage) C. When the client is in step 4 (severe persistent stage) D. Whenever the client needs it, at any time and at any step

D. Whenever the client needs it, at any time and at any step. - Attempts should be made to use systemic corticosteroids in an acute or rescue fashion: a short burst followed by tapering to the lowest possible dose and preferably discontinued, with inhaled steroids prescribed for chronic or maintenance therapy.

What is the easiest way to differentiate between otitis externa and otitis media? A. With otitis media, tender swelling is usually visible B. With otitis media, there is usually bilateral pain in the ears C. With otitis media, there is usually tenderness on palpation over the mastoid process D. With otitis externa, movement or pressure on the pinna is extremely painful

D. With otitis externa, movement or pressure on the pinna is extremely painful

Antibodies (inhibitors) directed against factor VIII can arise spontaneously in a number of situations. These include A. Clients who have mitral regurgitation B. As sequelae to a strep infection C. Clients on antibiotics D. Women who are several weeks postpartum after a normal labor and delivery

D. Women who are several weeks postpartum after a normal labor and delivery

Ginny age 62, comes to the office with a sudden onset of severe, throbbing eye pain and unilateral vision loss. Even before you examine her, you suspect A. a unilateral cataract B. iritis C. a detached retina D. acute angle-closure glaucoma

D. acute angle-closure glaucoma

Angie, age 17, is complaining of nasal congestion, sneezing, and itchiness of the eyes that worsens when she does yard work. She has been self-medicating with an over-the-counter nasal decongestant spray. Although this has been temporarily helpful, she reports that her symptoms are now worsening. You should A. counsel her to avoid yard work B. advise her of the rebound effect of the nasal spray C. order a nonsedating antihistamine and a corticosteroid nasal spray D. advise a combination nonsedating antihistamine and decongestant tablet

D. advise a combination nonsedating antihistamine and decongestant tablet

A 48-year-old man presents with a new-onset right eye vision change accompanied by dull pain, tearing, and photophobia. The right pupil is small, irregular, and poorly reactive. Vision testing obtained by using the Snellen chart is 20/30 OS and 20/80 OD. The most likely diagnosis is: A. unilateral herpetic conjunctivitis B. open-angle glaucoma C. angle-closure glaucoma D. anterior uveitis

D. anterior uveitis

Risk factors for POAG include all of the following except: A. African ancestry B. type 2 diabetes mellitus C. advanced age D. blue eye color

D. blue eye color

Treatment options in acute and recurrent allergic conjunctivitis include all of the following except: A. cromolyn ophthalmic drops B. oral antihistamines C. ophthalmological antihistamines D. corticosteroid ophthalmic drops

D. corticosteroid ophthalmic drops

According to the ARIA treatment guidelines, which of the following medications affords the LEAST control of rhinorrhea associated with allergic rhinitis? A. anticholinergic nasal spray B. antihistamine nasal spray C. corticosteroid nasal spray D. cromolyn nasal spray

D. cromolyn nasal spray

When percussing the chest, hyperresonance often is heard with A. lung cancer B. pneumonia C. pleural effusion D. emphysema

D. emphysema

A common site for atopic dermatitis in an adult is on the: A. Dorsum of the hand B. Face C. Neck. D. Flexor surfaces

D. flexor surfaces

All of the following are consistent with normal age-related vision changes EXCEPT: A. need for increased illumination B. increasing sensitivity to glare C. washing out of colors D. gradual loss of peripheral vision

D. gradual loss of peripheral vision

In infants and childre, the cardinal sign of right-sided congestive heart failure is A. cyanosis B. edema of the lower extremities C. tachypnea D. hepatomegaly

D. hepatomegaly

When doing a pelvic examination on Tara, age 19, you note painful, raised, reddened lesions filled with fluid around the labia. You diagnose these as A. lesions associated with syphilis B. a vaginal infection C. condyloma D. herpetic vesicles

D. herpetic vesicles

Which of the following represents a therapeutic option for ABRS in a patient with no recent antimicrobial care with treatment failure after 72 hours of appropriate-dose antimicrobial therapy? A. clindamycin B. clarithromycin C. ofloxacin D. high-dose amoxicillin with clavulunate

D. high-dose amoxicillin with clavulunate

Immunizations in adults age 65 and older should include A. diphtheria, tetanus, and pertussis every 5 years and pneumococcal vaccine every 6 years starting at age 65 B. pneumococcal vaccine at age 65 and tetanus and diphtheria every 10 years C. diphtheria, tetanus, and acellular pertussis every 10 years and pneumococcal vaccine at age 65 D. influenza immunization every fall, tetanus and diphtheria every 10 years, and pneumococcal vaccine at age 65

D. influenza immunization every fall, tetanus and diphtheria every 10 years, and pneumococcal vaccine at age 65

Which of the following medications is most appropriate for allergic rhinitis therapy in an acutely symptomatic 24-year-old machine operator? A. nasal cromolyn B. diphenhydramine C. flunisolide nasal spray D. loratadine

D. loratadine

Which of the following is consistent with the visual problems associated with macular degeneration? A. peripheral vision loss B. blurring of near vision C. difficulty with distant vision D. loss of the central vision field

D. loss of the central vision field

Treatment options for POAG include all of the following topical agents except: A. beta-adrenergic antagonists B. alpha2-agonists C. prostaglandin analogues D. mast cell stabilizers

D. mast cell stabilizers

Which of the following medications affords the best relief of acute nasal itch? A. anticholinergic nasal spray B. oral decongestant C. corticosteroid nasal spray D. oral antihistamine

D. oral antihistamine

Your client Annie had a normal vaginal delivery approximately 24 hours ago. The charge nurse calls you stat to say that Annie cannot breathe and is extremely agitated. You suspect A. a panic attack B. sudden onset pneumonia C. a stroke D. pulmonary embolism

D. pulmonary embolism

A 78-year-old woman has early bilateral senile cataracts. Which of the following situations would likely pose the greatest difficulty? A. reading the newspaper B. distinguishing between the primary colors C. following extraocular movements D. reading road signs while driving

D. reading road signs while driving

Prevention and prophylaxis in Meniere's disease include all of the following except: A. avoiding ototoxic drugs B. protecting the ears from loud noise C. limiting sodium intake D. restricting fluid intake

D. restricting fluid intake

Treatment options in generalized psoriasis vulgaris include all the following except: A. psoralen with ultraviolet A light (PUVA) therapy B. methotrexate C. cyclosporine D. systeic corticosteroids

D. systemic corticosteroids

Which of the following therapeutic modalities is not useful for th management of acute atopic dermatitis? A. Emollients B. Compresses C. Ultraviolet light D. Tars

D. tars

Andrea, age 16, is seen in your office for an injury to her eye. You remove a small foreign body from her cornea. There is no rust ring. In preparation for discharge, you would A. patch her eye B. prescribe anesthetic eye drops C. prescribe steroid eye drops D. tell her that she may return to normal activity and take an NSAID for pain if needed

D. tell her that she may return to normal activity and take an NSAID for pain if needed

Which of the following is an oral antimicrobial option for the treatment of a commonly acquired MRSA cutaneous infection: A. amoxicillin B. dicloxacillin C. cephalexin D. trimethoprim-sulfamethoxazole

D. trimethoprim-sulfamethoxazole

A patient with iron deficiency anemia takes iron supplementation daily. What should he be advised to avoid within a couple of hours of taking iron? a. An antacid b. Heavy exercise c. Potassium supplements d. Grapefruit juice

a. An antacid

An 18 year-old female patient has iron deficiency anemia. If the anemia has occurred in the past 3-4 months, what might be expected? a. An increased RDW b. A decreased RDW c. An elevated serum ferritin d. A decreased total iron binding capacity

a. An increased RDW

A 74 year-old is diagnosed with shingles. The NP is deciding how to best manage her care. What should be prescribed? a. An oral antiviral agent b. An oral antiviral agent plus an oral steroid c. An oral antiviral agent and topical steroid d. A topical steroid only

a. An oral antiviral agent

How often should hearing be screened in older adults? a. Annually b. Twice annually c. At each visit d. Only if symptoms exist

a. Annually

At what age would it be unusual to see thrush? a. At birth b. 2 months c. 6 months d. 8 months

a. At birth

An older adult presents with a pearly-domed nodular looking lesion on the back of the neck. It does not hurt or itch. He does not know how long he's had it. What is a likely etiology? a. Basal cell carcinoma b. Squamous cell carcinoma c. Malignant melanoma d. Actinic keratosis

a. Basal cell carcinoma

A short-acting anticholinergic medication can be used alone or in combination with a short-acting beta agonist to manage symptoms of which disease? a. COPD b. Benign prostatic hyperplasia c. Glaucoma d. Tachyarrhythmias

a. COPD

What medication used to treat patients who have GERD provides the fastest relief of heartburn symptoms? a. Calcium carbonate b. Ranitidine c. Amantadine d. Pantoprazole

a. Calcium carbonate

An adolescent takes isotretinoin for nodulocystic acne. She is on oral contraceptives. Both were prescribed by the dermatologist. The adolescent arrives in your clinic with a sinus infection. Her temperature is 99.5 degrees F and her blood pressure is 160/100. How should this be managed? a. Call the dermatologist to report the elevated BP b. Treat the sinus infection and recheck the BP in one week c. Discontinue the isotretinoin today d. Discontinue the oral contraceptive today

a. Call the dermatologist to report the elevated BP

A skin disorder has a hallmark finding of silvery scales. What word below describes this common condition? a. Chronic b. Infectious c. Contagious d. Acute

a. Chronic

A 74 year-old patient has peripheral artery disease (PAD). Which item listed below is the most important risk factor for PAD? a. Cigarette smoking b. Hyperlipidemia c. Diabetes d. Alcohol consumption

a. Cigarette smoking

A nurse practitioner performs a fundoscopic exam. He identifies small areas of dull, yellowish-white coloration in the retina. What might these be? a. Cotton wool spots b. Microaneurysms c. Hemorrhages d. Exudates

a. Cotton wool spots

A vegetarian presents to your clinic with iron deficiency anemia. What can the NP suffest to help alleviate this condition? a. Dark green leafy vegetables and dried peas and beans b. Mushrooms, oatmeal, and whole grain breads c. Beets, broccoli, and beef d. Baked potatoes, beets, and broccoli

a. Dark green leafy vegetables and dried peas and beans

A 16 year-old has been diagnosed with Lyme disease. Which drug should be used to treat him? a. Doxycycline b. Amoxicillin-clavulanate c. Trimethroprim-sulfamethoxazole d. Cephalexin

a. Doxycycline

A patient has heavy menses. Which lab value below reflects an iron deficiency anemia? a. Elevated TIBC b. Decreased TIBC c. Normal serum iron d. Decreased RDW

a. Elevated TIBC

What are the most common signs and symptoms associated with mononucleosis? a. Fatigue and lymphadenopathy b. Cough and pharyngitis c. Splenomegaly and fever d. Rash and pharyngitis

a. Fatigue and lymphadenopathy

A 7 year-old has a complaint of ear pain. If he has otitis externa, which complaint is most likely? a. He has tragal pain b. He has difficulty hearing the TV c. He has fever d. He has a concurrent upper respiratory infection

a. He has tragal pain

A patient with a positive history if a tick bite about 2 weeks ago and erythema migrans has a positive ELISA for Borrelia. The Western blot is positive. How should he be managed? a. He should receive doxycycline for Lyme disease b. He should receive penicillin for syphilis c. He does not have Lyme disease or syphilis d. He needs additional testing to confirm Lyme disease

a. He should receive doxycycline for Lyme disease

The gold standard for diagnosing pneumonia on chest x-ray is: a. Infiltrates b. interstitial fluid c. cavitation d. "pooling"

a. Infiltrates

An older female adult with pendulous breasts complains of a little "burning" under her breasts. The nurse practitioner observes a yeasty smelling discharge with mild maceration under the breasts. What is this? a. Intertrigo b. Impetigo c. Tinea corporis d. Fungal dermatitis

a. Intertrigo

Which suggestion below is the standard for treating iron deficiency anemia in infants and children? a. iron supplementation in divided doses between meals with orange juice b. Iron supplementation once daily with the largest meal c. Meat > 5 servings per week with citrus juice d. Iron supplementation with orange juice 5 times weekly

a. Iron supplementation in divided doses between meals with orange juice

In a patient with mononucleosis, which laboratory abnormality is most common? a. Lymphocytosis and atypical lymphocytes b. Elevated monocytes c. A decreased total white count d. Elevated liver enzymes

a. Lymphocytosis and atypical lymphocytes

Which medication listed below could potentially exacerbate CHF in a susceptible individual? a. Naproxen b. HCTZ c. Lovastatin d. Loratadine

a. Naproxen

A patient who is 65 "suffers with allergies". She describes that she's had this since she was in her teens. What agent could be safely used in her to help with rhinitis, sneezing, pruritis, and congestion? a. Nasal steroid b. Ipratropium c. Antihistamine d. Decongestant

a. Nasal steroid

A patient calls your office. He states that he just came in from the woods and discovered a tick on his upper arm. He states that he has removed the tick and the area is slightly red. What should be advised? a. No treatment is needed b. He should be prescribed doxycycline c. He needs a topical scrub to prevent Lyme disease d. He should come to the office for a ceftriaxone injection

a. No treatment is needed

The agent commonly used to treat patients with scabies is permethrin. How often is it applied to eradicate scabies? a. Once b. Once daily for 3 days c. Twice daily for 3 days d. Once daily for one week

a. Once

An eczematous skin reaction may result from A. Penicillin B. Allopurinol (Zyloprim) C. An oral contraceptive D. Phenytoin (Dilantin)

a. Penicillin

What is the proper technique to safely remove a tick from a human to help prevent Lyme disease? a. Pull them off with tweezers b. Use petroleum jelly c. Use isopropyl alcohol d. Use a hot match

a. Pull them off with tweezers

A patient presents with severe toothache. She reports sensitivity to heat and cold. There is visible pus around the painful area. What is this termed? a. Pulpitis b. Caries c. Gingivitis d. Periodontitis

a. Pulpitis

A 39 year-old has a sudden onset of a red eye. He reports sensitivity to light and the sensation of a foreign body, though his history for a foreign body is negative. He does not wear contact lenses. How should the NP manage this? a. Refer to ophtalmology b. Treat for viral conjunctivitis c. Treat for bacterial conjunctivitis d. Observe for 24 hours if visual acuity is normal

a. Refer to ophthalmology

Persons with which skin phototype (SPT) sunburn easily after 30 minutes in the sun but never tan? A. SPT I B. SPT II C. SPT III D. SPT IV

a. SPT I

A homeless patient presents to the free clinic. She should be screened for disease that are most prevalent in this population. These would be: a. TB, HIV, and hepatitis b. pregnancy and STDs c. urinary tract infection and STDs d. diabetes, HIV, and foot ulcers

a. TB, HIV, and hepatitis

A one week-old infant has a mucopurulent eye discharge bilaterally. What explains the etiology of the discharge? a. The mother probably has an STD b. The infant likely has a plugged tear duct c. This is bacterial conjunctivitis d. This viral conjunctivitis

a. The mother probably has an STD

A one week-old infant has mucopurulent eye discharge bilaterally. What explains the etiology of the discharge? a. The mother probably has an STD b. The infant likely has a plugged tear duct c. This is bacterial conjunctivitis d. This is viral conjunctivitis

a. The mother probably has an STD

A 48 year-old patient has the following laboratory values. How should they be interpreted? HCV IgG (+), RIBA (radio immuno blot assay) (+) a. The patient has hepatitis C b. The patient does not have hepatitis C c. The patient should consider immunization d. The results are indeterminate

a. The patient has hepatitis C

A 4 year-old was diagnosed and treated for left acute otitis media 4 weeks ago. She is here today for a well-child visit. There is an effusion in the left ear. She denies complaints. How should this be managed? a. This should be monitored b. She should be given another antibiotic c. She should be evaluated with pneumatic otoscopy d. She needs a tympanogram

a. This should be monitored

What is the most common nutrition syndrome in elderly patients? a. Undernutrition b. Over nutrition c. Malnourishment d. Vitamin B12 deficiency

a. Undernutrition

A possible adverse effect with the use of a first generation antihistaine such as dipenhydraine in an 80 year old man is: A. Urinary retention B. hyertension C. tachycardia D. urticaria

a. Urinary retention

A patient with eczema asks for a recommendation for a skin preparation to help with xerosis. What should the NP respond? a. Use a petroleum based product b. Use a hypoallergenic lotion c. Use any hypoallergenic product d. No particular product is better than the other

a. Use a petroleum based product

A patient presents to clinic with a complaint of a red eye. Which assessment below rules out the most worrisome diagnoses? a. Usual visual acuity b. Normal penlight exam c. Normal fundoscopic exam d. Negative photophobia

a. Usual visual acuity

Which medication listed below can exacerbate the symptoms of GERD? a. Verapamil b. Metformin c. Ferrous sulfate d. Ceftriaxone

a. Verapamil

Which of the following can NOT be a microcytic anemia? a. Vitamin B12 b. Anemia of chronic disease c. Iron deficiency anemia d. Thalassemia

a. Vitamin B12

A patient with asthma presents with an acute episode of wheezing, coughing, and fever. He is wheezing in the right upper lobe. His cough is non-productive, and he denies nasal symptoms. Which symptoms are not likely related to his asthma? a. Wheezing and fever b. Coughing and wheezing c. Fever and coughing d. Coughing and fever

a. Wheezing and fever

An example of a drug that targets the renin-angiotensin-aldosterone system is: a. an ACE inhibitor b. a beta blocker c. a calcium channel blocker d. a diuretic

a. an ACE inhibitor

The symptoms of asthma in an elderly patient: a. are more likely to be a cough b. are more likely to be characterized by wheezing c. have greater variability than in younger adults d. are distinct from chronic heart failure

a. are more likely to be a cough

Cranial nerve II could be assessed in a young infant by: a. assessing squinting response to bright light b. feeling for squeezing of fingers c. observing response to loud noise d. assessing rapid eye movement

a. assessing squinting response to bright light

A patient is found to have eosinophilia. An expected finding is: a. asthma exacerbation b. bronchitis c. hepatitis d. osteoporosis

a. asthma exacerbation

An example of a first generation cephalosporin used to treat a skin infection is: a. cephalexin b. cefuroxime c. cefdinir d. ceflamore

a. cephalexin

An elderly female with emphysema who is treated with inhaled steroids should: a. consider treatment for osteoporosis prophylaxis b. have oxygen at home for exacerbations c. use inhaled steroid and beta-agonists d. be watched for softening of teeth

a. consider treatment for osteoporosis prophylaxis

A patient who is at high risk for skin cancer should: a. examine his skin monthly for changes b. be examined by a dermatologist quarterly c. use emollients regularly d. eat foods high in vitamin C

a. examine his skin monthly for changes

An unusual symptom associated with acute bronchitis is: a. fever b. cough c. pharyngitis d. purulent sputum

a. fever

A common symptom in older patients with cataracts is: a. glare b. poor peripheral vision c. falls d. eye pain

a. glare

HIV testing during pregnancy: a. is recommended during pregnancy b. is an "opt-in" approach c. is better performed in the third trimester d. produces many false positives

a. is recommended by ACOG

An example of a short-acting beta agonist is: a. levalbuterol b. salmeterol c. mometasone d. beclomethasone

a. levalbuterol

A patient with mononucleosis would most likely have: a. lymphocytosis b. eosinophilia c. leukocytosis d. monocytosis

a. lymphocytosis

Epstein-Barr virus is responsible for: a. mononucleosis b. the most common cause of pharyngitis c. most teenage cases of pharyngitis d. viral pharyngitis in young children

a. mononucleosis

An overweight 76 year-old female with a recent onset of diabetes has longstanding hypertension and hyperlipidemia. She has developed atrial fibrillation, The nurse practitioner knows that she will be at risk for: a. an S3 gallop b. CHF c. shortness of breath d. hypothyroidism

b. CHF

Which long-acting antihistamine listed below is sedating? a. Loratadine b. Cetirizine c. Azelastine d. Fexofenadine

b. Cetirizine

What laboratory test could help differentiate acute bronchitis from pneumonia in a patient with a productive cough? a. CBC b. Chest x-ray c. Sputum specimen d. Pulmonary function tests

b. Chest x-ray

An 80 year-old is having difficulty hearing. When the nurse practitioner examines him, she is unable to visualize the tympanic membrane because of cerumen. This produces what kind of hearing loss? a. Tympanic b. Conductive c. Sensorineural d. Artificial

b. Conductive

A tympanic membrane (TM) is erythematous. Which factor listed below is NOT the cause of an erythematous TM? a. Acute inflammation b. Coughing c. High fever d. Crying

b. Coughing

A patient presents with small vesicles on the lateral edges of his fingers and intense itching. On close inspection, there are small vesicles on the palmar surface of the hand. What is this called? a. Seborrheic dermatitis b. Dyshidrotic dermatitis c. Herpes zoster d. Varicella zoster

b. Dyshidrotic dermatitis

A 24 year-old patient presents to your clinic. She states that she has vomited for the last 5 mornings and until early afternoon. She feels better in the evenings. She denies fever. What lab tests should be monitored? a. Progesterone and ketones b. Electrolytes and serum pregnancy c. Electrolytes and hepatitis panel d. Metabolic panel and potassium level

b. Electrolytes and serum pregnancy

A patient has a lower leg wound that appears infected. It is red, warm to touch, and edematous. He had an acute onset of pain, symptoms, and low grade fever. What is this? a. Cellulitis b. Erysipelas c. Impetigo d. An allergic reaction

b. Erysipelas

A contact lens wearer presents with an erythematous conjunctiva. He denies blurred vision. There is scant drainage and crusting around the eye. He reports that there was crusting when he woke up this morning. How should the exam begin? a. The patient should wash his hands b. His visual acuity should be measured in each eye c. Fluorescein stainging should be assessed d. Extraocular eye movements should be assessed

b. His visual acuity should be measured in each eye

The cranial nerve responsible for vision is Cranial Nerve: a. 2 b. II c. two d. ii

b. II

A child and father live in an old house. They both are found to be lead toxic. What type anemia is typically observed in patients who are lead toxic? a. Pernicious anemia b. Iron deficiency anemia c. Lead anemia d. Anemia of chronic disease

b. Iron deficiency anemia

Ipratropium is very widely used in the treatment of COPD. Which of the following statements about ipratropium is correct? a. It slows thre progression of COPD b. It decreases parasympathetic tone and produces bronchodilation c. It has anti-inflammatory actions and reduces bronchoconstriction d. It is less effective than a beta agonist in producing bronchodilation

b. It decreases parasympathetic tone and produces bronchodilation

Which medication is contraindicated for lone use in treating asthma? a. Short-acting bronchodilator b. Long-acting bronchodilator c. Inhaled steroid d. Oral steroid

b. Long-acting bronchodilator

An older adult has suspected B12 deficiency. Which of the following lab indices is more indicative of a B12 deficiency? a. Microcytosis b. Macrocytosis c. Leukocytosis d. Thrombocytopenia

b. Macrocytosis

A vitamin B12 deficiency might be suspected in an older patient with what complaints? a. Fatigue and restless legs b. Memory issues and glossitis c. Painful legs with exercise d. Insomnia and anorexia

b. Memory issues and glossitis

Which of the following characteristics are always present in a patient with COPD? a. Productive cough b. Obstructed airways c. Shortness of breath d. Hypercapnia

b. Obstructed airways

A patient has a "herald patch" and is diagnosed with pityriasis rosea. Where is the "herald patch" found? a. On the affected limb b. On the chest c. Close to the scalp d. Behind one of the ears

b. On the chest

A patient presents with tragal pain. What is the most likely diagnosis? a. Otitis media b. Otitis externa c. Presbycusis d. Mastoiditis

b. Otitis externa

A 42 year-old hypertensive patient was given a thiazide diuretic 4 weeks ago. On his return visit today, he reports feeling weak and tired. What should the NP consider to evaluate the weakness and fatigue? a. Blood pressure b. Potassium level c. CBC d. CPK

b. Potassium level

A 32 year-old patient is a newly diagnosed diabetic. She has developed a sinus infection. Her symptoms have persisted for 10 days. Six weeks ago she was treated with amoxicillin for an upper respiratory infection. It cleared without incident. What should be recommended today? a. Prescribe amoxicillin again b. Prescribe amoxicillin-clavulanate today c. Do not prescribe an antibiotic; a decongestant is indicated only d. Prescribe a decongestant and antihistamine

b. Prescribe amoxicillin-clavulanate today

A 3 year-old has been recently treated for an upper respiratory infection (URI) but drainage from the right nostril persists. What should the NP suspect? a. Allergic rhinitis b. Presence of a foreign body c. Unresolved URI d. Dental caries

b. Presence of a foreign body

A 95 year-old male has lost muscle mass as he has aged. He does not have any underlying disease that has caused this loss. What is this termed? a. Hypoproteinemia b. Sarcopenia c. Cachexia d. Dithering

b. Sarcopenia

Which medication should be avoided in a patient with a sulfa allergy? a. Sulfonylurea b. Sulfamethoxazole c. Naproxen d. Cefazolin

b. Sulfamethoxazole

A 74 year-old male patient has sustained a laceration to his foot. His last tetanus shot was more than 10 years ago. He has completed the primary series. What should be recommended? a. Tetanus toxoid only b. Tetanus and diphtheria only c. His primary series will protect him d. Tetanus, diphtheria, and acellular pertussis (Tdap)

b. Tetanus and diphtheria only

A 6 month-old infant has a disconjugate gaze. The nurse practitioner observes that the 6 month old tilts his head when looking at objects in the room. Which statement is true? a. Nystagmus will be present b. The infant will have an abnormal cover/uncover test c. The patient's vision is 20/200 d. He needs at CT to tule out an ocular tumor

b. The infant will have an abnormal cover/uncover test

A 43 year-old patient who has been diagnosed with hepatitis B has the following laboratory values. How should they be interpreted? HCV IgG(-), RIBA (radio immuno blot assay) (-) a. The patient has hepatitis B and hepatitis C b. The patient does not have hepatitis C c. The patient could have hepatitis C d. The results are indeterminate

b. The patient does not have hepatitis C

A 3 year-old female child had a fever of 102 degrees F for 3 days. Today she woke up from a nap and is afebrile. She has a maculopapular rash. Which statement is true? a. This child probably has measles b. The rash will blanch c. This is a streptococcal rash d. This could be Kawasaki disease

b. The rash will blanch

How should the class effect of the nasal steroids be described? a. There is alot of variation among agents within the class b. There are no significant systemic effects with these c. There are high rates of nasal bleeding d. It is generally no well-tolerated

b. There are no significant systemic effects with these

The nurse practitioner performs a fundoscopic exam on a patient who has recently been diagnosed with hypertension. The nurse practitioner identifies AV nicking. a. This is an incidental finding b. This is indicative of long standing hypertension c. The patient should be screened for diabetes d. The patient should be referred to ophthalmology

b. This is indicative of long standing hypertension

A 75 year-old male quit smoking 3 years ago. He has come in today for an exam. He is walking daily and has lost excess weight. On examination, the patient is noted to have an absence of hair growth on his lower legs. Which statement is true regarding this patient? a. This is a normal consequence of aging b. This might indicate disease in the lower extremities c. It might be from exercise initiation d. This is from long-term smoking

b. This might indicate disease in the lower extremities

What finding characterizes shingles? a. Pain, burning, and itching b. Unilateral dermatomal rash c. Grouped vesicles d. Resolution of rash and crusting

b. Unilateral dermatomal rash

A one year-old patient's mother reports allergy to gelatin. The mother describes the reaction as "lips swelling and breathing difficulties which necessitated a trip to the emergency department". Which immunizations should be avoided: a. Varicella, DTap, and MMR b. Varicella and MMR c. IPV, DTap, and hepatitis B d. Hepatitis A and B, meningococcal

b. Varicella and MMR

A patient has suspected scarlet fever. He likely has a sandpaper rash and: a. negative throat swab b. a positive rapid strept test c. diarrhea with abdominal cramps d. petechiae on the fingertips

b. a positive rapid strept test

Syphilis may present as: a. a discharge b. a rash c. a painful lesion d. dysuria

b. a rash

Patients with cough variant asthma: a. all wheeze b. all cough c. can cough or wheeze d. have dyspnea

b. all cough

The primary therapeutic intervention for patients who present with hives is: a. steroids b. anti-histamines c. calcium channel blockers d. topical steroid cream

b. anti-histamines

A 9 year-old has been diagnosed with chickenpox. A drug that should be avoided in him is: a. penicillin b. aspirin c. ibuprofen d. sulfa

b. aspirin

Group A Strept pharyngitis: a. is characterized by a single symptom b. can be accompanied by abdominal pain c. usually does not have exudative symptoms d. is commonly accompanied by an inflamed uvula

b. can be accompanied by abdominal pain

The most common symptom associated with acute bronchitis is: a. fever b. cough c. pharyngitis d. purulent sputum

b. cough

A serum ferritin level: a. could indicate thalassemia in a patient b. demonstrates the amount of iron in storage c. indicates when a patient has iron deficiency anemia d. confirms a low hemoglobin and hematocrit

b. demonstrates the amount of iron in storage

A 75 year-old patient with long-standing hypertension takes an ACE inhibitor and a thiazide diuretic daily. Today his blood pressure is 128/88, mildly elevated compared to usual; heart rate is 105 bpm. He has developed dyspnea on exertion and peripheral edema over the past several days. This probably indicates: a. worsening hypertension b. development of congestive heart failure (CHF) c. noncompliance with medication d. fluid or sodium excess

b. development of congestive heart failure (CHF)

The major laboratory abnormality noted in patients with pneumonia is: a. eosinophilia b. leukocytosis c. Gram stain positive d. leukopenia

b. leukocytosis

A 71 year-old female presents with a vesicular rash that burns and itches. Shingles is diagnosed. An oral antiviral: a. must be started within 72 hours of the onset of symptoms b. must be started within 72 hours of the onset of the rash c. can be started at anytime d. will nearly eliminate the risk of post-herpetic neuralgia

b. must be started within 72 hours of the onset of the rash

A patient with anemia of chronic disease probably has a: a. macrocytic anemia b. normocytic anemia c. hypochromic anemia d. hyperchromic anemia

b. normocytic anemia

A patient takin an ACE inhibitor should avoid: a. strenous exercise b. potassium supplements c. protein rich meals g. grapefruit juice

b. potassium supplements

The most common symptoms associated with gastroesophageal reflux disease (GERD) are heartburn and: a. cough b. regurgitation and dysphagia c. cough and hoarseness d. belching and sore throat

b. regurgitation and dysphagia

An immune response to Group A Streptococcal infections involving the heart is: a. Kawasaki syndrome b. rheumatic fever c. hemolytic disease d. pericarditis

b. rheumatic fever

The hearing loss associated with aging involves: a. the 8th cranial nerve b. sensorineural hearing loss c. conductive hearing loss d. noise damage

b. sensorineural hearing loss

The pneumococcal immunization in infants has: a. decreased the episodes of acute otitis media due to H. flu b. shifted the pathogenesis to fewer cases of S. pneumoniae c. eradicated acute otitis media due to S. pneumoniae d. improved the prognosis of acute otitis media

b. shifted the pathogenesis to fewer cases of S. pneumoniae

A nurse practitioner is working in a minor care area of an emergency department. A patient without insurance arrives that has a puncture wound caused by an unknown sharp object in a trash container. A dirty needle is suspected. The nurse practitioner: a. should administer a tetanus injection only since he has no insurance b. should prescribe appropriate medications for HIV exposure even though she knows he can't afford them c. should not mention the possibility of HIV from a dirty needle d. can offer to buy the HIV medications for $50 with her employee discount at the pharmacy next door

b. should prescribe appropriate medications for HIV exposure even though she knows he can't afford them

Mild persistent asthma is characterized by: a. limitation in activity due to bronchoconstriction b. symptoms occurring more than twice weekly c. wheezing and coughing during exacerbations d. shortness of breath with exercise

b. symptoms occuring more than twice weekly

When examining the vessels of the eye: a. the veins are smaller than the arteries b. the arteries are smaller than the veins c. the arteries are dark red d. the arteries pulsate

b. the arteries are smaller than the veins

A young child has developed a circumferential lesion on her inner forearm. It is slightly raised, red, and is pruritic. It is about 2.5 cm in diameter. This is probably related to: a. a genetic disorder b. the child's new cat c. juvenile rheumatoid arthritis d. a psoriatic lesion

b. the child's new cat

When can a child with chickenpox return to daycare? a. 24 hours after he is fever-free b. 48 hours after he is fever-free c. After all lesions have crusted over d. When he no longer itches

c. After all lesions have crusted over

A 70 year-old male patient has an elevated MCV with an anemia. His triglycerides are 420. What should be suspected? a. Pernicious anemia b. Folate deficiency c. Alcohol abuse d. Hypertriglyceridemia

c. Alcohol abuse

A two year-old with sickle cell anemia (SCA) should receive which immunizations? a. All routine childhood immunizations at an accelerated rate b. All routine childhood immunizations at a decelerated rate c. All routine childhood immunizations at the usual time d. Immunizations should be limited in this group

c. All routine childhood immunizations at the usual time

An elderly patient has been diagnosed with shingles on the right lateral aspect of her trunk. It appeared initially yesterday. It is very painful. How should she be managed? a. Treatment with a topical lidocaine patch only b. An oral antiviral agent and NSAIDs c. An oral antiviral agent and pain medication d. An oral antiviral agent, pain medication, and oral steroids

c. An oral antiviral agent and pain medication

What is the most important thing a woman can do to have youthful, attractive skin? A. Keep well hydrated B. Use sunscreen with an SPF of at least 45 C. Avoid smoking D. Use mild defatted or glycerine soaps

c. Avoid smoking

A 44 year-old non-smoker is diagnosed with pneumonia. He is otherwise healthy and does not need hospitalization at this time. Which antibiotic can be used for empirical treatment according to the 2007 Infectious Diseases Society of America/American Thoracic Society? a. Erythromycin b. Levofloxacin c. Azithromycin d. Amoxicillin

c. Azithromycin

A healthy 7 year-old child is diagnosed with pneumonia. He is febrile but in no distress. What is the preferred treatment for him? a. Supportive measures, it is probably viral b. Amoxicillin, doses 80-100 mg/kg/d c. Azithromycin d. Doxycycline

c. Azithromycin

A patient with diabetes has a right lower leg that has recently become edematous, erythematous, and tender to touch over the anterior shin. There is no evidence of pus, and the leg is warm to touch. What is an important diagnosis to consider? a. Deep vein thrombosis (DVT) b. Buerger's disease c. Cellulitis d. Venous disease

c. Cellulitis

A patient has a penicillin allergy. He describes an anaphylactic reaction. Which medication class should be specifically avoided in him? a. Quinolones b. Macrolides c. Cephalosporins d. Tetracyclines

c. Cephalosporins

A 6 year-old being treated for community-acquired pneumonia (CAP) has been taking azithromycin in therapeutic doses for 72 hours. His temperature has gone from 102F to 101F. What should be done? a. Continue the same dose and monitor his status b. Increase the dose to high dose azithromycin c. Change antibiotics to a penicillin d. This is probably viral, stop the antibiotic

c. Change antibiotics to a penicillin

A patient has 2 palpable, tender, left pre-auricular nodes that are about 0.5cm in diameter. What might also be found in this patient? a. Sore throat b. Ulceration on the tongue c. Conjunctivitis d. Ear infection

c. Conjunctivitis

A patient has developed rapid loss of hearing over the past several weeks. What cranial nerve should be assessed? a. Cranial Nerve III b. Cranial Nerve V c. Cranial Nerve VIII d. Cranial Nerve X

c. Cranial Nerve VIII

A 63 year-old male has been your patient for several years. He is a former smoker who takes simvastatin, ramipril, and an aspirin daily. His blood pressure and lipids are well controlled. He presents to your clinic with complaints of fatigue and "just not feeling well" for the last few days. His vital signs and exam are normal. His hepatitis panel is negative for infectious hepatitis. What is the most likely cause of his elevated liver enzymes? a. He has received a generic version of simvastatin b. He is an alcoholic in denial c. Daily grapefruit consumption for past 10 days d. Rare liver toxicity from a usual dose of simvastatin

c. Daily grapefruit consumption for the past 10 days

An older adult wants to be revaccinated with the pneumonia vaccine. He states that his last one was 8 or 10 years ago and his friend was revaccinated. What is the recommendation of CDC and ACIP on revaccination after age 65 in an individual who has already received one at or after age 65? a. Revaccination is optional b. Revaccination is recommended every 5 years c. Do not revaccinate d. Revaccinate never more than twice

c. Do not revaccinate

Which group of medications would not be used to treat a patient with CHF? a. Ramipril, aspirin, metoprolol b. Digoxin, furosemide, aspirin c. Fosinopril, HCTZ, verapamil d. Furosemide, enalapril, aspirin

c. Fosinopril, HCTZ, verapamil

A patient who presents with a complaint of sudden decreased visual acuity has a pupil that is about 4 mm, fixed: The affected eye is red. What might be the etiology? a. Stroke b. Brain tumor c. Glaucoma d. Cataract

c. Glaucoma

A patient presents with hematuria, RBC casts, and proteinuria. What is a likely explanation? a. Acute tubular necrosis b. Chronic renal insufficiency c. Glomerular disease d. Prerenal disease

c. Glomerular disease

A patient with mononucleosis has pharyngitis, fever, and lymphadenopathy. His symptoms started 3 days ago. a. He will have a positive "Monospot" b. He will have a normal CBC c. He could have negative "Monospot" d. He could have a positive Monospot and a normal CBC

c. He could have negative "Monospot"

A patient has the following laboratory values. What does this mean? Hepatitis A: (-) IgM (-) IgG a. He has hepatitis A b. He has immunity to hepatitis A c. He has no immunity to hepatitis A d. More data is needed

c. He has no immunity to hepatitis A

An infant is brought to the nurse practitioner because his gaze is asymmetrical. Which finding indicates a need for referral to ophthalmology? a. He is 2 months of age b. He is 3 months of age c. He has persistent strabismus d. His red reflex is normal

c. He has persistent strabismus

A patient with environmental allergies presents to your clinic. She takes an oral antihistamine every 24 hours. What is the most effective single maintenance medication for allergic rhinitis? a. Antihistamine b. Decongestant c. Intranasal glucocorticoids d. Leukotriene blockers

c. Intranasal glucocorticoids

What is true regarding the zoster vaccine given to older adults to prevent shingles? a. It is a weakened form of the chickenpox virus b. It is the same as the chickenpox virus c. It contains significantly more virus than the chickenpox vaccine d. It is not related to the chickenpox immunization at all

c. It contains significantly more virus than the chickenpox vaccine

Papilledema is noted in patient with a headache. What is the importance of papilledema in this patient? a. It is not related to this patient's headache b. It is an incidental finding in patients with migraines c. It could be an important finding in this patient d. This is a common finding in patients with headaches

c. It could be an important finding in this patient

Client teaching is an integral part of successfully treating pediculosis. Which of the following statements would you incorporate in your teaching plan? A. "It's ok to resume sharing combs, headsets, etc. after being lice free for 1 month." B. "Soak your combs and brushes in rubbing alcohol for 8 hours." C. "Itching may continue after successful treatment for up to a week." D. "Spraying of pesticides in the immediate environment is essential to prevent recurrence."

c. Itching may continue after successful treatment for up to a week

A patient who is 52 years old presents to your clinic for an exam. You notice a yellowish plaque on her upper eyelid. It is painless. What should the NP assess? a. Vision in the affected eye b. Sedimentation rate c. Lipid levels d. Liver function studies

c. Lipid levels

A 6 day-old has a mucopurulent eye discharge bilaterally. What historical finding explains the etiology of the discharge? a. Infant is Hepatitis B positive b. Infant received silver nitrate drops c. Mother has chlamydia d. Delivery was by C-section

c. Mother has chlamydia

Which antihistamines are preferred agents in treating allergic rhinitis? a. Short-acting b. Long-acting c. Once daily, non-sedating d. Long-acting, sedating

c. Once daily, non-sedating

A 63 year-old male has been your patient for several years. He is a former smoker who takes simvastatin, ramipril, and an aspirin daily. His blood pressure and lipids are well controlled. He presents to your clinic with complaints of fatigue and "just not feeling well" for the last few days. His vital signs and exam are normal. What should be done next? a. Order a CBC and consider waiting a few days if normal b. Inquire about feelings of depression and hopelessness c. Order a CBC, metabolic panel, TSH, and urine analysis d. Order a B12 level, TSH, CBC, and chest x-ray

c. Order a CBC, metabolic panel, TSH, and urine analysis

A patient recently received an antibiotic for 10 days for pneumia. His respiraotry symptoms have resolved but today he calls the office. He reports having severe watery diarrhea, abdominal cramping, and low-grade fever. What should be done? a. Give an anti-diarrheal agent b. Encourage the patient to force fluids c. Order stool specimens d. Wait 24 hours for resolution of symptoms

c. Order stool specimens

Swimmer's ear is diagnosed in a patient with tragal tenderness. What other symptom might he have? a. Otitis media b. Hearing loss c. Otic itching d. Fever

c. Otic itching

A 74 year-old patient has a pill-rolling tremor. With what disease is this often associated? a. Psychosomatic disorder b. Multiple sclerosis c. Parkinson's disease d. Charcot-Marie-Tooth Syndrome

c. Parkinson's disease

An adolescent has acne. The nurse practitioner prescribed a benzoyl peroxide product for him. What important teaching point should be given to this adolescent regarding the benzoyl peroxide? a. Don't apply this product more than once daily b. This often causes peeling of the skin c. Photosensitivity of the skin can occur d. Hypersensitivity can occur with repeated use

c. Photosensitivity of the skin can occur

A patient has a urinary tract infection. What findings on a urine dipstick best describe a typical urinary tract infection? a. Positive leukocytes b. Positive nitrates c. Positive leukocytes, positive nitrites d. Positive nitrates and hematuria

c. Positive leukocytes, positives nitrites

An elderly patient who has a red eye with tearing was diagnosed with conjunctivitis. What characteristics below reflect viral conjunctivitis? a. Moderate tearing b. Profuse exudate c. Profuse tearing d. Moderate exudate

c. Profuse tearing

Pastia lines are present in which disease? A. Toxic shock syndrome B. Rocky Mountain spotted fever C. Scarlet fever D. Meningococcemia

c. Scarlet fever

Which of the following is a secondary skin lesion? A. Acne nodule B. Neoplasm C. Seborrheic dermatitis D. Herpes simplex

c. Seborrheic dermatitis

Sophie brings in her husband, Nathan, age 72, who is in a wheelchair. On his sacral area he has a deep crater with full-thickness skin loss involving necrosis of subcutaneous tissue that extends down to the underlying fascia. Which pressure ulcer stage is this? A. Stage I B. Stage II C. Stage III D. Stage IV

c. Stage III

A mother in her first trimester has a 5 year-old who has Fifth disease. What implication does this have for the mother? a. She does not have to worry about transmission to the fetus b. She may get a mild case of Fifth disease c. There is a risk of fetal death if she becomes infected d. The mother should have a fetal ultrasound today

c. There is a risk of fetal death if she becomes infected

A 68 year-old adult who smokes ask the nurse practitioner about the benefits of quitting "at my age". What should the nurse practitioner reply? a. There is not much benefit because you've smoked for 40 years b. This would help decrease your risk of developing COPD c. This will decrease your risk of all cause mortality after 5 years of stopping d. Your heart will profit the most from quitting smoking

c. This will decrease your risk of all cause mortality after 5 years of stopping

A patient who has been in the sun for past few weeks is very tanned. He has numerous 3-6mm light colored flat lesions on his trunk. What is the likely etiology? a. Tinea corporis b. Tinea unguium c. Tinea versicolor d. Human papilloma virus

c. Tinea versicolor

A female has a urine specimen that is positive for leukocytes and nitrites. There is blood in the specimen. An appropriate diagnosis is: a. urinary tract infection b. asymptomatic bacteriuria c. UTI with hematuria d. UTI or chlamydia

c. UTI with hematuria

An older adult has begun to use over the counter dipenhydramine to help him go to sleep. What serious side effect can occur in older adults with this medication? a. Daytime sleepiness b. Bradycardia c. Urinary retention d. Anti-psychotic behavior

c. Urinary retention

An elderly patient with COPD receives a prescription for a beta agonist. What prescribing considerations must be made in this patient? a. Use this early in the morning b. Use 2 puffs every 8 hours c. Use 1 puff every 4-6 hours d. Rinse your mouth out well after using

c. Use 1 puff every 4-6 hours

A 6 year-old patient with sore throat has coryza, hoarseness, and diarrhea. What is the likely etiology? a. Group A Streptococcus b. H. parainfluenzae c. Viral etiology d. Mycoplasma

c. Viral etiology

A 70 year-old male has a yellowish, triangular nodule on the side of the iris. This is probably: a. a stye b. a chalazion c. a pinguecula d. subconjunctival hemorrhage

c. a pinguecula

The single most effective maintenance therapy for allergic rhinitis is: a. an antihistamine b. a decongestant c. a topical nasal steroid d. a topical antihistamine

c. a topical nasal steroid

The nurse practitioner sees a child who presents with fatigue and purpura on his lower extremities. His temperature is normal. The differential includes: a. anemia b. hypothyroidism c. acute leukemia d. Kawasaki syndrome

c. acute leukemia

Hand-foot-and-mouth disease and herpangina: a. both produce pustules on oral mucous membranes b. potentially can cause orchitis in infected males c. are viral infections caused by Coxsackie viruses d. will only occur in the spring and summer

c. are viral infections caused by Coxsackie viruses

A patient who abuses alcohol will probably exhibit: a. elevated alkaline phosphatase b. decreased TSH c. elevated ALT, AST, and GGT d. elevated AST only

c. elevated ALT, AST, and GGT

A 4 year-old has been diagnosed with measles. The nurse practitioner identifies Koplik's spots. These are: a. spots on the skin that are pathognomonic for measles b. red rings found on the tongue that have a white granular area inside the ring c. found on the inside of the cheek and are granular d. blanchable areas on the trunk and extremities

c. found on the inside of the cheek and are granular

The most common risk factor for developing hepatitis B is: a. homosexual activity b. injecting drug use c. heterosexual activity d. body piercings

c. heterosexual activity

The greatest risk of transmitting HIV is during: a. the acute phase b. the time that detectable antibody is present c. high viral load periods d. late infection phase

c. high viral load periods

Which of the following is the least potent topical corticosteroid? A. betamethasone dipropionate 0.1% (Diprosone) B. clobetasol ppropionate 0.05% (Cormax) C. hydrcorrtisone 2.5% D. flucinnonide 0.05% (Lidex)

c. hydrocortisone 2.5%

Thalassemia minor can be recognized by: a. microcytic, normochromic red cells b. normocytic, normochromic red cells c. microcytic, hypochromic red cells d. normocytic, hyperchromic red cells

c. microcytic, hypochromic red cells

A child has 8-10 medium brown cafe au lait spots > 1 cm in diameter. The differential diagnosis should include: a. multiple sclerosis b. Bell's palsy c. neurofibromatosis d. neuroblastoma

c. neurofibromatosis

A 4 year-old child with otitis media with effusion: a. needs an antibiotic b. probably has a viral infection c. probably has just had acute otitis media d. has cloudy fluid in the middle ear

c. probably has just had acute otitis media

The term caput succedaneum refers to: a. cradle cap b. atopic dermatitis c. scalp edema d. asymmetrical head shape

c. scalp edema

Classic symptoms of a deep vein thrombosis include: a. swelling, pain, redness b. calf complaints, pain with walking, history of exercise c. swelling, pain, and discoloration in lower extremity d. warmth, edema, and relief of pain with walking

c. swelling, pain, and discoloration in lower extremity

A healthcare provider ("the HCP") was stuck with a needle from a patient suspected to be infected with HIV ("the patient"). A rapid HIV test was performed and was found to be positive. This means that the: a. healthcare provider has been infected with HIV b. the patient is infected with HIV c. the HIV status of the patient requires further testing d. the HIV status of the healthcare provider requires further testing

c. the HIV status of the patient requires further testing

The difficulty in treating an older adult with an oral antifungal agent for a fungal infection of the toenail is: a. the absorption of the medication b. the difficulty in applying the medication twice daily c. the tolerability of the medication d. the ineffectiveness of the treatment on older adults

c. the tolerability of the medication

A 40 year-old female patient presents to the clinic with multiple, painful reddened nodules on the anterior surface of both legs. She is concerned. These are probably associated with her history of: a. deep vein thrombosis b. phlebitis c. ulcerative colitis d. alcoholism

c. ulcerative colitis

A characteristic of an ACE inhibitor induced cough is that it: a. is mildly productive b. is worse at nighttime c. usually begins within a week of starting therapy d. is more common in men

c. usually begins within a week of starting therapy

The major difference between varicose veins and arteriosclerosis is the: a. limbs affected b. gender affected c. vessels affected d. degree of pain

c. vessels affected

A 15 year-old male has worked this summer as a lifeguard at a local swimming pool. He complains of itching in the groin area. He is diagnosed with tinea cruris. The nurse practitioner is likely to identify: a. swelling of the scrotum b. macular lesions on the penis c. well marginated half moon macules on the inner thigh d. maceration of the scrotal folds with erythema of the penis

c. well marginated half moon macules on the inner thigh

What advice should be given to a parent who has a child with Fifth disease? a. This commonly causes pruritis in young children b. He can return to school when the rash has disappeared c. Acetaminophen should be avoided in this child d. A parent may experience joint aches and pains

d. A parent may experience joint aches and pains

The throat swab done to identify Streptococcal infection was negative in a 12 year-old female with tonsillar exudate, fever, and sore throat. What statment is true regarding this? a. A second swab should be done to repeat the test b. The patient does not have Strept throat c. The patient probably has mononucleosis d. A second swab should be sent to microbiology

d. A second swab should be sent to microbiology

A patient has been prescrbed metronidazole for treatment of C. difficile. What should be avoided in this patient? a. Excess fluids b. Vitamin B12 c. Grapefuit juice d. Alcohol

d. Alcohol

Which of the following medications does not warrant monitoring of potassium levels? a. Fosinopril b. Candesartan c. Hydrochlorothiazide d. Amlodipine

d. Amlodipine

A Gram stain of which lesion reveals large, square ended, gram positive rods that grow easily on blood agar? A. Dermatophyte infection B. Tuberculosis (scrofuloderma) C. Sarcoidosis D. Anthrax

d. Anthrax

A child is diagnosed with bronchiolitis. Which choice below would NOT be part of patient management? a. Antipyretics b. Nebulized bronchodilators c. Oral steroids d. Antibiotics

d. Antibiotics

A patient has been diagnosed with mononucleosis. Which statement is correct? a. He is likely an adolescent male b. Splenomegaly is more likely than not c. He cannot be co-infected with Strept d. Cervical lymphadenopathy may be prominent

d. Cervical lympphadenopathy may be prominent

A patient has nasal septal erosion with minor bleeding. There is macerated tissue. What is a likely etiology? a. Improper use of a nasal steroid b. Chronic sinusitis c. Severe allergic rhinitis d. Cocaine abuse

d. Cocaine abuse

A patient with allergic rhinitis has developed a sinus infection. He takes fexofenadine daily. In addition to an antibiotic, what should be part of his medication regimen? a. Stop the fexofenadine and add a decongestant b. Add a decongestant and a nasal steroid c. Continue the fexofenadine and antibiotic only d. Continue the fexofenadine and add a decongestant

d. Continue the fexofenadine and add a decongestant

A patient cannot stick his tongue out of his mouth. What cranial nerve is responsible for this movement? a. Cranial Nerve III b. Cranial Nerve VII c. Cranial Nerve X d. Cranial Nerve XII

d. Cranial Nerve XII

A patient has been diagnosed with MRSA. She is sulfa allergic. Which medication could be used to treat her? a. Augmentin b. Trimethoprim-sulfamethoxazole (TMPS) c. Ceftriaxone d. Doxycycline

d. Doxycycline

The nurse practitioner is examining a 3-month old infant who has normal development. She has identified an alopecic area at the occipital area. What should be done? a. Order a TSH b. Order a hydrocortisone cream for once daily application c. Suspect child abuse d. Encourage the caregiver to change the infant's head position

d. Encourage the caregiver to change the infant's head position

A 9 year-old female has presented to your clinic because of a rash on the left, upper area of her anterior trunk. She is embarrassed and very reticent to lift her clouse because her nipple will be exposed. How should the NP proceed? a. Examine the patient with the area covered as much as possible b. The NP should lift the blouse to expose the area for exam c. Ask the accompanying caregiver to expose the area for exam d. Examine all other areas of the trunk, then ask the child to lift her blouse

d. Examine all other areas of the trunk, then ask the child to lift her blouse

AV nicking is identified in a patient with what disease? a. Glaucoma b. Cataracts c. Diabetes d. Hypertension

d. Hypertension

A three year-old presents with hematuria, petechia, and a platelet count of 50,000 (Normal = 150,000-400,000/ml). The rest of his CBC is normal. He had an upper respiratory infection about 2 weeks ago. On exam today, he is found to have petechiae and bruises. The most likely diagnosis is: a. polycythemia vera b. acute lymphocytic leukemia (ALL) c. Von Willebrand's disease d. Idiopathic thrombocytopenia purpura (ITP)

d. Idiopathic thrombocytopenia purpura (ITP) -low platelets is thrombocytopenia. ITP is most common type in children b/t 2-4yrs old.

Which statement is true about vitamin B12? a. It is easily absorbed through the gastrointestinal tract b. Deficiencies are seen in elderly patients only c. Low levels can result in elevated lipid levels d. Inadequate amounts can produce cognitive changes

d. Inadequate amounts can produce cognitive changes

A patient is diagnosed with mild chronic heart failure (CHF). What drug listed below would be a good choice for managing his symptoms and improving long-term outcomes? a. Verapamil b. Digoxin c. Furosemide d. Monopril

d. Monopril

A 63 year-old male has been your patient for several years. He is a former smoker who takes simvastatin, ramipril, and an aspirin daily. His blood pressure and lipids are well controlled. He presents to your clinic with complaints of fatigue and "just not feeling well" for the last few days. His vital signs and exam are normal. His CBC, TSH, urine analysis are normal. His liver enzymes are six times the upper limits of normal. What should be done next? a. Order a hepatitis panel and stop his medications b. Refer to gastroenterology c. Refer for a toxicology evaluation d. Order a hepatitis panel and stop his lovastatin and aspirin

d. Order a hepatitis panel and stop his lovastatin and aspirin

Which of the following symptoms is typical of GERD? a. Chest pain b. Hoarseness c. Sore throat d. Pyrosis

d. Pyrosis

A patient reports to the minor care area of the emergency department after being bitten by a dog. The patient states that the dog had a tag around his neck and had been seen roaming around the neighborhood. The dog did not exhibit any odd behavior. How should this be managed? a. If the bites are only minor, do not mention rabies prophylaxis to the patient b. Give the patient tetanus immunization only. Don't call animal control. c. Clean the wounds, provide tetanus and rabies prophylaxis d. Report the bite to animal control and administer appropriate medical care

d. Report the bite to animal control and administer appropriate medical care

A 10 year-old has thick, demarcated plaques on her elbows. Which features are suggestive of psoriasis? a. Scaly lesions on the scalp b. Pruritis around the lesions c. A scale border around the plaques d. Silvery scales that are not pruritic

d. Silvery scales that are not pruritic

A patient with pneumonia reports that he has rust colored sputum. What pathogen should the nurse practitioner suspect? a. Mycoplasma pneumoniae b. Chlamydophila pneumoniae c. Staphylococcus aurues d. Streptococcus pneumoniae

d. Streptococcus pneumoniae

A patient presents to your clinic with a painless red eye. Her vision is normal, but her sclera has a blood red area. What is this termed? a. Conjunctivitis b. Acute iritis c. Glaucoma d. Subconjuctival hemorrhage

d. Subconjunctival hemorrhage

A 4 month-old infant has thrush. The mother is breastfeeding. She reports that her nipples have become red, irritated, and sensitive. What should the nurse practitioner advise the mother of this baby to treat thrush? a. Have the mother exercise good hygiene of her nipples b. Administer an oral antifungal suspension to the mother c. Administer an oral antifungal suspension to the infant d. Treat the infant with an oral antifungal suspension, and the mother's nipples with a topical antifungal agent

d. Treat the infant with an oral antifungal suspension, and the mother's nipples with a topical antifungal agent

A female patient who takes oral contraceptives has just completed her morning exercise routine. She complains of pain in her right calf. Her blood pressure and heart rate are normal. She is not short of breath. Her calf is red and warm to touch. What is NOT part of the differential diagnosis? a. Deep vein thrombosis b. Cellulitis c. Calf muscle strain d. Trochanteric bursitis

d. Trochanteric bursitis

Mycoplasma pneumonia is: a. a diagnosis of exclusion b. a typical respiratory pathogen c. only identifiable on chest x-ray d. a disease with extrapulmonary manifestations

d. a disease with extrapulmonary manifestations

Topical 5-fluorouracil (5-FU) is used to treat: a. atopic dermatitis b. hepatitis c. thalassemia d. basal cell carcinoma

d. basal cell carcinoma

A B12 deficiency can produce: a. a microcytic anemia b. pernicious anemia c. sideroblastic anemia d. insomnia

d. insomnia

An elderly male diagnosed with a microcytic, hypochromic anemia: a. should be worked up for a malignancy b. could consume excess amounts of alcohol c. will have a increased RBC count d. may have a GI bleed

d. may have a GI bleed

A 12 month-old was screened for iron deficiency anemia and found to be anemic. The nurse practitioner ordered oral iron. In one month the child's hemoglobin was re-assessed. It increased only minimally. This may be due to many reasons, but NOT because: a. the mother administered it with his bottle of milk b. the child had an insufficient dosage c. the child has leukemia d. of his diet

d. of his diet

A 4 year-old presents with fever, rhinorrhea, and a high ptiched whooping cough. This is: a. bronchiolitis b. croup c. foreign body aspiration d. pertussis

d. pertussis

The most common cause of acute pharyngitis in children is: a. S. pyogenes b. H. influenzae c. M. pneumoniae d. respiratory viruses

d. respiratory viruses

A patient who is 60 years old complains of low back pain for the last 5-6 weeks. She states that the severity is about 4/10 and that she gets no relief from sitting, standing, or lying. The NP should consider: a. sciatica b. ankylosing spondylitis c. disc disease d. systemic illness

d. systemic illness

A treatment considered first-line for a patient with allergic rhinitis is a: a. decongestant b. sedating antihistamine c. a leukotriene blocker d. topical nasal steroid

d. topical nasal steroid

The most common cause of pneumonia in very young children is: a. S. pneumonia b. S. aureus c. mycoplasma d. viruses

d. viruses

Match the following: 1. First degree burn 2. Second-degree burn 3. Third-degree burn A. affected skin blanches easily B. Surface is raw and moist C. Affected area is white and leathery

1 - A 2 - B 3 - C

Janice states that her son is allergic to eggs and she heard that he should not receive the flu vaccine. How do you respond? A. "Although measles, mumps, rubella, and influenza vaccines contain a minute amount of egg, most egg-allergic individuals can tolerate these vaccines without any problems." B. "Most of the allergic reactions are caused by the actual vaccinations; therefore, a skin test should be done first." C. You're right. We should not give this vaccination to your son." D. "He should not have a skin test done if he has this allergy because a serious cellulitis may occur at the site of testing."

A. "Although measles, mumps, rubella, and influenza vaccines contain a minute amount of egg, most egg-allergic individuals can tolerate these vaccines without any problems."

Jill's son, Nathan, has asthma, and she cannot decide whether he should play Little League baseball or not. She asks for your advice. What do you tell her? A. "Exercise should be encouraged rather than restricted" B. "Sports are recommended unless exercise-induced bronchospasm occurs" C. "He should find some other type of recreational activity" D. "If he has a nebulization treatment before playing, he should do fine"

A. "Exercise should be encouraged rather than restricted"

The most common human papillomavirus types associated with cutaneous, nongenital warts include: A. 1,2, and 4 B. 6 and 11 C. 16 and 18 D. 32 and 36

A. 1,2, and 4

Which of the following clinical findings constitutes a possible emergency situation? A. A fiery red epiglottis B. Sudden onset of hoarseness C. Purulent drainage from the external canal D. Tragal tenderness

A. A fiery red epiglottis - A finding of a fiery red epiglottis signals epiglottitis. Since airway obstruction can be rapid with epiglottitis, immediate referral to the emergency room is warranted.

In a young child, unilateral purulent rhinitis is most often caused by A. A foreign body B. A viral infection C. A bacterial infection D. An allergic reaction

A. A foreign body

Silas, age 82, comes to your office with a failry new colostomy. Around the stoma he has a papular rash with satellite lesions. What does this indicate? A. A fungal infection, usually candida albicans B. An allergic reaction to the appliance C. A normal reaction to fecal drainage D. A fluid volume deficit

A. A fungal infection, usually Candida albicans

Sara comes today with numerous petechiae on her arms. You know that she is not taking warfarin (Coumadin). What other drugs do you ask her about? A. Aspirin or aspirin compounds B. Antihypertensive agents C. Oral contraceptives D. Anticonvulsants

A. Aspirin or aspirin compounds

Which of the following conditions is characterized by intermittent episodes of airway obstruction caused by bronchospasm, excessive bronchial secretion, or edema of bronchial mucosa? A. Asthma B. Atelectasis C. Acute bronchitis D. Emphysema

A. Asthma

Jennifer, age 32, has genital warts (condylomata) and would like to have them treated. All of the following could be applied except A. Benzoyl peroxide B. Podophyllin C. Trichloroacetic acid D. Liquid nitorgen

A. Benzoyl peroxide - Used for acne

Zinc oxide, magnesium silicate, ferric chloride, and kaolin are examples of A. Chemical sunscreens B. Physical sunscreens C. Agents used in tanning booths D. Emollients

A. Chemical sunscreens

A pulmonary function test, such as spirometry, is helpful in the diagnosis of A. Chronic bronchitis B. Lung cancer C. Pneumonia D. Tuberculosis

A. Chronic bronchitis - A pulmonary function test, such as spirometry, is helpful in the diagnosis of restrictive lung disease and obstructive lung disease, including chronic bronchitis and asthma.

What is a safe and effective treatment for psoriasis? A. Coal tar preparations B. Topical steroids C. Topical antibiotics D. Systemic antihistamines

A. Coal tar preparations

A 60 year old patient is noted to have clubbing of the fingers. What might this indicate? A. Coronary artery disease B. Cirrhosis C. Lead toxicity D. Iron deficiency anemia

A. Coronary artery disease Seen with chronic hypoxia as in cigarrette smokers, COPD, lung cancer, and Coronary artery disease.

Which medication below would be contraindicated in a patient with COPD? A. Cough suppressant with codeine B. Cough suppressant with dextromethorphan C. Mucolytics D. Anti-histamine/decongestants

A. Cough suppressant with codeine - Codeine would be contraindicated (or used with extreme caution) because the sedative effect of codeine or any narcotic can potentially worsen respiratory depression and worsen hypercapnia.

Which of the following is a predisposing condition for furunculosis? A. Diabetes mellitus B. Hypertension C. Peripheral vascular disease D. Chronic fatigue syndrome

A. Diabetes mellitus

Microtia refers to the size of the A. Ears B. Skull C. Pupils D. Eyes

A. Ears

Which structure fo the skin is responsible for storing melanin? A. Epidermis B. Dermis C. Sebaceous glands D. Eccrine sweat glands

A. Epidermis

Which test differentiates iron-deficiency anemia from the anemia of chronic disease in clients with normal or low mean corpuscular values? A. Ferritin B. Total iron-binding capacity C. Folate D. Erythrocyte sedimentation rate

A. Ferritin

The symptom triad most commonly associated with infectiouse mononucleosis is: A. Fever, pharyngitis, lymphadenopathy B. Fatigue, pharyngitis, fever C. Splenomegaly, fever, body aches D. Tonsillar exudates, lymphadenopathy, headache

A. Fever, pharyngitis, lymphadenopathy - Fatigue commonly accompanies mono, but this is not part of the triad.

Jack, age 59, has a nevus on his shoulder that has recently changed from brown to bluish black. You advise him to A. Have an excisional biopsy B. Monitor the nevus for a change at the end of 1 month C. Apply benzoyl peroxide solution D. Apply hydrocortisone cream 1%

A. Have an excisional biopsy

Coughing up blood or sputum that is streaked or tinged with blood is known as A. Hemoptysis B. Regurgitation C. Bloody sputum D. Rhinorrhea

A. Hemoptysis

Which is the most abundant immunoglobulin (Ig) found in the blood, lymph, and intestines? A. IgG B. IgA C. IgM D. IgD

A. IgG - IgG is active against bacteria, bacterial toxins, and viruses

When the donor and recipient of a transplant are identical twins, this is referred to as a(n) A. Isograft B. Autograft C. Allograft D. Xenograft

A. Isograft

Which of the following is a benign neoplasm? A. Leiomyoma B. Osteosarcoma C. Glioma D. Seminoma

A. Leiomyoma - Benign neoplasm of the smooth muscle

A 70 year old client with herpes zoster has a vesicle on the tip of the nose. This may indicate A. Ophthalmic zoster B. Herpes simplex C. Kaposi's sarcoma D. Orf and milker's nodules

A. Ophthalmic zoster

Mr. Marks, age 54, has COPD. He has recenty been experiencing difficulty in breathing. His arterial blood gas screening reveals pH 7.3, PaO2 57 Hg, PaCO2 54mm Hg, and oxygen saturation 84%. Mr. Marks has: A. Respiratory acidosis B. Respiratory alkalosis C. Metabolic acidosis D. Metabolic alkalosis

A. Respiratory acidosis - Respiratory acidosis results when the serum PaCO2 exceeds 45mm Hg and the serum pH is less than 7.35

The viral exanthem of Koplik's spots is present in A. Rubeola B. Rubella C. Fifth disease D. Varicella

A. Rubeola - Koplik's spots are observed on the buccal mucosa before the rash appears.

Permethrin (Elimite) applied over th body overnight from the neck down is the preferred treatment for A. Scabies B. Eczema C. Herpes simplex D. Psoriasis

A. Scabies

Your client Sally has had significant diarrhea for several days. What laboratory values would you expect to be increased? A. Serum chloride B. Serum potassium C. Serum sodium D. Serum bicarbonate

A. Serum chloride

Sidney, age 72, has just been diagnosed with temporal arteritis. What do you prescribe? A. Systemic corticosteroids B. Topical corticosteroids C. Antibiotics D. Antifungal preparations

A. Systemic corticosteroids - Treatment for temporal arteritis involves systemic corticosteroids and immunosuppressives. The ESR is frequently elevated and a biopsy reveals granulomas and giant cells.

Early descriptions of this disease were centered on the cutaneous manifestations. In fact, the word means "wolf" and was used to describe the destructive eating away of the skin that was a result of the rash. Which disease is this? A. Systemic lupus erythematosus B. Herpes Zoster (shingles) C. Malaria D. Rubeola

A. Systemic lupus erythematosus

What would the tumor node metastasis (TNM) classification be for a lung tumor that had carcinoma in situ, metastasis to the lymph nodes in the peribronchial or the ipsilateral hilar region, and distant metastasis to the spine? A. T1SN1M1 B. T2N0M0 C. T3N1M1 D. T0N1M0

A. T1SN1M1 - T is for the relative tumor size, N indicates the presence and extent of lymph node involvement, and M denotes distant metastases.

Which statement about chronic obstructive pulmonary disease (COPD) is true? A. The prevalence of COPD is directly related to increasing age. B. The incidence of COPD is about equal in men and in women C. Cigar or pipe smoking does not increase the risk of developing COPD. D. Environmental factors such as smoke do not affect the potential for COPD.

A. The prevalence of COPD is directly related to increasing age. - Men are affected much more often than women because the percentage of men who smoke is greater than that of women.

Dry, itchy skin in older adults results from A. The reduction of sweat and oil glands B. Loss of subcutaneous tissue C. Dermal thinning D. Decreased elasticity

A. The reduction of sweat and oil glands

Johnnie, age 52, presents with pruritus with no rash present. He has hypertension, diabetes, and ESRD. One of the differential diagnoses would be A. Uremia from chronic renal disease B. Contact dermatitis C. Lichen planus D. Psoriasis

A. Uremia from chronic renal disease

June, age 50, presents with soft, raised, yellow plaques on her eyelids at the inner canthus. She is concerned that they may be cancerous skin lesions. You tell her they are probably A. Xanthelasmas B. Pingueculae C. The result of arcus senilis D. Actinic keratoses

A. Xanthelasmas

A 22-year-old woman presents with a "bump" on her right eyelid. Examination reveals a 2-mm hard, non-tender swelling on the lateral border of the right eyelid margin. This is most consistent with: A. a chalazion B. a hordeolum C. blepharitis D. cellulitis

A. a chalazion

Which of the following is a first-line therapy for the treatment of ABRS in an adult with no recent antimicrobial use? A. amoxicillin B. trimethoprim-sulfamethoxazole C. clarithromycin D. levofloxacin

A. amoxicillin

The most commom cause of a chronic cough that is more severe early in the morning and produces yellowish-brown mucus is A. cigarette smoking B. asthma C. bronchogenic carcinoma D. angiotensin-converting enzyme (ACE) use

A. cigarette smoking

The mechanism of action of imiquimod is as a/an: A. immunomodulator B. antimitotic C. keraatolyttic D. irritant

A. immunomodulator

Treatment options in postherpetic neuralgia include all of the following except: A. methylprednisolone b. pregabalin C. nortriptyline D. topical lidocaine

A. methylprednisolone

A 19-year-old man presents with a chief complaint of a red, irritated right eye for the past 48 hours with eyelids that were "stuck together" this morning when he awoke. Examination reveals injected palpebral and bulbar conjunctiva; reactive pupils; vision screen with the Snellen chart of 20/30 in the right eye (OD), left eye (OS), and both eyes (OU); and purulent eye discharge on the right. This presentation is most consistent with: A. suppurative conjunctivitis B. viral conjunctivitis C. allergic conjunctivitis D. mecahnical injury

A. suppurative conjunctivitis

Treatment of otitis media with effusion usually includes: A. symptomatic treatment B. antimicrobial therapy C. an antihistamine D. a mucolytic

A. symptomatic treatment

Therapeutic interventions for the patient in Question 1 (giant cell arteritis), should include: A. systemic corticosteroid therapy for many months B. addition of an angiotensin-converting enzyme inhibitor (ACEI) to her antihypertensive regimen C. warfarin therapy D. initiation of topirimate (Topamax)

A. systemic corticosteroid therapy for many months

Sidney, age 72, has just been diagnosed with temporal arteritis. What do you prescribe? A. Systemic corticosteroids B. Topical corticosteroids C. Antibiotics D. Antifungal preparations

A. systemic corticosteroids

When prescribing itraconazole (Sporanox), the NP considers that: A. the drug is a cytochrome P-450 3A4 inhibitor B. one pulse cycle is recommended for fingernail treatment and two cycles are needed for toenail therapy C. continuous therapy is preerred in the presence of hepatic disease D. taking the drug on an empty stomach enhances the efficacy of the product

A. the drug is a cytochrome P-450 3A4 inhibitor

A risk factor for malignant external otitis includes: A. the presence of an immunocompromised condition B. age younger than 21 years C. a history of a recent upper respiratory tract infection D. a complicated course of otitis media with effusion

A. the presence of an immunocompromised condition

According to the Global Resources in Allergy (GLORIA) guidelines, which of the following is recommended for intervention in persistent allergic conjunctivitis? A. topical mast cell stabilizer with a topical antihistamine B. ocular decongestant C. topical nonsteroidal anti-inflammatory drug D. topical corticosteroid

A. topical mast cell stabilizer with a topical antihistamine

Which of the following findings is most consistent with the diagnosis of acute bacterial rhinosinusitis (ABRS)? A. upper respiratory tract infections persisting beyond 7 to 10 days or worsening after 5 to 7 days B. mild midfacial fullness and tenderness C. preauricular lymphadenopathy D. marked eyelid edema

A. upper respiratory tract infections persisting beyond 7 to 10 days or worsening after 5 to 7 days

In counseling a patient with seborrheic dermatitits on the scalp about efforts to clear lesions, you advise her to: A. use ketoconzazole shampoo B. apply petroleum jelly nightly to the affected area C. coat the area with high-potency corticosteroid cream three times a week D. expose the lesions periodically to sunlight

A. use ketoconazole shampoo

Decongestants work primarily through: A. vasoconstriction B. action on the H1 receptor sites C. inflammatory mediation D. peripheral vasodilation

A. vasoconstriction

An urticarial lesion is usually descriibed a: A. wheal B. plaque C. patch D. papule

A. wheal

Anne has a history of hyperlipidemia, and today you note a soft, yellowish, raised, waxy lesion beneath her right eyelid. You diagnose this as a A. xanthelasma B. hordeolum C. chalazion D. lagophthalmos

A. xanthelasma

An oral antimicrobial option for the treatment of methicillin-sensitive S. aureus includes all the following except: A. amoxicillin B. dicloxacillin C. cephalexin D. cefadroxil

Amoxicillin

Kathy, age 64, is a sun worshipper. She tells you that because she did not get skin cancer in her youth, she certainly will not get it now. How do you respond? A. "You're probably right; if you haven't had it by now, you're probably safe." B. "As you age, you have decreased pigment in your skin, which puts you at more risk." C. "You skin elasticity is decreased, so you have more of a chance of contracting skin cancer as you age." D. "Skin cancer is not dependent on age; anyone can get it."

B. "As you age, you have decreased pigment in your skin, which puts you at more risk."

Amy, age 36, is planning to go skiing with her fiance. He has warned her about frostbite and she is wondering what to do if frostbite should occur. You know she's misunderstood the directions when she tells you which of the following? A. "I should remove wet footwear if my feet are frostbitten." B. "I should rub the area with snow." C. "I should apply firm pressure with a warm hand to the area." D. "I should place my hands in my axillae if my hands are frostbitten."

B. "I should rub the area with snow."

Susie, age 6 months, has a candida infection in the diaper area. What do you suggest to the mother? A. "Use rubber or plastic pants to contain the infection and prevent it from getting to the thighs." B. "Keep the area as dry as possible." C. "Use baby powder with cornstarch." D. "Keep Susie away from other babies until the infection is cleared up."

B. "Keep the area as dry as possible."

Mavis is 70 years old and wonders if she can donate her corneas when she dies. How do you respond? A. "As long as you don't have any chronic illness, you corneas may be harvested." B. "They will use corneas only from persons younger than age 65." C. "What makes you feel like you are dying?" D. "Don't think about such terrible things now."

B. "They will use corneas only from persons younger than age 65."

The most common reason for a chronic cough in children is A. Asthma B. A postinfection C. Postnasal drip D. An irritant

B. A postinfection

The antibiotic of choice for beta-lactamase coverage of otitis media is A. Amoxicillin (Amoxil) B. Amoxicillin and potassium clavulanate (Augmentin) C. Azithromycin (Zithromax) D. Prednisone (Deltasone)

B. Amoxicillin and potassium clavulanate (Augmentin)

Which of the following workers is at risk for developing pneumoconiosis? A. Farmers B. Coal miners C. Construction workers D. Potters

B. Coal miners

When palpating the skin over the clavicle of James, age 84, you notice tenting, which is A. Indicative of dehydration B. Common in thin older adults C. A sign of edema D. Indicative of scleroderma

B. Common in thin older adults

A patient was recently hospitalized for chronic heart failure. Since discharge he has complained of difficulty hearing. Which of the following medications is most likely responsible? A. Digoxin B. Furosemide C. Ramipril D. Metoprolol

B. Furosemide - Many medications are ototoxic. Hearing loss secondary to use of the following medications should always be assessed: aminoglycosides, vancomycin, erythromycin, loop diuretics (like furosemide), the antimalarials, and cisplatin.

Which of the following is a genotoxic carcinogen? A. Vinyl chloride polymers B. Chemotherapy drugs C. Asbestos D. Wood and leather dust

B. Genotoxic carcinogens directly alter DNA and cause mutations

Which of the following is an X-linked recessive disorder commonly seen in African American men? A. Sickle cell anemia B. Glucose-6-phosphate dehydrogenase deficiency C. Pyruvate kinase deficiency D. Bernard-Soulier syndrome

B. Glucose-6-phosphate dehydrogenase deficiency - It is an enzyme defect that causes episodic hemolytic anemia because of the decreased ability of red blood cells to deal with oxidative stresses.

Which of the following symptom(s) is (are) most indicative of mononucleosis (Epstein-Barr virus)? A. Rapid onset of anterior cervical adenopathy, fatigue, malaise, and headache B. Gradual onset of fatigue, posterior cervical adenopathy, fever and sore throat C. Gradual and seasonal onset of pharyngeal erythema D. Rapid onset of cough, congestion and headache

B. Gradual onset of fatigue, posterior cervical adenopathy, fever and sore throat

Tee, age 64, presents with a sore throat. Your assessment reveals tonsillar exudate, anterior cervical adenopathy, presence of a fever, and absence of a cough. There is a high probability of which causative agent? A. Haemophilus influenzae B. Group A beta-hemolytic streptococcus C. Epstein-Barr virus D. Rhinovirus

B. Group A beta-hemolytic streptococcus - When the four symptoms present as a cluster, there is a high probability that the infection is caused by group A beta-hemolytic streptococcus

Purulent matter in the anterior chamber of the eye is called A. Hyphema B. Hypopyon C. Anisocoria D. Pterygium

B. Hypopyon

Increased tactile fremitus occurs with A. Pleural effusion B. Lobar pneumonia C. Pneumothorax D. Emphysema

B. Lobar pneumonia - Occurs with compression or consolidation of lung tissue, such as occurs in conditions like lobar pneumonia

A skin lesion which is a solid mass is described as a: A. Macule B. Papule C. Vesicle D. Bullae

B. Papule

A common infective agent in domestic pet cat bites is: A. viridans streptococcus species B. Pasteurella multocida C. Bacteroides species D. Haemophilus influenzae

B. Pasteurella multocida -80% get infected

What condition occurs in almost all persons beginning around age 42-46? A. Arcus senilis B. Presbyopia C. Cataracts D. Glaucoma

B. Presbyopia

You suspect that a pregnant client with a diagnosis of anemia practices pica. What makes you suspect this? A. She smokes when no one is looking B. She keeps boxes of starch under her bed C. Her breath smells like alcohol D. There are many empty vitamin bottles in the trash

B. She keeps boxes of starch under her bed

Samantha, age 52, has an acrochordon on her neck. She refers to this as a A. Nevus B. Skin tag C. Lipoma D. Wart

B. Skin tag

Marty has a hordeolum in his right eye. You suspect that the offending organism is A. Herpes simplex virus B. Staphylococcus C. Candida albicans D. Escherichia coli

B. Staphylococcus

Cough and congestion result when breathing A. Carbon monoxide B. Sulfur dioxide C. Tear gas D. Carbon dioxide

B. Sulfur dioxide - Carbon monoxide produces dizziness, headache and fatigue. Tear gas irritates the conjunctiva and produces a flow of tears. Carbon dioxide produces sleepiness.

With a chronic allergy, a client's nasal mucosa appear A. Swollen and red B. Swollen, boggy, pale and gray C. Hard, pale, and inflamed D. Bright pink and inflamed

B. Swollen, boggy, pale and gray

A patient with psoriasis has used a topical hydrocortisone cream for years to treat exacerbations when they occur. She presents today and states that this cream "just doesn't work anymore." What word describes this? A. Rebound effect B. Tachyphylaxis C. Tolerance D. Lichenification

B. Tachyphylaxis - Tachyphylaxis is the word used to describe a gradual and progressively poorer clinical response to a treatment or medication. Particularly true of topical glucocorticoids, bronchodilators, nitroglycerine, & antihistamines (overused).

A common site for atopic dermatitits in an infant is on the: A. the diaper area B. the face C. the neck D. the psosterior trunk

B. The face

Your 17 year-old client Hillary has infectious mononucleosis. You expect her blood work to reflect the following: A. Decreased serum globulins B. Thrombocytopenia and elevated transaminase C. High potassium and blood urea nitrogen (BUN) D. Elevated serum protein

B. Thrombocytopenia and elevated transaminase

Sandra, age 69, is complaining of dry skin. What do you advise her to do? A. Bathe every day B. Use tepid water and a mild cleansing cream C. Use a dehumidifier D. Decrease the oral intake of fluids

B. Use tepid water and a mild cleansing cream

How should Tommy, age 2 1/2, have his vision screened? A. Using a Snellen letter chart B. Using the Allen test C. Using a Snellen E chart D. Using a Rosebaum chart

B. Using the Allen test

Sally, age 49, has had asthma for several years but has never used a peak flow meter. Should you now recommend it? A. No, she has been managing fine without it. B. Yes, she might recognize early signs of deterioration. C. Present the option and let Sally decide. D. No, at her age it is not recommended.

B. Yes, she might recognize early signs of deterioration. - Daily peak flow monitoring has long been recommended for clients with asthma.

A linear arrangement along a nerve distribution is a description of which type of skin lesion? A. Annular B. Zosteriform C. Keratotic D. Linear

B. Zosteriform

Jennifer, age 36, has systemic lupus erythematosus. She exhibits erythematous raised patches with adherent keratotic scaling and follicular plugging. This is characteristic of A. a malar rash B. a discoid rash C. photosensitivity D. an oral ulcer

B. a discoid rash

What is the connection between the surface of the skin and an underlying structure called? A. An ulcer B. A sinus C. An erosion D. An abscess

B. a sinus

A 64 year old man with seborrhea mentions that his skin condition is better in the summer and worse in the winter. You respond: A. sun exposure is a recommended therapy for the treatment of this condition. B. although sun exposure is noted to improve the skin lesions associated with seborrhea, its use as a therapy is potentially associated with an increased rate of skin cancer C. the lower humidity in the summer months noted in many areas of North America contributes to the improvement of seborrheic lesions D. he should use high potency topical corticosteroids during the winter months, tapering these off for the summer months

B. although sun exposure is noted to improve the skin lesions associated with seborrhea, its use as a therapy is potentially associated with an increased rate of skin cancer

In the treatment of allergic rhinitis, leukotriene modifiers should be used as: A. an agent to relieve nasal itch B. an inflammatory inhibitor C. a rescue drug D. an intervention in acute inflammation

B. an inflammatory inhibitor

A patient with asthma is using his "rescue" medication once daily. How should this be managed? He must receive a prescription for: a. a bronchodilator b. an inhaled steroid c. a long-acting beta agonist d. a leukotriene blocker

B. an inhaled steroid

With regard to pharyngitis caused by group C streptococci, the NP considers that: A. potential complications include glomerulonephritis B. appropriate antimicrobial therapy helps to facilitate more rapid resolution of symptoms C. infection with these organisms carries a significant risk of subsequent rheumatic fever D. acute infectious hepatitis can occur if not treated with an appropriate antimicrobial

B. appropriate antimicrobial therapy helps to facilitate more rapid resolution of symptoms

Which of the following is consistent with the visual problems associated with macular degeneration? A. peripheral vision loss B. blurring of near vision C. difficulty with distant vision D. loss of the central vision field

B. blurring of near vision

A 25-year-old woman has a 3-day-history of left ear pain that began after 1 week of URI symptoms. On physical examination, you find that she has AOM. She is allergic to penicillin (use results in a hive-form reaction). The most appropriate antimicrobial option for this patient is: A. ciprofloxacin B. clarithromycin C. amoxicillin D. cephalexin

B. clarithromycin (Biaxin)

When advising the patient about scabies contagion, you inform her that: A. mites can live for many weeks away from the host B. close personal contact with an infected person is usually needed to contract this disease C. casual contact with an infected person is likely to result in infestation D. bedding used by an infected person must be destroyed

B. close personal contact with an infected person is usually needed to contract this disease

You have initiated therap for an 18 year old man with acne vulgaris and have prescribed tetracycine. He returns in 3 weeks complaining that his skin is "no better." Your next action is to: A. counsel him that 6 to 8 weeks of treatment is often needed before significant improvement is achieved B. discontinue the tetracycline and initiate minocycline therapy C. advise him that antibiotics are likely not an effective treatment for him and should not be continued D. add a second antimicrobial agent

B. counsel him that 6 to 8 weeks of treatment is often needed before significant improvement is achieved

When caring for an adult with an outbreak of shingles, you advise that: A. there is no known treatment for this condition. B. during outbreaks, the chickenpox virus is shed C. although they are acutely painful, the lesions heal well without scarring or lingering discomfort D. this condition commonly strikes young and old alike

B. during outbreaks, the chickenpox virus is shed

Which of the following medications commonly prescribed for tuberculosis cannot be taken by pregnant women? A. Isoniazid (INH) B. Rifampin (RIF) C. Pyrazinamide (PZA) D. Ethambutol (EMB)

C Pyrazinamide (PZA) - Pyrazinamide should not be taken by pregnant women. Active tuberculosis should be treated with isoniazid and ethambutol and continued for at least 18 months to prevent relapse..

Mycostatin (Nystatin) is ordered for Michael, who has an oral fungal infection. What instructions do you give Michael for taking the medication? A. "Don't swallow the medication because it's irritating to the gastric mucosa." B. "Take the medication with meals so that it's better absorbed." C. "Swish and swallow the medication." D. "Apply the medication only to the lesions."

C. "Swish and swallow the medication."

A child's central visual acuity is 20/30 by age A. 18 months B. 2 years C. 3 years D. 4 years

C. 3 years

The leading cause of blindness in persons age 20-60 in the United States is A. Macular degeneration B. Glaucoma C. Diabetic retinopathy D. Trauma

C. Diabetic retinopathy

How would you grade tonsils that touch the uvula? A. Grade 1+ B. Grade 2+ C. Grade 3+ D. Grade 4+

C. Grade 3+

Tonsils that touch the uvula are graded in which way? A. Grade 1+ B. Grade 2+ C. Grade 3+ D. Grade 4+

C. Grade 3+

Which hypersensitivity reaction results in a skin test that is erythematous with edema within 3-8 hours? A. Anaphylactic reaction B. Cytotoxic reaction C. Immune complex-mediated reaction D. Delayed hypersensitivity reaction

C. Immune complex-mediated reaction - May result from serum sickness, systemic lupus erythematosus, or rheumatoid arthritis

How often should you order a complete blood count for your client? A. Routinely B. Before dental work C. In the case of infection D. If she is pregnant

C. In the case of infection

Samantha, age 12, appears with ear pain. When you begin to assess her ear, you tug on her normal appearing auricle, eliciting severe pain. This leads you to suspect A. Otitis media B. Otitis media with effusion C. Otitis externa D. Primary otalgia

C. Otitis externa

When you examine the tympanic membrane, which of these structures is visible? A. Stapes B. Cochlea C. Pars flaccida D. Round window

C. Pars flaccida

What is the meaning of the term "shift to the left" or "left shift"? A. This indicates a rise in basophils B. This indicates a rise in monocytes C. This indicates a rise in neutrophils D. This indicates a rise in lymphocytes

C. This indicates a rise in neutrophils - An elevated WBC count and a relative increase in segmented and band neutrophils. Usually seen in acute bacterial infections, it indicates clinically that the body is responding to an acute need before the neutrophils can fully mature in the bone marrow.

A patient reports a penicillin allergy. What question regarding the allergy should the nurse practitioner ask to determine whether a cephalosporin should be prescribed? A. Have you ever taken a cephalosporin? B. How long ago was the reaction? C. What kind of reaction did you have? D. What form of penicillin did you take?

C. What kind of reaction did you have? - About 2-10% of patients who are penicillin allergic are also cephalosporin allergic. This class of medication should not be excluded in penicillin allergic patients unless they report an anaphylactic reaction.

You are teaching Shawna, age 14 with asthma, to use a home peak expiratory flow meter daily to measure gross changes in peak expiratory flow. Which "zone" would rate her expiratory compliance as 50%-80% of her personal best? A. White zone B. Green zone C. Yellow zone D. Red zone

C. Yellow zone - Green zone - 80%-100% of personal best. No asthma symptoms, continue normal medication regimen - Yellow zone - 50%-80% of personal best. Asthma symptoms may be starting and signals caution - Red zone - < 50% of personal best. An asthma attack is occurring and she should take her inhaled beta-2 agonist and repeat peak flow.

You prescribe nasal corticosteroid spray for a patient with allergic rhinitis. What is the anticipated onset of symptom relief with its use? A. immediately with the first spray B. 1 to 2 days C. a few days to a week D. 2 or more weeks

C. a few days to a week

Which of the following is absent in otitis media with effusion? A. fluid in the middle ear B. otalgia C. fever D. itch

C. fever

A benign heart murmur, previously undocumented and discovered after an episode of pharyngitis, may be a clue to the diagnosis of A. scarlet fever B. Reye's syndrome C. rheumatic fever D. diiphtheria

C. rheumatic fever

The most common trigger of rosacea is A. cold weather B. skin care products C. sun exposure D. dust

C. sun exposure

Your client Mr. Lane, who has COPD, reports a worsening of his respiratory symptoms. You review his entire medication list, noting that he is taking a beta blocker for his hypertension and nitrates for his angina in addition to the theophylline that he takes daily for his COPD. You decide to A. switch him to a different bronchodilator B. discontinue the beta blocker C. switch his beta blocker to a calcium channel blocker D. discontinue his nitrates

C. switch his beta blocker to a calcium channel blocker

First-line therapy for angular chelitis therapy includes the use of: A. metronidazole gel B. hydrocortisone cream C. topical nystatin D. oral ketoconazole

C. topical nystatin

Prophylaxis for the first episode of Pneumocystis jiroveci pneumonia, an oppurtunistic disease in HIV-infected adults and adolescents, is A. rifampin (Rimactane) 600 mg PO qd B. isoniazid (Nitrazid) 300 mg PO plus pyridoxine (Beesix) 50 mg PO qd C. trimethoprim-sulfamethoxazole (TMP-SMZ) (Bactrim) 1 DS PO qd D. clarithromycin (Biaxin) 500 mg PO bid

C. trimethoprim-sulfamethoxazole (TMP-SMZ) (Bactrim) 1 DS PO qd

A 17 year old with nodulocystic acne is employed as a cook at a local establishment. What might have the greatest positive impact in managing his acne? A. Retin-A plus minocycline B. Benzoyl peroxide plus erythromycin C. Isotretinoin (Accutane) D. A change of occupation

C.Isotretinoin (Accutane) - Nodulocystic acne is the most severe form of acne vulgaris. They can be palpated and usually seen on the skin, although they actually are under the skin's surface. They develop when the follicle wall ruptures and leaks pus and cell contents in the dermis. The contaminated material infects adjoining follicles & the nodule developds. Isotretinoin, OCPs, & oral antibiotics are only treatment.

Shelley has esophageal cancer and asks you if alcohol played a part in its development. How do you respond? A. "Your cancer was caused by your cigarette smoking, nothing else." B. "Alcohol is also a carcinogen." C. "Alcohol directly alters the DNA and causes mutations." D. "Alcohol modifies the metabolism of carcinogens in the esophagus and increases their effectiveness."

D. "Alcohol modifies the metabolism of carcinogens in the esophagus and increases their effectiveness."

Henry is having difficulty getting rid of a corneal infection. He asks you why. How do you respond? A. "We can't determine the causative agent." B. "Antibiotics have difficulty getting to that area." C. "Because the infection was painless, it was not treated early enough." D. "Because the cornea doesn't have a blood supply, an infection can't be fought off as usual."

D. "Because the cornea doesn't have a blood supply, an infection can't be fought off as usual."

Suzanne has a 7 year old daughter who has had two recent infestations of lice. She asks you what she can do to prevent this. You respond, A. "After two days of no head lice, her bedding is lice free." B. "Boys are more susceptible, so watch out for her brother also." C. "After several infestations, she is now immune and is no longer susceptible." D. "Don't let her share hats, combs, or brushes with anyone."

D. "Don't let her share hats, combs, or brushes with anyone."

You're teaching Mitch, age 18, about his tinea pedis. You know he doesn't understand your directions when he tells you which of the following? A. "I should dry between my toes every day." B. "I should wash my socks with bleach." C. "I should use an antifungal powder twice a day." D. "I should wear rubber shoes in the shower to prevent transmission to others."

D. "I should wear rubber shoes in the shower to prevent transmission to others."

You're teaching Mitch, age 18, about his tinea pedis. You know he doesn't understand your directions when he tells you which of the following? A. "I should dry between my toes every day." B. "I should wash my socks with bleach." C. "I should use an antifungal powder twice a day." D. "I should wear rubber shoes in the shower to prevent transmission to others."

D. "I should wear rubber shoes in the shower to prevent transmission to others." - Rubber or plastic-soled shoes can harbor the fungus and therefore should not be worn.

Menstruating women lose approximately how much iron per day? A. 5 mg B. 10 mg C. 0.1-1.6 mg D. 2 mg

D. 2 mg

Treatment for Stage I pressure ulcer may include A. An enzymatic preparation B. Systemic antibiotics C. Surgical treatment with muscle flaps D. A transparent, semipermeable membrane dressing

D. A transparent, semipermeable membrane dressing

Judy, age 67, complains of a sudden onset of impaired vision, severe eye pain, vomiting, and a headache. You diagnose the following condition and refer for urgent treatment A. Cataracts B. Macular degeneration C. Presbyopia D. Acute glaucoma

D. Acute glaucoma

Which of the following can trigger a lesion in the client with psoriasis? A. A headache B. Salt water C. Aspirin D. Alcohol

D. Alcohol

The total loss of hair on all parts of the body is referred to as A. Female pattern alopecia B. Alopecia atreata C. Alopecia totalis D. Alopecia universalis

D. Alopecia universalis

The total loss of hair on all parts of the body is referred to as A. Female pattern alopecia B. Alopecia atreata C. Alopecia totalis D. Alopecia universalis

D. Alopecia universalis

Marissa is going to have a splenectomy for her idiopathic thrombocytopenic purpura (ITP). Which of the following statements about this is true? A. Splenectomy produces permanent remission in 70-90% of children with ITP B. Splenectomy is a first-line option C. Vitamin B12 supplementation is essential D. Anticoagulant therapy is indicated postoperatively when the platelet count rises

D. Anticoagulant therapy is indicated postoperatively when the platelet count rises

Janine, age 29, has numerous transient lesions that come and go and is diagnosed with urticaria. What do you order? A. Aspirin B. NSAIDs C. Opioids D. Antihistamines

D. Antihistamines

A peak flow meter is used for patients who have: A. Chronic bronchitis B. Emphysema C. Pneumonia D. Asthma

D. Asthma

A client with a nutritional deficiency of vitamin C may have A. Dry skin and loss of skin color B. Thickened skin that is dry or rough C. Flaky skin, sores in the mouth, and cracks at the corners of the mouth D. Bleeding gums and delayed wound healing

D. Bleeding gums and delayed wound healing

Which of the following is a "trigger" that may aggravate rocacea? A. Nonalcoholic beer B. Salt C. Mild exercise D. Caffeine

D. Caffeine

Adverse effects from prolonged or high-potency topical corticosteroids use to an open lesion may include A. Epidermal proliferation B. Striae C. Vitiligo D. Easy bruisability

D. Easy bruisability

Allergic rhinitis results from which immunoglobulin-mediated type 1 hypersensitivity? A. IgA B. IgD C. IgG D. IgE

D. IgE

Where is the epitrochlear lymph node located? A. In front of the ear B. Halfway between the angle and the tip of the mandible C. In the posterior triangle along the edge of the trapezius muscle D. In the inner condyle of the humerus

D. In the inner condyle of the humerus

When examining the newborn, what is the minimum number of cafe-au-lait spots that should be of concern? A. Any number B. 1-3 C. 4-5 D. More than 5

D. More than 5

You've been counseling your client about her asthma. You realize she doesn't understand your suggestions when she tells you that she'll do which of the following: A. Cover the mattress and pillows in airtight, dust-proof covers. B. Wash the bedding weekly and dry it on a hot setting for 20 minutes. C. Avoid sleeping on natural fibers such as wool or down. D. Open the windows and air out the room daily.

D. Open windows and air out the room daily. - Opening the windows daily would allow allergens to enter.

A common cause of conductive hearing loss in adults age 20-40 is A. Trauma B. Otitis media C. Presbycusis D. Otosclerosis

D. Otosclerosis

Frank, a 66 year old white male who is on diuretic therapy, presents with an elevated hematocrit. He also has splenomegaly on examination, as well as subjective complaints of blurred vision, fatigue, headache, and tinnitus. You suspect A. Multiple myeloma B. Waldenstrom's macroglobulinemia C. Dehydration related to use of diuretics D. Polycythemia vera

D. Polycythemia vera - An elevated hematocrit due to contracted plasma volume, rather than increased red blood cell mass, may be due to diuretic use or may occur without obvious cause.

The most common cause of sensoineural hearing loss is A. Trauma B. Tympanic membrane sclerosis and scarring C. Otosclerosis D. Presbycusis

D. Presbycusis

The test in which a small, radioactive tracer dose of cyanocobalamin is given by mouth an then a 24 hour urine sample is collected and assayed for radioactivity is the A. Coombs' test B. Oligonucleotide probe test C. Spherocytic test D. Schilling test

D. Shilling test

Which of the following is most typical of a patient with allergic rhinitis? A. Normal nasal turbinates B. Cough C. Post nasal drip D. Symptoms are associated with exposure to cats

D. Symptoms are associated with exposure to cats - Nasal turbinates in patient with allergic rhinitis tend to be pale, boggy, and often erythematous. They may appear blue. They are not normal in appearance. Cough may be due to post nasal drip, but is not universal. Post nasal drip is common but does not exist in all patients with allergic rhinitis.

Treatment options for uncomplicated hordeolum include all of the following except: A. erythromycin ophthalmic ointment B. warm compresses to the affected area C. incision and drainage D. oral antimicrobial therapy

D. oral antimicrobial therapy

A firm, painless, relatively fixed submandibular node would most likely be seen in the diagnosis of: A. herpes simplex B. acute otitis media (AOM) C. bacterial pharyngitis D. oral cancer

D. oral cancer

Treatment options in suppurative conjunctivits include all of the following ophthalmic preparations except: A. bacitracin-polymyxin B B. ciprofloxacin C. erythromycin D. penicillin

D. penicillin

You are trying to make a diagnosis of a lung problem described by your client Mr. Fry, age 65. In making the distinction between emphysema and chronic bronchitis, you know that chronic bronchitis A. usually presents with a mild cough B. has an insidious progressive dyspnea and usually occurs after age 50 C. demonstrates a markedly increased residual volume D. presents with wheezing and rhonci

D. presents with wheezing and rhonci

Subjective perception of altered equilibrium

Dizziness

Rhythmic oscillations of the eyes

Nystagmus

Perception of abnormal hearing or head noises

Tinnitus

Perception that the person or the environment is moving

Vertigo

An older patient has an initial presentation of symptomatic heart failure but is stable. He presents with increased heart rate, lower extremity edema, and mild shortness of breath with exertion. This patient has been given a diuretic in the office for volume overload. What other medication should be added today? a. ACE inhibitor b. Beta-blocker c. Calcium channel blocker d. Oxygen

a. ACE inhibitor

A key component of the approach to a patient who has atopic dermatitis is hydration. Which agent should be avoided? a. Lotions b. Creams c. Thick creams d. Ointments

a. Lotions

A patient is diagnosed with tinea pedis. A microscopic examination of the sample taken from the infected area would likely demonstrate: a. hyphae b. yeast c. spores d. a combination of hyphae and spores

a. hyphae

A 70 year-old African American male complains of pain in his back and trunk. Cardiovascular disease is ruled out. He has a normocytic normochromic anemia with hypercalcemia. A likely diagnosis is: a. multiple myeloma b. lymphoma c. leukemia d. prostate cancer

a. multiple myeloma

The viral exanthem of Koplik's spots is present in A. Rubeola B. Rubella C. Fifth disease D. Varicella

a. rubeola

The risk of HIV transmission is increased: a. when other STDs are present b. in females c. when patients are aware of their HIV status d. in patients with diabetes

a. when other STDs are present

To enhance the potency of a topical corticosteroid, the presciber recommends that the patient apply the preparation: A. to dry skin by gentle rubbing B. and cover with an occlusive dressing C. before bathing D. with an emollient

and cover with an occlusive dressing

The usual age for vision screening in young children takes place at: a. 2 years b. 3 years c. 4 years d. 5 years

b. 3 years

A topical treatment for basal cell carcinoma is: a. sulfacetamide lotion b. 5-fluorouracil c. tetracycline lotion d. trichloroacetic acid

b. 5-fluorouracil

How should a 20 year-old college age student be screened for TB? He presents with cough, night sweats, and weight loss. a. A chest x-ray b. A TB skin test c. A sputum specimen d. Questionnaire about symptoms

b. A TB skin test

A patient presents with symptoms of the flu during flu season. How should he be screened for the flu? a. A nasal culture b. A nasal swab c. CBC d. Based on symptoms

b. A nasal swab

A patient with diarrhea is tested for C. difficile. How soon should the enzyme immunoassay (EIA) yield results? a. Within 20 minutes b. About 24 hours c. About 3 days d. Less than one week

b. About 24 hours

A patient with COPD has been using albuterol with good relief of his shortness of breath. He is using it 3-4 times daily over the past 4 weeks. How should the NP manage this? a. Encourage its use b. Add a long acting anticholinergic c. Tell him to use it only once daily d. Add an oral steroid

b. Add a long acting anticholinergic

A 6 year-old has been diagnosed with thrush. Which drug should be used to treat him? a. Doxycyline b. Amoxicillin c. Azithromycin d. Cephalexin

b. Amoxicillin

A 3 year-old healthy child is diagnosed with pneumonia. He is febrile but in no distress. What is the preferred treatment for him? a. Supportive measures, it is probably viral b. Amoxicillin, doses 80-100 mg/kg/d c. Azithromycin d. Doxycycline

b. Amoxicillin, doses 80-100 mg/kg/d

What statement below describes anemia in older adults? a. An evaluation should take place in older adults who present with hemoglobin less than 14g/dL b. Anemias may have more than one origin and co-exist in older adults c. Faster return of hemoglobin to normal states in older adults d. An evaluation should take place in an older adult who presents with an increase in hemoglobin > 1g/dl within one year

b. Anemias may have more than one origin and co-exist in older adults

Which of the following will decrease the risk of acute otitis media in a 6 month old? a. Cigarette smoke exposure b. Breastfeeding c. Sucking on pacifiers d. Vitamin D supplementation

b. Breastfeeding

A patient exhibits petechiae on both lower legs but has no other complaints. How should the NP proceed? a. Refer to hematology b. Order a CBC c. Order blood cultures d. Stop aspirin and re-assess in one week

b. Order a CBC

A 70 year-old presents urgently to the nurse practitioner clinic with angioedema. This began less than an hour ago. He is breathing without difficulty. What medication may have caused this? a. Aspirin b. Ramipril c. Penicillin d. Metformin

b. Ramipril

The most common place for basal cell carcinoma to be found is the: a. scalp b. face c. anterior shin d. upper posterior back

b. face

The nurse practitioner identifies satellite lesions in a 6 month-old infant. These are: a. worrisome in an infant this age b. indicative of candidal infection c. found in the axilla bilaterally d. completely benign lesions present at birth

b. indicative of candidal infection

The rubella vaccine is contraindicated in pregnant women because: a. it can cause rubella in the mother b. it can cause rubella in the infant c. it does not cross the placenta d. neurological toxicity may occur in the mother

b. it can cause rubella in the infant

A child with a sandpaper textured rash probably has: a. rubeola b. strept infection c. varicella d. roseola

b. strept infection

An infant is diagnosed with diaper dermatitis. Satellite lesions are visible. This should be treated with: a. a moisture barrier like zinc oxide b. topical anti-fungal agent c. a topical antibacterial agent d. a low potency steroid cream

b. topical anti-fungal agent

Most cases of atopic dermatitis exacerbations are treated with: a. emollients b. topical steroids c. antihistamines d. antibiotics

b. topical steroids

Conjunctivitis: a. produces blurred vision in the affected eye b. usually begins as a viral infection c. results in anterior cervical lymphadenopahty d. is common in patients who are nearsighted

b. usually begins as a viral infection

Clinical features of bullous impetigo include: A. intense itch B. vesicular lesions C. dermatomal pattern D. systemic symptoms such as fever and chills

b. vesicular lesions

A 2 month-old is diagnosed with thrush. A microscopic exam of this patient's saliva demonstrates: a. hyphae b. yeast c. spores d. a combination of hyphae and yeast

b. yeast

A 35 year-old patient who is HIV positive is diagnosed with thrush. A microscopic exam of this patient's saliva demonstrates: a. hyphae b. yeast c. spores d. a combination of hyphae and yeast

b. yeast

A patient is diagnosed with thrush. A microscopic exam of this patient's saliva demonstrates: a. hypahae b. yeast c. spores d. a combination of hyphae and yeast

b. yeast

How long should an 18 month-old with acute otitis media be treated with an antibiotic? a. 5 days b. 5-7 days c. 10 days d. Until the erythema has resolved

c. 10 days

At what age should screening for oral health begin? a. At birth b. 3 months c. 6 months d. 1 year

c. 6 months

A 3 year-old has been diagnosed with acute otitis media. She is penicillin allergic (a non-Type 1 hypersensitivity reaction). How should she be managed? a. Amoxicillin b. Amoxicillin-clavulanate c. A cephalosporin d. A macrolide

c. A cephalosporin

A 6 year-old complains that his legs hurt. His mother states that he has complained for past 2 weeks, and she thought it was from "playing too hard". When asked to identify the painful areas, the child points to the midshaft of the femurs. He grimaces slightly when asked to walk. What should be part of the differential diagnosis? a. Osgood-Schlatter disease b. Growing pains c. Acute lymphocytic leukemia (ALL) d. Psychogenic pain

c. Acute lymphocytic leukemia (ALL)

In a patient who is diagnosed with mastoiditis, which of the following is most likely? a. Recent history of pharyngitis b. Fever, cough c. Displaced pinna d. Nuchal rigidity

c. Displaced pinna

Patient with asthma: a. all wheeze b. all cough c. can cough or wheeze d. have dyspnea

c. can cough or wheeze

A patient demonstrates leukocytosis. This means: a.. he has a bacterial infection b. he has a viral infection c. he has an infection of unknown origin d. he does not have an infection

c. he has an infection of unknown origin

A patient is in the clinic with a 36 hour history of diarrhea and moderate dehydration. Interventions should include: a. oral rehydration with tea, cola, or Gatorade b. IV rehydration c. oral rehydration with an electrolyte replenishment solution d. resumption of usual fluid intake and solid foods intake

c. oral rehydration with an electrolyte replenishment solution

A 4 year-old presents with fever, rhinorrhea, and paroxysmal, high-pitched whooping cough. This is: a. bronchiolitis b. croup c. pertussis d. epiglottitis

c. pertussis

The most common sequela of the flu in older adults is: a. meningitis b. hepatitis c. pneumonia d. myocardial infarction

c. pneumonia

A patient with a primary case of scabies was probably infected: a. 1-3 days ago b. 1 week ago c. 2 weeks ago d. 3-4 weeks ago

d. 3-4 weeks ago

When does a child's vision approximate 20/20? a. 2 years b. 3 years c. 4 years d. 5-6 years

d. 5-6 years

The main difference between cellulitis and erysipelas is the: a. organism b. length of time that infection lasts c. treatment d. area involved

d. area involved

A 3 year-old has fluid in the middle ear that does not appear infected. The eardrum appears normal. This is referred to as: a. serous otitis media b. acute otitis media c. otitis media with effusion d. middle ear effusion

d. middle ear effusion

A patient was exposed to HIV through sexual intercourse. He should be followed with screening tests to identify seroconversion for: a. 4-6 weeks b. 3-4 months c. about 6 months d. one year

d. one year

How many grams of a topical cream or ointment are needed for a single application to the hands? A. 1 B. 2 C. 3 D. 4

B. 2 grams

A burn that is about twice as large as an adult's palmar surface of the hand encompasses a BSA of approximately: A. 1 B. 2 C. 3 D. 4

B. 2%

The incubation period for S. pyogenes is usually: A. 1 to 3 days B. 3 to 5 days C. 6 to 9 days D. 10 to 13 days

B. 3 to 5 days

How many grams of a topical cream or ointment are needed for a single application to the entire body? A. 10-30 B. 30 -60 C. 60-90 D. 90-120

B. 30 to 60 grams

Which percentage of individuals for whom it is indicated typically receive the pneumococcus vaccine? A. 10% B. 30% C. 60% D. 90%

B. 30%

Candidiasis may occur in many parts of the body. James, age 29, has it in the glans of his penis. What is your diagnosis? A. Balanitis B. Thrush C. Candidal paronychia D. Subungal candida

A. Balanitis

Which type of skin cancer is most common in adults? A. Basal cell B. Squamous cell C. Melanoma D. Actinic keratosis

A. Basal cell

Jennifer, age 32, has genital warts (condylomata) and would like to have them treated. All of the following could be applied except A. Benzoyl peroxide B. Podophyllin C. Trichloroacetic acid D. Liquid nitorgen

A. Benzoyl peroxide

What is the normal respiratory rate of an 18 month old child while awake? A. 58-75 breaths per minute B. 30-40 breaths per minute C. 23-42 breaths per minute D. 19-36 breaths per minute

B. 30-40 breaths per minute

You observe a mother showing her infant a toy. You note that the infant can fixate on, briefly follow, and then reach for the toy. You suspect the infant is A. 2 months old B. 4 months old C. 6 months old D. 8 months old

B. 4 months old

What percentage of persons who smoke one pack of cigarettes per day or more have a cough? A. 10%-25% B. 40%-60% C. 75% D. 100%

B. 40%-60%

Samantha is being given platelets because of acute leukemia. One "pack" of platelets should raise her count by how much? A. 2,000-4,000 mm3 B. 5,000-8,000 mm3 C. 9,000-12,000 mm3 D. About 15,000 mm3

B. 5,000-8,000 mm3 - One pack equals about 50ml.

Treatment options for actinic keratosis include topical: A. hydrocortisone B. 5-fluorouracil C. acyclovir D. doxepin

B. 5-fluorouracil

Previous full term pregnancies, even if uncomplicated, may cause a woman to lose approximately how much iron? A. 1500-1700 mg B. 500-1000 mg C. 250-500 mg D. 1200-1500 mg

B. 500-1000 mg

Persons requiring home oxygen will have an oxygen saturation below A. 90% B. 85% C. 80% D. 75%

B. 85%

You examine a patient with red, tender thermal burn that has excellent capillary refill involving the entire surface of the anterior right thigh. The estimated involved body surface area is approximately: A. 5% B. 9% C. 13% D. 18%

B. 9%

Which patient might be expected to have the worst FEV1? A. An asthma patient in the green zone B. A 65 year old with emphysema C. A 60 year old with pneumonia D. A patient with a B12 deficiency

B. A 65 year old with emphysema - FEV1 stands for "forced expiratory volume in one second".

Your client is unable to differentiate between sharp and dull stimulation on both sides of her face. You suspect A. Bell's palsy B. A lesion affecting the trigeminal nerve C. A stroke D. Shingles

B. A lesion affecting the trigeminal nerve

Mark, age 18, has a persistent sore throat, fever, and malaise not relieved with penicillin therapy. What do you order next? A. A throat culture B. A monospot test C. A rapid antigen test D. A Thayer-Martin plate test

B. A monospot test

Screening infants for anemia should occur at what age? A. 6 months B. No screening is recommended C. 9 months D. 12 months

C. 9 months - The cutoff points for a diagnosis of anema at this age are a hemoglobin below 11g/dl or a hematocrit below 33%.

Shelley, age 47, is complaining of red eye. You are trying to decide between a diagnosis of conjunctivitis and iritis. One distinguishing characteristic between the two is A. Eye discomfort B. Slow progression C. A ciliary flush D. No change in or slightly blurred vision

C. A ciliary flush

Which method can be safely used to remove cerumen in a 12 month old child's ear? A. A size 2 ear curette B. Irrigation using hot water from a 3cc syringe C. A commercial jet tooth cleaner D. Cerumen should not be removed from a child this young

C. A commercial jet tooth cleaner - Only if no suspicion of perforated tympanic membrane

The "gold standard" for definitive dianosis of sickle cell anemia is A. A reticulocyte count B. The sickle cell test C. A hemoglobin electrophoresis D. A peripheral blood smear

C. A hemoglobin electrophoresis - Test that determines the presence of hemoglobin S

Jamie, age 6, was bitten by a dog. Her mother asks you if the child needs antirabies treatment. You tell her, A. "If the dog was a domestic pet that had been vaccinated, the wound should be cleaned and irrigated." B. "Antirabies treatment must be started immediately." C. "Rabies can be contracted only through the bites of wild animals." D. "Wait until you have observed the animal for 2 weeks to determine if it is rabid."

A. "If the dog was a domestic pet that had been vaccinated, the wound should be cleaned and irrigated."

Your 24 year old client, whose varicella rash just erupted yesterday, asks you when she can go back to work. What do you tell her? A. "Once all the vesicles are crusted over." B. "When the rash is entirely gone." C. "Once you have been on medication for at least 48 hours." D. "Now, as long as you stay away from children and pregnant women."

A. "Once all the vesicles are crusted over."

Sam, age 4, is brought into the clinic by his father. His tympanic membrane is perforated from otitis media. His father asks about repair of the eardrum. How do you respond? A. "The eardrum, in most cases, heals within several weeks." B. "We need to schedule Sam for a surgical repair." C. "He must absolutely stay out of water for 3-6 months." D. "If the eardrum is not healed in several months, it can be surgically repaired."

A. "The eardrum, in most cases, heals within several weeks."

Of the nearly 46 million adults who smoke, 34% try to quit each year, but how many actually succeed? A. 2.5% B. 10% C. 20% D. 50%

A. 2.5%

You see a kindergartner with impetigo and advise that she can return ____ hours after initiating effective antimicrobial therapy: A. 24 B. 48 C. 72 D. 96

A. 24

The normal ratio of the artery-to-vein width in the retina as viewed through the ophthalmoscope is A. 2:3 B. 3:2 C. 1:3 D. 3:1

A. 2:3

What would be the 1 minute Apgar score for a newborn in good condition who needs only suctioning of the nose and mouth and otherwise routine care? A. 7-10 B. 6-8 C. 3-6 D. 0-2

A. 7-10

Sylvia has glaucoma and has started taking a medication that acts as a diuretic to reduce the intraocular pressure. Which medication is she taking? A. A carbonic anhydrase inhibitor B. A beta-adrenergic receptor blocker C. A miotic D. A mydriatic

A. A carbonic anhydrase inhibitor

The most common cause of a white pupil (leukokoria or leukocoria) in a newborn is A. A cataract B. Retinoblastoma C. Persistent hyperplastic primary vitreous D. Retinal detachment

A. A cataract - Some infants require no treatment, however, surgery may be performed

When you are assessing the internal structure of the eye, absence of a red reflex may indicate A. A cataract or a hemorrhage into the vitreous humor B. Acute iritis C. Nothing, this is a normal finding in older adults D. Diabetes or long-standing hypertension

A. A cataract or a hemorrhage into the vitreous humor

Silas, age 82, comes to your office with a failry new colostomy. Around the stoma he has a papular rash with satellite lesions. What does this indicate? A. A fungal infection, usually candida albicans B. An allergic reaction to the appliance C. A normal reaction to fecal drainage D. A fluid volume deficit

A. A fungal infection, usually Candida albicans - May be a consequence of persistent skin moisture or an adverse effect of antibiotic therapy

What is an excessive amount of collagen that develops during scar formation called? A. A keloid B. A skin tag C. An angioma D. A keratosis

A. A keloid

The most frequent cause of laryngeal obstruction in an adult is A. A piece of meat B. A tumor C. Mucosal swelling from an allergic reaction D. Inhalation of a carcinogen

A. A piece of meat

Susie, age 10, has a cough that characteristically occurs all day long, but never during sleep. You suspect: A. A psychogenic cough (or habit) B. Allergic rhinitis C. Pertussis D. Postnasal drip

A. A psychogenic cough (or habit)

Jill presents with symptoms of hay fever and you assess the nasal mucosa of her turbinates to be pale. What diagnosis do you suspect? A. Allergic rhinitis B. Viral rhinitis C. Nasal polyps D. Nasal vestibulitis from folliculitis

A. Allergic rhinitis

Which tumor marker may detect a tumor of the ovary or testes? A. Alpha fetoprotein B. Carcinoembryonic antigen C. Human chorionic gonadotropin D. Cancer antigen 125

A. Alpha fetoprotein - May be elevated with embryonal cell tumors of the ovary or testis, hepatocellular carcinoma, and choriocarcinoma.

Which of the following medications will not increase the likelihood of photosensitivity? A. Amoxicillin B. Sulfa drugs C. Fluoroquinolones D. Doxycycline

A. Amoxicillin

A basal cell carcinoma is A. An epithelial tumor that originates from either the basal layer of the epidermis or cells in the surrounding dermal structures B. A malignant tumor of the squamous epithelium of the skin or mucous membranes C. An overgrowth and thickening of the cornified epithelium D. Lined with epithelium and contains fluid or a semisolid material

A. An epithelial tumor that originates from either the basal layer of the epidermis or cells in the surrounding dermal structures

Physiological changes in the immune system of older adults include A. An increase in immunoglobulin A and G antibodies B. A high rate of T-lymphocyte proliferation C. An increase in the number of cytotoxic T cells D. An increase in CD8, which affects regulation of the immune system

A. An increase in immunoglobulin A and G antibodies

What is the most effective treatment for urticaria? A. An oral antihistamine B. Dietary management C. Avoidance of the offending trigger D. A glucocorticosteroid

A. An oral antihistamine

The well-established risk factor(s) for a nosocomial pneumonia caused by a multidrug-resistant organism is(are): A. Antibiotic exposure and a hospital stay of more than 1 week B. Age greater than 65 and having COPD C. Having outpatient surgery D. Having allergies to multiple antibiotics

A. Antibiotic exposure and a hospital stay of more than 1 week.

Manny, age 16, was hit in the eye with a baseball. He developed pain in the eye, decreased visual acuity, and injection of the globe. You confirm the diagnosis of hyphema by finding blood in the anterior chamber. What treatment would you recommend while Manny is waiting to see the ophthalmologist? A. Apply bilateral eye patches B. Have Manny lie flat C. Refer him to an ophthalmologist within a week D. Make sure Manny is able to be awakened every 30 minutes

A. Apply bilateral eye patches

Your client, Mr. Jones, has Sjorgen's syndrome. Which treatment do you suggest? A. Artificial tears and chewing sugarless gum B. Frequent rinsing out of the mouth with mouthwash C. Drinking at least one glass of milk per day D. Removing wax from the ears at regular intervals

A. Artificial tears and chewing sugarless gum - Sjorgen's syndrome is a multisystem autoimmune disease characterized by dysfunction of the exocrine glands, specifically notable for dry eyes and dry mouth.

Theophylline when given for COPD acts: A. As a bronchodilator B. By decreasing and strengthening respirations C. To impede mucociliary clearance D. With a negative inotropic effect.

A. As a bronchodilator - Theophylline, a methylxanthine derivative, acts as a bronchodilator, decreases dyspnea, improves mucociliary clearance, improves gas exchange, enhances respiratory muscle performance, increases neuroinspiratory drive, and has a positive inotropic effect.

Janice states that her son is allergic to eggs and she heard that he should not receive the flu vaccine. How do you respond? A. "Although measles, mumps, rubella, and influenza vaccines contain a minute amount of egg, most egg-allergic individuals can tolerate these vaccines without any problems." B. "Most of the allergic reactions are caused by the actual vaccinations; therefore, a skin test should be done first." C. You're right. We should not give this vaccination to your son." D. "He should not have a skin test done if he has this allergy because a serious cellulitis may occur at the site of testing."

A. "Although measles, mumps, rubella, and influenza vaccines contain a minute amount of egg, most egg-allergic individuals can tolerate these vaccines without any problems." - Many of the allergic reactions are caused by the gelatins in the vaccinations and not the actual vaccinations

The ABCDEs of melanoma identification include which of the following? A. Asymmetry: one half does not match the other half B. Border: the borders are regular; they are not ragged, notched, or blurred C. Color (pigmentation) is uniform D. Diameter: the diameter is greater than 10mm

A. Asymmetry: one half does not match the other half

Martha is experiencing an acute clinical syndrome characterized by fever, night sweats, lethargy and malaise, myalgias, arthralgias, general lymphadenopathy, pharyngitis, maculopapular rash, and a headache. You suspect this to be an acute primary infection that usually follows exposure to HIV. How many weeks ago do you think she was exposed to HIV? A. 1 week B. 2-4 weeks C. 1-2 months D. 2-4 months

B. 2-4 weeks

Which of the following topical preparations should not be used in intertriginous areas or in the perineum because they increase maceration? A. Gel solutions B. Ointments C. Emollient creams D. Powders

B Ointments

When Judith, age 15, asks you to explain the 20/50 vision in her right eye, you respond A. "You can see at 20ft with your left eye what the normal person can see at 50ft." B. "You can see at 20ft with your right eye what the normal person can see at 50ft." C. "You can see at 50ft with your right eye what the normal person can see at 20ft." D. "You can see at 50ft with the left eye what the normal person can see at 20ft."

B. "You can see at 20ft with your right eye what the normal person can see at 50ft." - If vision is greater than 20/30, refer the client to an ophthalmologist or optometrist

Patients with strep throat can be cleared to return to work or school after __ hours of antimicrobial therapy. A. 12 B. 24 C. 36 D. 48

B. 24

Susie, age 6 months, has a candida infection in the diaper area. What do you suggest to the mother? A. "Use rubber or plastic pants to contain the infection and prevent it from getting to the thighs." B. "Keep the area as dry as possible." C. "Use baby powder with cornstarch." D. "Keep Susie away from other babies until the infection is cleared up."

B. "Keep the area as dry as possible." - Clients must be taught to decrease favorable environmental conditions for candida such as moisture, warmth and poor air circulation.

What is the normal ratio of the anteroposterior chest diameter to the transverse chest diameter? A. 1:1 B. 1:2 C. 2:3 D. 2:1

B. 1:2 - An anteroposterior measurement that equals the transverse measurement is defined as a barrel chest, which usually indicates some obstructive lung disorder.

Which of the following medications commonly prescribed for tuberculosis cannot be taken by pregnant women? A. Isoniazid (INH) B. Rifampin (RIF) C. Pyrazinamide (PZA) D. Ethambutol (EMB)

C Pyrazinamide (PZA) - Pyrazinamide should not be taken by pregnant women. Active tuberculosis should be treated with isoniazid and ethambutol and continued for at least 18 months to prevent relapse..

Tina, age 49, is on multiple drug therapy for tuberculosis. She asks you how long she needs to take the drugs. You respond: A. 6 weeks to 2 months B. 4-6 months C. 6-9 months D. 1 year

C. 6-9 months

A 25-year-old woman who has seasonal allergic rhinitis likes to spend time outdoors. She asks you when the pollen count is likely to be the lowest. You respond: A. "Early in the morning" B. "During breezy times of the day" C. "After a rain shower" D. "When the sky is overcast"

C. "After a rain shower"

Harry uses a high-potency corticosteroid cream for his dermatoses. You tell him the following A. "You must use this for an extended period of time for it to be effective." B. "It will work better if you occlude the lesion." C. "It may exacerbate you concurrent condition of tinea corporis." D. "Be sure to use it daily."

C. "It may exacerbate your concurrent condition of tinea corporis."

Client teaching is an integral part of successfully treating pediculosis. Which of the following statements would you incorporate in your teaching plan? A. "It's ok to resume sharing combs, headsets, etc. after being lice free for 1 month." B. "Soak your combs and brushes in rubbing alcohol for 8 hours." C. "Itching may continue after successful treatment for up to a week." D. "Spraying of pesticides in the immediate environment is essential to prevent recurrence."

C. "Itching may continue after successful treatment for up to a week."

You examine a patient who has had poison ivy for 3 days. She asks if she can spread it to her family members. You reply: A. "Yes, but not after crusting has occurred." B. "Yes, the fluid in the blister can transmit it." C. "No, transmission does not occur from the blister's contents." D. "No, you've progressed beyond that stage."

C. "No, transmission does not occur from the blister's contents."

Mattie says she has heard that it is not good to let a baby go to bed with a bottle. She says that she has always done this with her other children and wonders why it is not recommended. How do you respond? A. "A bottle in the baby's mouth forces the baby to breathe through the nose. If the nose is clogged, the baby will not get enough oxygen." B. "A nipple, when placed in the mouth for long periods of time, can cause toothe displacement." C. "Normal mouth bacteria act on the sugar in the bottle contents to form acids, which will break down the tooth enamel and destroy the teeth even before they come in." D. "This encourages the baby to continually want to drink at night. When the child is older, it will become a habit, and the child will end up wearing diapers into the preschool years."

C. "Normal mouth bacteria act on sugar in the bottle contents to form acids, which will break down the tooth enamel and destroy the teeth even before they come in."

Allie, age 5, is being treated with radiation for cancer. Her mother asks about the effect radiation will have on Allie's future growth. Although she knows that a specialist will be handling Allie's care, her mother asks for your opinion. How do you respond? A. "Let's worry about the cancer first, then see how her growth is affected." B. "Chemotherapy may effect her futur growth, but not radiation." C. "She will probably have growth hormone problems, in which case she can then begin growth hormone therapy." D. That's the least of your worries now; everything will turn out ok."

C. "She will probably have growth hormone problems, in which case she can then begin growth hormone therapy."

Natasha, age 4, has amblyopia. How do you respond when her mother asks about treatment? A. "We'll wait until she's 7 years old before starting treatment." B. "Treatment needs to be started now. We'll cover her 'bad' eye." C. "Treatment needs to be started now. We'll cover her 'good' eye." D. "No treatment is necessary. She'll outgrow this."

C. "Treatment needs to be started now. We'll cover her 'good' eye."

Mandy's 16 year old daughter has hepatitis A. Which of the following statements made by Mandy indicates that she understands the teaching you've just completed? A. "I guess she needs to be hospitalized until she's recovered." B. "We'll keep her at home with strict isolation precautions." C. "We'll stop at the store and buy plastic eating utensils." D. "We'll stop at the drugstore and pick up prescription medications immediately."

C. "We'll stop at the store and buy plastic eating utensils." - Clients with Hepatitis A should have separate eating and drinking utensils or use disposable ones.

Maria asks if being overweight predisposes her to cancer. How do you respond? A. "No, you have the came risk as a normal-weight individual." B. "You have less of a risk of cancer than normal-weight individuals because you have protein stores to combat mutant cells." C. "Yes, you have an increased risk of hormone-dependent cancers because of your obesity." D. "Yes, you have an increased risk because you have many more cells in all the organs of your body."

C. "Yes, you have an increased risk of hormone-dependent cancers because of your obesity." - Because of excessive body fat. Sex hormones are synthesized from fat, and obese people have excessive amounts of the hormones that feed hormone-dependent malignancies such as cancer of the breast, bowel, ovary, endometrium, and prostate.

How do you respond when Diane, age 29, asks why she gets sores on her lips every time she sits out in the sun for an extended period of time? A. "You are allergic to the sun and must wear sunblock on your lips." B. "Your lips are dry to begin with and you must keep them moist at all times." C. "You have herpes simplex that recurs with sunlight exposure." D. "You're probably allergic to your lip balm."

C. "You have herpes simplex that recurs with sunlight exposure."

Joan had a modified mastectomy with radiation therapy 10 years ago. She asks when she can have her blood pressure or needle sticks taken in the affected arm. How do you respond? A. "If it's been 10 years and you've had no problems, you can discontinue those precautions." B. "Because you didn't have a radical mastectomy, you can do those things now." C. "You must observe these precautions forever." D. "As long as you do limb exercises and have established collateral drainage, you can discontinue these precautions."

C. "You must observe these precautions forever." - Lymphadema may occur many years after a mastectomy (whether radical or modified) or radiation therapy on the affected side.

Marie asks what she can do for Sarah, her 90 year old mother, who has extremely dry skin. You respond A. "After bathing every day, use a generous supply of moisturizers." B. "Use a special moisturizing soap every day." C. "Your mother does not need a bath every day." D. "Increase your mother's intake of fluids."

C. "Your mother does not need a bath every day."

Sandra, age 19, is pregnant. She is complaining of breathlessness, tiredness, and weakness and is pale. After diagnosing anemia, you order medication and tell her to take it A. Only with meals because it can be irritating to the stomach B. In the morning if she experiences morning sickness C. 1 hour before eating or between meals D. At bedtime

C. 1 hour before eating or between meals - Sandra's symptoms indicate anemia, which is probably caused by a poor diet. She probably has an iron and folic acid deficiency.

A child's head circumference is routinely measured at each well-child visit until age A. 12 months B. 18 months C. 2 years D. 5 years

C. 2 years

How many grams of a topical cream or ointment are needed for a single application to an arm? A. 1 B. 2 C. 3 D. 4

C. 3 grams

Sickle cell anemia affects African Americans. Approximately 1 in 400 African Americans in the United States has sickle cell disease (SCD). Advances in treatment have been made, but life expectancy is still limited. The mean survival time for men with the disease is approximately A. 24 years B. 34 years C. 42 years D. 52 years

C. 42 years - 42 for men and 48 for women

Your client had a colostomy several weeks ago and is having difficulty finding a permanent appliance that fits. How long do you tell him to wait for the stoma to shrink before buying a permanent appliance? A. 2-4 weeks B. 4-6 weeks C.6-8 weeks D. Just over 2 months

C. 6-8 weeks

Your client had a colostomy several weeks ago and is having difficulty finding a permanent appliance that fits. How long do you tell him to wait for the stoma to shrink before buying a permanent appliance? A. 2-4 weeks B. 4-6 weeks C.6-8 weeks D. Just over 2 months

C. 6-8 weeks - Before 6-8 weeks, the stoma needs to be measured weekly to find a well-fitting appliance

Barbie, age 27, had her spleen removed after an automobile accident. You are seeing her in the office for the first time since her discharge from the hospital. She asks you how her surgery will affect her in the future. How do you respond? A. "Your red blood cell production will be slowed." B. "Your lymphatic system may have difficulty transporting lymph fluid to the blood vessels." C. "You'll have difficulty storing the nutritional agents needed to make red blood cells." D. "You may have difficulty salvaging iron from old red blood cells for use."

D. "You may have difficulty salvaging iron from old red blood cells for use."

Mandy was given a diagnosis of flu 2 days ago and wants to start on the "new flu medicine" right away. What do you tell her? A. "The medication is effective only if started within the first 48 hours after the symptoms begin." B. "If you treat a cold, it goes away in 7 days; if you don't treat it, it goes away in 1 week." C. "The medicine has not proven its effectiveness." D. "I'll start you on zanamivir today. It may shorten the course of the disease and perhaps lessen the severity of your symptoms."

D. "I'll start you on zanamivir today. It may shorten the course of the disease and perhaps lessen the severity of your symptoms."

Susan states that her fiance has been frostbitten on the nose while skiing and is fearful that it will happen again. What do you tell her? A. "Don't worry - as long as he gets medical help the first few hours after being frostbitten again, he'll recover." B. "Once frostbitten, he should not go skiing again." C. "If it should happen again, massage the nose with a dry hand." D. "Infarction and necrosis of the affected tissue can happen with repeated frostbite."

D. "Infarction and necrosis of the affected tissue can happen with repeated frostbite."

Barbara, age 72, states that she was told she had atrophic macular degeneration and asks you if there is any treatment. How do you respond? A. "No, but 5 years from the time of the first symptoms, the process usually stops." B. "Yes, there is a surgical procedure that will cure this." C. "If we start medications now, they may prevent any further damage." D. "Unfortunately, there is no effective treatment, but I can refer you to a rehabilitation agency that can help you adjust to the visual loss."

D. "Unfortunately, there is no effective treatment, but I can refer you to a rehabilitation agency that can help you adjust to the visual loss."

Nancy recently had a mastectomy and refuses to look at the site. Her husband does all the dressing changes. When she comes into the office for a postoperative checkup, what would you say to her? A. "You'll look at it when you're ready." B. "You must look at it today." C. "Everything's going to be ok. It looks fine." D. "You have to accept this eventually; just glance at it today."

D. "You have to accept this eventually, just glance at it today." - You'll look at it when you're ready may mean that she will never be ready. It is better to take a matter of fact approach and attempt to have her at least glance at it.

Mr. Forrest has a positive purified protein derivative (PPD) result of 7 mm. Preventative therapy for tuberculosis should be administered for at least A. 10 weeks B. 12 weeks C. 10 months D. 12 months

D. 12 months


Conjuntos de estudio relacionados

Microsoft Certified: Azure Administrator Associate AZ103

View Set

9: PROSOCIAL BEHAVIOUR: HELPING OTHERS

View Set

Lesson 12: Applying Security Solutions for Cloud and Automation

View Set

Technical writing 2208-ENGL-2338-503

View Set

SHRM Learning System Practice Exam

View Set

Chapter 10 Meetings & One Person Corporation

View Set

Analytics in Operations Midterm 3

View Set